Solucionario Callen Termodinamica

  • Uploaded by: lexleo1000
  • 0
  • 0
  • January 2021
  • PDF

This document was uploaded by user and they confirmed that they have the permission to share it. If you are author or own the copyright of this book, please report to us by using this DMCA report form. Report DMCA


Overview

Download & View Solucionario Callen Termodinamica as PDF for free.

More details

  • Words: 41,314
  • Pages: 204
Loading documents preview...
UNIVERSIDAD DE CHILE Departamento de F´ısica Facultad de Ciencias ˜ noa Las Palmeras 3425, Nu˜ Santiago de Chile

Apuntes de un curso de ´ mica Termodina

RODRIGO FERRER P.

Editores

:

Felipe Gonz´alez Roberto Navarro LATEX

Contenidos I

Principios Generales de la Termodin´ amica Cl´ asica

1. POSTULADOS Y CONDICIONES DE EQUILIBRIO 1.1. Introducci´on . . . . . . . . . . . . . . . . . . . . . . . . . . . . . 1.2. Energ´ıa Interna U . . . . . . . . . . . . . . . . . . . . . . . . . . 1.2.1. Manipulaci´on de los sistemas termodin´amicos . . . . . . 1.2.2. La energ´ıa U es medible y controlable . . . . . . . . . . . 1.3. Unidades . . . . . . . . . . . . . . . . . . . . . . . . . . . . . . . 1.4. Par´ametros Extensivos . . . . . . . . . . . . . . . . . . . . . . . 1.5. Segundo postulado. Entrop´ıa . . . . . . . . . . . . . . . . . . . . 1.6. Relaci´on Fundamental . . . . . . . . . . . . . . . . . . . . . . . ´ 1.6.1. CONSECUENCIAS MATEMATICAS . . . . . . . . . . 1.7. Ecuaciones de Estado . . . . . . . . . . . . . . . . . . . . . . . . 1.7.1. De las Ecuaciones de Estado a la Ecuaci´on Fundamental 1.8. Concepto de Temperatura . . . . . . . . . . . . . . . . . . . . . 1.8.1. ¿De d´onde a d´onde fluye el calor? . . . . . . . . . . . . . 1.9. Equilibrio Mec´anico . . . . . . . . . . . . . . . . . . . . . . . . . ´ 2. GAS IDEAL MONOATOMICO 2.1. Propiedades del Gas Ideal Monoat´omico . . . . . . . . . 2.2. Expansi´on adiab´atica contra el vac´ıo . . . . . . . . . . . 2.3. Relaci´on de Euler . . . . . . . . . . . . . . . . . . . . . . 2.4. Relaci´on de Gibbs-Duhem . . . . . . . . . . . . . . . . . 2.5. De las Ecuaciones de Estado a la Ecuaci´on Fundamental 2.6. Temperatura del Gas Ideal . . . . . . . . . . . . . . . . .

3. CALORES ESPEC´IFICOS 3.1. Dos tipos de calores espec´ıficos y capacidades cal´oricas 3.2. Capacidades Cal´oricas . . . . . . . . . . . . . . . . . . 3.3. Relaciones Entre los Calores Espec´ıficos . . . . . . . . . 3.4. Problemas . . . . . . . . . . . . . . . . . . . . . . . . . 3.4.1. Equilibrio frente al flujo de materia . . . . . . . 3.4.2. Variaci´on de la temperatura en la atm´osfera . . 3.4.3. Experimento de R¨ uchardt (1929) . . . . . . . . 3.4.4. Cambios de temperatura . . . . . . . . . . . . . 3.4.5. Pistones M´oviles . . . . . . . . . . . . . . . . .

. . . . . . . . .

. . . . . .

. . . . . . . . .

. . . . . .

. . . . . . . . .

. . . . . .

. . . . . . . . .

. . . . . .

. . . . . . . . .

11 . . . . . . . . . . . . . .

. . . . . .

. . . . . . . . .

. . . . . . . . . . . . . .

. . . . . .

. . . . . . . . .

. . . . . . . . . . . . . .

. . . . . .

. . . . . . . . .

. . . . . . . . . . . . . .

. . . . . .

. . . . . . . . .

. . . . . . . . . . . . . .

. . . . . .

. . . . . . . . .

. . . . . . . . . . . . . .

. . . . . .

. . . . . . . . .

. . . . . . . . . . . . . .

. . . . . .

. . . . . . . . .

. . . . . . . . . . . . . .

. . . . . .

. . . . . . . . .

. . . . . . . . . . . . . .

. . . . . .

. . . . . . . . .

. . . . . . . . . . . . . .

. . . . . .

. . . . . . . . .

. . . . . . . . . . . . . .

13 13 14 15 16 21 21 22 22 23 29 30 34 35 35

. . . . . .

39 39 40 41 42 42 44

. . . . . . . . .

47 47 47 47 54 54 55 58 61 64

4

CONTENIDOS

4. PROCESOS REVERSIBLES E IRREVERSIBLES 67 4.1. Procesos Quasi-Est´aticos . . . . . . . . . . . . . . . . . . . . . . . . . . . . . . . . . 67 4.2. Definici´on de Reversibilidad . . . . . . . . . . . . . . . . . . . . . . . . . . . . . . . 69 4.2.1. Cambios reversibles e irreversibles de temperatura. . . . . . . . . . . . . . . 69 ´ 5. CICLOS TERMODINAMICOS 5.1. Ciclo de Carnot . . . . . . . . . . . . . . . . . . . . . 5.2. Ciclo de Carnot con Entrop´ıa versus la Temperatura. 5.3. Ciclo de Carnot con la Presi´on versus el Volumen. . . 5.4. Ciclo de Joule . . . . . . . . . . . . . . . . . . . . . . 5.5. Ciclo de Otto . . . . . . . . . . . . . . . . . . . . . . 5.5.1. Motor de Dos Tiempos . . . . . . . . . . . . . 5.6. Ciclo de Diesel . . . . . . . . . . . . . . . . . . . . . 5.7. Cooling Energy Ratio . . . . . . . . . . . . . . . . . . 5.8. Trabajo M´aximo . . . . . . . . . . . . . . . . . . . .

II

. . . . . . . . .

. . . . . . . . .

. . . . . . . . .

. . . . . . . . .

. . . . . . . . .

. . . . . . . . .

. . . . . . . . .

. . . . . . . . .

. . . . . . . . .

. . . . . . . . .

. . . . . . . . .

. . . . . . . . .

. . . . . . . . .

. . . . . . . . .

. . . . . . . . .

. . . . . . . . .

. . . . . . . . .

Teor´ıa Cin´ etica de los Gases

75 75 80 82 83 84 87 88 89 91

95

6. MOVIMIENTO DE LAS PART´ICULAS 97 6.1. Ecuaciones de estado de un gas ideal . . . . . . . . . . . . . . . . . . . . . . . . . . 97 6.2. Choques Contra una Pared . . . . . . . . . . . . . . . . . . . . . . . . . . . . . . . . 101 ´ 7. FORMULA DE MAXWELL 107 7.1. Deducci´on de la Ecuaci´on . . . . . . . . . . . . . . . . . . . . . . . . . . . . . . . . 107 7.2. Problemas . . . . . . . . . . . . . . . . . . . . . . . . . . . . . . . . . . . . . . . . . 115

III

Potenciales Termodin´ amicos

125

8. TRANSFORMACIONES DE LEGENDRE 127 8.1. El Principio de M´ınima Energ´ıa . . . . . . . . . . . . . . . . . . . . . . . . . . . . . 127 8.2. Transformaciones de Legendre . . . . . . . . . . . . . . . . . . . . . . . . . . . . . . 131 8.2.1. Transformaci´on de Legendre Inversa . . . . . . . . . . . . . . . . . . . . . . . 134 ´ 9. POTENCIALES TERMODINAMICOS 9.1. Potencial de Helmholtz . . . . . . . . . 9.2. Potencial Entalp´ıa . . . . . . . . . . . 9.3. Potencial de Gibbs . . . . . . . . . . . 9.4. Funciones Generalizadas de Massieu . . 9.5. Relaciones de Maxwell . . . . . . . . . 9.6. Problemas . . . . . . . . . . . . . . . .

. . . . . .

. . . . . .

. . . . . .

. . . . . .

. . . . . .

. . . . . .

. . . . . .

. . . . . .

. . . . . .

. . . . . .

. . . . . .

. . . . . .

. . . . . .

. . . . . .

. . . . . .

. . . . . .

. . . . . .

. . . . . .

. . . . . .

. . . . . .

. . . . . .

. . . . . .

. . . . . .

. . . . . .

. . . . . .

137 137 138 138 140 141 142

10. EL PRINCIPIO EXTREMAL 145 10.1. Principio de M´ınima Energ´ıa . . . . . . . . . . . . . . . . . . . . . . . . . . . . . . . 145 10.2. Principio Minimal Para la Energ´ıa Libre de Helmholtz . . . . . . . . . . . . . . . . 147 10.3. Principio de Entalp´ıa M´ınima . . . . . . . . . . . . . . . . . . . . . . . . . . . . . . 148

CONTENIDOS

Rodrigo Ferrer

5

10.4. El Proceso de Joule Thomson . . . . . . . . . . . . . . . . . . . . . . . . . . . . . . 150 ´ 11. EL CUADRADO TERMODINAMICO 11.1. Reducci´on de Derivadas . . . . . . . 11.2. Aplicaciones Simples . . . . . . . . . 11.2.1. Compresi´on Adiab´atica . . . . 11.2.2. Compresi´on Isot´ermica . . . . 11.2.3. Expansi´on Libre . . . . . . . . 11.2.4. 7.4-8, H. CALLEN . . . . . .

. . . . . .

. . . . . .

. . . . . .

. . . . . .

. . . . . .

. . . . . .

. . . . . .

. . . . . .

. . . . . .

. . . . . .

. . . . . .

. . . . . .

. . . . . .

. . . . . .

. . . . . .

. . . . . .

. . . . . .

. . . . . .

. . . . . .

. . . . . .

. . . . . .

. . . . . .

. . . . . .

. . . . . .

. . . . . .

. . . . . .

155 156 158 158 161 162 164

´ 12. ESTABILIDAD EN LOS SISTEMAS TERMODINAMICOS 167 12.1. Condiciones de Estabilidad para los Potenciales Termodin´amicos . . . . . . . . . . . 171

IV

Cambios de Fase

175

13. TRANSICIONES DE FASE DE PRIMER ORDEN 13.1. Ecuaci´on de la Presi´on de Vapor . . . . . . . . . . . 13.2. Curva de Coexistencia de Fases . . . . . . . . . . . 13.3. Ecuaci´on de Van der Waals . . . . . . . . . . . . . 13.4. Transiciones de Fase en Sistemas Monocomponentes 13.5. La Discontinuidad en el volumen . . . . . . . . . . 13.6. Discontinuidad en la Entrop´ıa . . . . . . . . . . . . 13.7. La Ecuaci´on de Clausis Clapeyron . . . . . . . . . .

V

. . . . . . .

. . . . . . .

. . . . . . .

. . . . . . .

. . . . . . .

. . . . . . .

. . . . . . .

. . . . . . .

. . . . . . .

. . . . . . .

. . . . . . .

. . . . . . .

. . . . . . .

. . . . . . .

. . . . . . .

. . . . . . .

. . . . . . .

. . . . . . .

Ap´ endice

´ A. TEOREMAS MATEMATICOS n A.1. Continuidad en R . . . . . A.2. Expansi´on de Taylor . . . . A.3. Funciones Compuestas . . . A.4. Funciones Impl´ıcitas . . . .

177 177 178 181 184 188 189 194

197 . . . .

. . . .

. . . .

. . . .

. . . .

. . . .

. . . .

. . . .

. . . .

. . . .

. . . .

. . . .

´ DE UNIDADES B. CONSTANTES Y CONVERSION

. . . .

. . . .

. . . .

. . . .

. . . .

. . . .

. . . .

. . . .

. . . .

. . . .

. . . .

. . . .

. . . .

. . . .

. . . .

. . . .

. . . .

. . . .

. . . .

199 199 200 200 200 201

6

CONTENIDOS

´Indice de figuras 1.1. 1.2. 1.3. 1.4. 1.5. 1.6. 1.7. 1.8.

Sistema Macrosc´opico. . . . . . . . . . . . . Sistema aislado provisto de un pist´on. . . . . Trabajo de un pist´on. . . . . . . . . . . . . . Gr´afico de presi´on versus volumen. . . . . . Procesos. . . . . . . . . . . . . . . . . . . . . Sistema monocomponente. . . . . . . . . . . Pared m´ovil buscando el equilibrio. . . . . . Dos sistemas separados por una membrana.  2 − 13 ¯ 1.9. Gr´afico de S = S vRo θ versus UA . . . . . 1.10. Sistema aislado . . . . . . . . . . . . . . . . 1.11. Pared movi´endose . . . . . . . . . . . . . . . 1.12. Estado inicial del sistema. . . . . . . . . . .

. . . . . . . .

. . . . . . . .

. . . . . . . .

. . . . . . . .

. . . . . . . .

. . . . . . . .

. . . . . . . .

. . . . . . . .

. . . . . . . .

. . . . . . . .

. . . . . . . .

. . . . . . . .

. . . . . . . .

. . . . . . . .

. . . . . . . .

. . . . . . . .

. . . . . . . .

. . . . . . . .

. . . . . . . .

. . . . . . . .

. . . . . . . .

. . . . . . . .

15 15 17 18 19 21 22 27

. . . .

. . . .

. . . .

. . . .

. . . .

. . . .

. . . .

. . . .

. . . .

. . . .

. . . .

. . . .

. . . .

. . . .

. . . .

. . . .

. . . .

. . . .

. . . .

. . . .

. . . .

. . . .

28 34 35 37

2.1. Expansi´on adiab´atica. . . . . . . . . . . . . . . . . . . . . . . . . . . . . . . . . . . 2.2. Dispositivo experimental para medir temperatura. . . . . . . . . . . . . . . . . . . . 2.3. Temperatura del vapor (izquierda) y del azufre (derecha). . . . . . . . . . . . . . . .

40 44 45

3.1. 3.2. 3.3. 3.4.

Flujo de materia a trav´es de una membrana. . . . . . . . . . . . . . . . . . . . . . Columna de aire en un tubo aislado adiab´aticamente. . . . . . . . . . . . . . . . . . Recipiente con gas, cerrado. . . . . . . . . . . . . . . . . . . . . . . . . . . . . . . . Fuentes inagotables de calor (reservoir). Por m´as calor que entreguen, no cambian su temperatura. . . . . . . . . . . . . . . . . . . . . . . . . . . . . . . . . . . . . . . 3.5. Pist´on movi´endose debido a un calefactor. . . . . . . . . . . . . . . . . . . . . . . .

54 55 58

4.1. 4.2. 4.3. 4.4.

Superficie definida por la ecuaci´on fundamental. . . Superficie definida por la ecuaci´on fundamental.. . . Proceso aleatorio para llegar de A a H. . . . . . . . Proceso isentr´opico (dS = 0) para llegar de A a B.

. . . .

. . . .

. . . .

. . . .

. . . .

. . . .

. . . .

. . . .

. . . .

. . . .

. . . .

. . . .

. . . .

. . . .

. . . .

. . . .

. . . .

. . . .

67 68 70 71

5.1. 5.2. 5.3. 5.4. 5.5. 5.6. 5.7. 5.8.

Ciclo de Carnot. . . . . . . . . . . . . . . . . Ciclo de Carnot: Entrop´ıa vs. Temperatura. Ciclo de Carnot: Temperatura vs. Entrop´ıa. Ciclo de Carnot: Presi´on vs. volumen. . . . . Ciclo de Joule: Presi´on vs. volumen. . . . . . Ciclo de Otto: Presi´on vs. volumen. . . . . . Motor de cuatro tiempos. . . . . . . . . . . . Motor de Dos Tiempos. . . . . . . . . . . . .

. . . . . . . .

. . . . . . . .

. . . . . . . .

. . . . . . . .

. . . . . . . .

. . . . . . . .

. . . . . . . .

. . . . . . . .

. . . . . . . .

. . . . . . . .

. . . . . . . .

. . . . . . . .

. . . . . . . .

. . . . . . . .

. . . . . . . .

. . . . . . . .

. . . . . . . .

. . . . . . . .

75 80 81 82 83 84 85 87

. . . . . . . .

. . . . . . . .

. . . . . . . .

. . . . . . . .

61 64

´INDICE DE FIGURAS

8

5.9. Ciclo de Diesel: Presi´on vs. volumen. . . . . . . . 5.10. Ciclo de refrigeraci´on (ciclo de Carnot invertido) . 5.12. Proceso de trabajo m´aximo. El trabajo entregado tregado QRHS es m´ınimo si el proceso completo es 6.1. 6.2. 6.3. 6.4. 6.5. 6.6. 6.7.

. . . . . . . . . . . . . . . . . . . . . . . . . . . . . . . . WRW S es m´aximo y el calor rreversible (∆Stotal = 0). .

7.1. N´ umero de particulas con velocidades entre (los planos) wx y wx + dwx . . . . . . . . 7.2. N´ umero de particulas con velocidades entre (los planos) wx y wx + dwx y entre wy y wy + dwy . . . . . . . . . . . . . . . . . . . . . . . . . . . . . . . . . . . . . . . . . . 7.3. N´ umero de particulas con velocidades entre wx y wx + dwx , wy y wy + dwy y entre wz y wz + dwz . . . . . . . . . . . . . . . . . . . . . . . . . . . . . . . . . . . . . . . 7.4. Bulbo esf´erico. . . . . . . . . . . . . . . . . . . . . . . . . . . . . . . . . . . . . . . . 7.5. Flujo de materia . . . . . . . . . . . . . . . . . . . . . . . . . . . . . . . . . . . . .

107

. . . . . .

. . . . . .

. . . . . .

. . . . . .

. . . . . .

. . . . . .

. . . . . .

. . . . . .

. . . . . .

. . . . . .

. . . . . .

. . . . . .

. . . . . .

. . . . . .

. . . . . .

. . . . . .

. . . . . . .

. . . . . .

. . . . . . .

. . . . . .

. . . . . . .

. . . . . .

. . . . . . .

. . . . . .

. . . . . . .

. . . . . .

. . . . . . .

. . . . . .

. . . . . . .

. . . . . .

. . . . . . .

. . . . . .

. . . . . . .

. . . . . .

. . . . . . .

93 98 98 99 100 100 101 102

Proceso termodin´amico a U = cte. . Sistema equilibr´andose. . . . . . . . Pared diat´ermica y fija. . . . . . . . Curva de una funci´on y : R → R. . Curvas y(x) = x2 + cte. . . . . . . . Envolventes de una curva. . . . . .

. . . . . . .

88 90

. . . . . . .

8.1. 8.2. 8.3. 8.4. 8.5. 8.6.

Mol´ecula chocando contra una pared. . . . . . . . . . . . . . . Mol´ecula chocando contra una pared. . . . . . . . . . . . . . . Cantidad de part´ıculas con velocidad wx . . . . . . . . . . . . . Espacio de velocidades bidimensional. . . . . . . . . . . . . . . Espacio de velocidades bidimensional en coordenadas polares. Espacio de velocidades tridimensional . . . . . . . . . . . . . . Choques contra una Pared. . . . . . . . . . . . . . . . . . . . .

. . . . . . en. . .

. . . . . .

. . . . . .

109 110 118 119 128 130 131 132 132 133

9.1. Hermann von Helmholtz. 1821-1894 . . . . . . . . . . . . . . . . . . . . . . . . . . . 137 9.2. Josiah Willard Gibbs. 1839-1903 . . . . . . . . . . . . . . . . . . . . . . . . . . . . . 139 10.1. Sistema termodin´amico unido a una fuente reversible de trabajo (RWS). . . . . . . 148 10.2. Cilindro con pist´on m´ovil y resistencia el´ectrica en contacto con una fuente de presi´on.150 10.3. Proceso de “estrangulamiento” de un gas. . . . . . . . . . . . . . . . . . . . . . . . 150 10.4. Isentalp´ıas (s´olido), temperaturas de inversi´on (oscuro), y curva de coexistencia para el nitr´ogeno; semi-cuantitativa. . . . . . . . . . . . . . . . . . . . . . . . . . . . . . 151 11.1. El cuadrado termodin´amico. . . . . . . . . . . . . . . . . . . . . . . . . . . . . . . . 155 11.2. Compresi´on Adiab´atica de un Gas. . . . . . . . . . . . . . . . . . . . . . . . . . . . 159 11.3. Gas expandi´endose libremente . . . . . . . . . . . . . . . . . . . . . . . . . . . . . . 162 12.1. Para una relaci´on fundamental de curva convexa, la entrop´ıa se aumenta a trav´es de una transferencia de energ´ıa entre los dos subsistemas. Tal sistema es inestable. . . . 12.2. Relaci´on fundamental de curva c´oncava. Este sistema es estable. . . . . . . . . . . . 12.3. Envolvente de las tangentes sobre la curva de la ecuaci´on fundamental, termodin´amicamente estable. . . . . . . . . . . . . . . . . . . . . . . . . . . . . . . . . . . . . . . 12.4. U vs. S, U vs. V, U vs. T y U vs. P. La energ´ıa es convexa de los par´ametros extensivos, y c´oncava de los par´ametros intensivos. . . . . . . . . . . . . . . . . . .

167 168 169 173

Rodrigo Ferrer

´INDICE DE FIGURAS

9

13.1. Cambio de fase: l´ıquido → gas. . . . . . . . . . . . . . . . . . . . . . . . . . . . . . 177 13.2. Interfase en un proceso isot´ermico. La presi´on y temperatura se mantienen constantes en ese proceso. . . . . . . . . . . . . . . . . . . . . . . . . . . . . . . . . . . . . . . 178 13.3. Punto cr´ıtico en un cambio de fase. . . . . . . . . . . . . . . . . . . . . . . . . . . . 179 13.4. Gr´afico de la Presi´on en funci´on de la Temperatura. . . . . . . . . . . . . . . . . . . 181 13.5. De izquierda a derecha: Potencial de Gibbs Molar vs. Temperatura, volumen Molar vs. Temperatura, Entrop´ıa Molar vs. Temperatura. . . . . . . . . . . . . . . . . . . 181 13.6. Isotermas de Van der Waals. T1 < T2 < T3 . . . . . . . . . . . . . . . . . . . . . . . . 182 13.7. Gr´afico pb vs Vb . . . . . . . . . . . . . . . . . . . . . . . . . . . . . . . . . . . . . . . 184 13.8. Diagrama de fase del agua. La regi´on de estabilidad de la fase gaseosa est´a representada por una angosta e indiscernible l´ınea horizontal. . . . . . . . . . . . . . . . . . 185 13.9. Isoterma particular de la ecuaci´on de Van der Waals. . . . . . . . . . . . . . . . . . 186 13.10. Dependencia isot´ermica del potencial molar de Gibbs de la presi´on. . . . . . . . . . 187 13.11. Dependencia isot´ermica del potencial molar de Gibbs de la presi´on y la temperatura.187 13.12. La isoterma f´ısica de Van der Waals. La isoterma “fundamental” es SOMKFDA, pero la construcci´on de igual a´rea la convierte en la isoterma f´ısica SOKDA . . . . . 188 13.13. Gr´afico P vs. v para una transici´on de fase. . . . . . . . . . . . . . . . . . . . . . . 189 13.14. Discontinuidad en la entrop´ıa molar. El ´area entre las isotermas adyacentes est´a relacionada con la discontinuidad en la entrop´ıa y, por lo tanto, con el calor latente. . 190 13.15. Clasificaci´on de fases del plano P − v. . . . . . . . . . . . . . . . . . . . . . . . . . 194 13.16. Punto cr´ıtico de un gas. . . . . . . . . . . . . . . . . . . . . . . . . . . . . . . . . . 194

10

´INDICE DE FIGURAS

Parte I Principios Generales de la Termodin´ amica Cl´ asica

Cap´ıtulo 1 Postulados y Condiciones de Equilibrio 1.1.

Introducci´ on

“Una teor´ıa es tanto m´as impresionante mientras mayor sea la simplicidad de sus premisas, mientras m´as tipos de cosas relacione y mientras m´as grande sea su ´area de aplicaci´on. De aqui la profunda impresi´on que la termodin´amica cl´asica caus´o sobre m´ı. Estoy convencido de que es la u ´nica teor´ıa f´ısica de contenido universal que dentro del ´ambito de aplicabilidad de sus conceptos b´asicos, nunca ser´a echada por tierra.” EINSTEIN, 1946.

Postulado: Proposici´on cuya verdad se admite sin pruebas y que es necesaria para servir de base a ulteriores razonamientos. Una caracter´ıstica relevante de la termodin´amica es el reducido n´ umero de postulados que requiere para su formulaci´on. Gobernada por los principios m´as generales de la f´ısica, en su marco de trabajo, todos los problemas se pueden resolver. Es una teor´ıa que admite correcciones cu´anticas y relativistas: Bajas temperaturas, hoyos negros, etc. La Termodin´amica, la Teor´ıa Cin´etica de los Gases y la Mec´anica Estad´ıstica, aparecen con “disfraces” matem´aticos diferentes, aunque describen esencialmente el mismo fen´omeno. Se renuncia al m´etodo inductivo convencional a favor de un enfoque en base a postulados. Estos postulados son justificados por el ´exito posterior de la teor´ıa cin´etica en la interpretaci´on de resultados experimentales. Originalmente todos los t´opicos cl´asicos de la f´ısica se desarrollaron por inducci´on directa desde la observaci´on experimental Newton → Lagrange y Hamilton Coulomb y Ampere → Maxwell

CAP´ ITULO 1

14

POSTULADOS Y CONDICIONES DE EQUILIBRIO

VENTAJAS DE UN ENFOQUE CON POSTULADOS ◦ Concisi´on para expresar la consistencia interna y la estructura l´ogica de la teor´ıa. ◦ Una vez familiarizados con los postulados, podemos ver las cosas con mayor profundidad e intuici´on que la que nos permite el enfoque elemental. ◦ Se pueden hacer importantes extensiones y generalizaciones Formulaci´on Hamiltoniana → Mec´anica Cu´antica ◦ Razones de simplicidad en el formalismo.

1.2.

Energ´ıa Interna U

Nos enfocaremos en estudiar los llamados Sistemas Termodin´amicos Simples, caracterizados por ser: ◦ Macrosc´opicamente homog´eneos. ◦ Isotr´opicos ◦ Sin carga ◦ Qu´ımicamente inertes ◦ Suficientemente grandes como para que se desprecien los efectos de la superficie. ◦ Libres de campos el´ectricos, magn´eticos y gravitacionales. Un ejemplo de este tipo de sistema, son los sistemas hidrost´aticos, que tienen: ◦ Volumen V . ◦ Componentes qu´ımicos N1 ,. . . ,Nr , donde Ni ≡ N o moles del tipo i =

N o mol´eculas del tipo i . 6.024 × 1023

(Pueden reaccionar qu´ımicamente manteniendo la integridad de las especies at´omicas individuales).

Los sistemas macrosc´opicos tienen bien definidas y precisas energ´ıas, obedeciendo leyes de conservaci´on. Entonces, U, N, N1 , . . . , Nr ser´an nuestras variables termodin´amicas en el caso de sistemas hidrost´aticos.

1.2 Energ´ıa Interna U

Rodrigo Ferrer

Q1

Q1 , Q2 , M1 , M2

V 1 2

Q2

15

M1 V1 ,

1 2

M 2V 2 , M 1gh 1 , M 2 gh 2

M1 M2 Q Q , k 1 2 −G r r 12 12

Figura 1.1: Sistema Macrosc´opico.

Sistema Simple es aqu´el que puede ser descrito con pocas variables.

Postulado I : Existen estados particulares de los sistemas hidrost´aticos simples (llamados estados de equilibrio) que, macrosc´opicamente, est´an completamente caracterizados por la energ´ıa interna U , el volumen V y el n´ umero de moles N1 , N2 , . . . , Nr de sus distintos componentes.

Notemos que los sistemas pueden depender de otras variables distintas a las hidrost´aticas, entre ellas las correspondientes a propiedades el´ectricas, el´asticas, magn´eticas, etc. Todos los sistemas tienen una bien definida energ´ıa interna.

1.2.1.

Manipulaci´ on de los sistemas termodin´ amicos

◦ Se hace sobre las variables macrosc´opicas y los modos “ocultos” (el calor). ◦ Suceden dentro de paredes con determinadas caracter´ısticas.

Figura 1.2: Sistema aislado provisto de un pist´on.

16

CAP´ ITULO 1

POSTULADOS Y CONDICIONES DE EQUILIBRIO

PARED RESTRICTIVA Fija: No hay VOLUMEN trabajo mec´anico Impermeable: No hay MATERIA intercambio de “energ´ıa qu´ımica ”. Adiab´ atica: No hay CALOR intercambio de calor (ąQ = 0) VOLUMEN Restrictiva respecto CALOR a la energ´ıa (U = cte) VOLUMEN CALOR AISLADA MATERIA

PARED NO RESTRICTIVA M´ ovil: Hay trabajo mec´anico. Permeable: Hay cambio de “energ´ıa qu´ımica ”. Diat´ ermica: Hay intercambio de calor No restrictiva respecto a la energ´ıa ABIERTA

Tabla 1.1: Restricciones sobre una Pared.

1.2.2.

La energ´ıa U es medible y controlable

La existencia de las paredes adiab´aticas, diat´ermicas, fijas, m´oviles, permeables, impermeables, etc, hace posible controlar y medir la energ´ıa interna U. En la Figura 1.3 se presentan dos ejemplos de manipulaci´on sobre un sistema. El primero corresponde a la variaci´on de la energ´ıa interna de un gas cuando interact´ ua con un p´endulo; la disminuci´on de la energ´ıa potencial de la masa aumenta la energ´ıa interna del gas. En el segundo, el sistema no est´a aislado y la variaci´on de energ´ıa interna es producto de un trabajo mec´anico y de una cantidad de calor que se le otorga al sistema mediante una resistencia el´ectrica. Trabajo Quasi-Est´ atico: Usando el modelo de la Figura 1.3 definimos este trabajo. El pist´on se baja muy lentamente por la acci´on de la fuerza F~ , de modo que el sistema siempre est´a en equilibrio, lo que nos permite saber el valor de la presi´on (Figura 1.3) directamente o a trav´es de las ecuaciones de estado. El trabajo realizado por la fuerza (energ´ıa aplicada al sistema) en un desplazamiento dx es: dWa = F dx .

(1.1)

El aumento de la energ´ıa del sistema ser´a entonces: F dW = −dWa = −F dx = − A dx = −P dV . A

(1.2)

Para un sistema t´ermicamente aislado, un cambio infinitesimal dV en su volumen produce un cambio en su energ´ıa interna dado por dU = −P dV .

1.2 Energ´ıa Interna U

Rodrigo Ferrer

17

Figura 1.3: Trabajo de un pist´on. Si adem´as le damos una cantidad infinitesimal ąQ de calor1 , mediante una resistencia, tendremos: dU = −P dV + ąQ = ąW + ąQ , o bien, ąQ = dU + P dV = dU − ąW .

(1.3)

Esta ecuaci´on representa la conservaci´on de la energ´ıa y se conoce como la Primera ley de la Termodin´amica.

Ejemplo Considere un sistema tal que cuando se hace un cambio quasi-est´atico y adiab´atico en su volumen, su presi´on cambia seg´ un la ecuaci´on 5

P = A V −3 ,

(1.4)

con N y A constantes. Encontrar el trabajo quasi-est´atico realizado sobre el sistema y el flujo neto de calor hacia el sistema en cada uno de los procesos de la Figura 1.4. 1

La barrita en esta diferencial es para indicar que no se trata de una diferencial exacta, es decir, no es posible definir una funci´ on trabajo que dependa s´ olo de los valores de las variables termodin´amicas.

18

CAP´ ITULO 1

POSTULADOS Y CONDICIONES DE EQUILIBRIO

P [atm]

entra calor A

A’

a)

32

b)

oc pr o es

entra calor ati

iab

ad co

c) 1

sale calor

P=AV −5/3

sale calor

entra calor

B’ 1

B 8

V [litros]

Figura 1.4: Gr´afico de presi´on versus volumen. ´ n: Solucio Todos los procesos parten del estado de equilibrio termodin´amico A, y llegan al estado de equilibrio termodin´amico B, a trav´es de caminos cuyos puntos corresponden a estados de equilibrio termodin´amico. Calculemos el cambio en la energ´ıa interna cuando el sistema cambia su estado A por el B, mediante el proceso adiab´atico en el cual no hay transferencia de calor (ąQ = 0). Usando la conservaci´on de la energ´ıa, este cambio de energ´ıa interna ser´a igual al trabajo involucrado para llevar al sistema desde el estado A al estado final B (ver (1.3)). Tenemos: Z

B

W =−

P dV , A

Z

8

V −5/3 dV , 1 8 3 −2/3 = AV , 2 1 3 −2/3 = A(8 − 1) , 2 3 = A(−0.75) , 2 = −1.125A L−2/3 . = −A

Calculemos la constante A. Como P V 5/3 = A, entonces 5

A = 1 · 8 3 = 32 [atm][lt]5/3 ,

1.2 Energ´ıa Interna U

Rodrigo Ferrer

19

luego, WA→B (adiab´aticamente) = −36 L · atm . Finalmente, ∆U = −36 L · atm ,   1 = −36 cal , 0.04131 = −871.46 cal . Esta cantidad es negativa, por lo tanto el sistema pierde energ´ıa en forma de trabajo. La energ´ıa interna en el estado B es 871.46 calor´ıas menor que en el estado A. Consideremos los cambios de energ´ıa del sistema, en forma de trabajo y de calor, en los distintos procesos. A

A’

A

B

B’

0

B 0

(a) A → A → B

(b) A → B → B

Figura 1.5: Procesos. ◦ En el proceso a), Figura 1.5(a), se tiene ∆WA→A0 = −32 · 7 = −224 [L · atm] . El sistema pierde energ´ıa en forma de trabajo: el sistema realiza trabajo. Por otro lado, en el camino desde A’ a B, no hay cambio en el volumen, por lo que ∆WA0 →B = 0 . Luego, ∆QA→B = ∆U − ∆W = −36 + 224 = 188 L · atm . En este proceso, ∆QA→B es la energ´ıa que entra al sistema en forma de calor. Observemos que de A → A0 entra calor, mientras que de A0 → B sale calor, pero en ambos casos, no se sabe cu´anto. ◦ En el proceso b), el trabajo realizado es el ´area bajo la curva, de donde Z B WA→B = − P dV , A   7 · 31 =− + 7 L · atm , 2 = −157.5 L · atm .

20

CAP´ ITULO 1

POSTULADOS Y CONDICIONES DE EQUILIBRIO

Entonces el sistema pierde energ´ıa en forma de trabajo; realiza trabajo. Adem´as, el sistema gana energ´ıa en forma de calor, puesto que ∆QA→B = −36 + 157.5 = 121.5 L · atm . Es decir, entra calor al sistema. ◦ Finalmente en el proceso c), Figura 1.5(b), y por los mismos argumentos usados en el an´alisis del proceso a), ∆WA→B 0 = 0 , Z

B

P dV = −1 · 7 = −7 L · atm .

∆WB 0 →B = − B0

Siendo una magnitud negativa, el trabajo es realizado por el sistema. El cambio en la energ´ıa en forma de calor es ∆QA→B = ∆U − ∆WA→B , = −36 + 7 = −29 L · atm . Como es una magnitud negativa, esta energ´ıa sale del sistema en forma de calor. De A → B 0 sale calor; no se sabe cu´anto. De B 0 → B entra calor; no se sabe cu´anto. Si hacemos que el sistema realice un proceso c´ıclico (A → A0 → B → B 0 → A), el sistema absorber´a calor (refrigerador) ya que el flujo neto de calor es positivo. En efecto, ◦ Por el proceso a), ∆Q1 ≡ ∆QA→B , = 188 L · atm . ◦ Por el proceso c) inverso, ∆Q2 ≡ ∆QB→A , = 29 L · atm . Luego, el flujo de calor est´a dado por ∆QT otal (ciclo) = ∆Q1 + ∆Q2 = 217 L · atm .

1.3 Unidades

Rodrigo Ferrer

1.3. 1 1 1 1 1 1 1

21

Unidades

[erg] [Joule] [cal] [Btu] [L · atm] [Watt · h] [Hp · h]

1 [erg] 1 107 4.19 · 107 1.06 · 1010 1.01 · 109 3.6 · 1010 2.68 · 1013

1 [Joule] 10−7 1 4.19 1054.35 101.3 3600 2.68 · 106

1[cal] 2.39 · 10−8 0.2389 1 252 24.21 860.42 6.42 · 105

1 [Btu] 9.48 · 10−11 9.48 · 10−4 0.003968 1 0.09607 3.4144 2546.14

1 [L · atm] 9.87 · 10−10 0.009869 0.04131 10.41 1 35.524 2.5 · 104

1 [Watt · h] 2.78 · 10−11 2.78 · 10−4 0.001163 0.2930 0.02815 1 745.7

1 [Hp · h] 3.72 · 10−14 3.724 · 10−7 1.559 · 10−6 3.929 · 10−4 3.775 · 10−5 1.34 · 10−3 1

Tabla 1.2: Tabla de conversi´on de unidades energ´eticas.

1 [erg] 1 [Joule]

C.G.S. M.K.S.(SI)

cent´ımetro, gramo, segundo. metro, kilogramo, segundo.

1 [cal] = Calor necesario para elevar la temperatura de 1 gramo de agua desde 14.5o C a 15.5o C. 1 [cal] = 4.186 [Joule] (Equivalente mec´anico del calor)

1.4.

Par´ ametros Extensivos

Figura 1.6: Sistema monocomponente. X es una variable (par´ametro) extensiva si X=

n X i=1

En este caso, n = 5

Xi

22

CAP´ ITULO 1

POSTULADOS Y CONDICIONES DE EQUILIBRIO

Ejemplos: U=

n X

Ui ;

V =

n X

i=1

Vi ;

N=

i=1

n X

Ni

i=1

(ojo, el sistema es de s´olo un componente: N1 , N2 , N3 , N4 , N5 son partes del mismo componente N )

1.5.

Segundo postulado. Entrop´ıa

Consideremos un cilindro aislado provisto de un pist´on adiab´atico que separa al cilindro en dos partes 1 y 2, definiendo as´ı dos subsistemas.

V 2

V 1 S

V 2

V 1 S

j

V 1

k

V 2 S max

Figura 1.7: Pared m´ovil buscando el equilibrio. Removamos, por ejemplo, la restricci´on respecto al volumen de cada subsistema, es decir, dejemos que la pared se mueva libremente. El Postulado I dice que el sistema alcanzar´a un nuevo estado de equilibrio con determinados valores de U1 , U2 , V1 , V2 , N1 y N2 . Esto implica que la pared se detendr´a. En este caso, cambiar´an U1 , U2 , V1 y V2 , pero no cambiar´an U1 + U2 , V1 + V2 ni N1 + N2 . ¿Cu´ales ser´an los nuevos valores de U1 , U2 , V1 y V2 ? El Postulado II permite calcularlos:

Postulado II : Existe una funci´on, llamada entrop´ıa , de los par´ametros extensivos de todo sistema compuesto, definida para todos los estados de equilibrio y con la siguiente propiedad: los valores que toman los par´ametros extensivos en la ausencia de alguna restricci´on son aquellos que maximizan la entrop´ıa de entre la totalidad de los estados de equilibrio restringidos.

En el caso de la pared m´ovil, impermeable y adiab´atica, el valor de los vol´ umenes ser´an aquellos que maximizan a la entrop´ıa sobre todos los estados que pueden resultar al imponer las restricciones a la pared.

1.6.

Relaci´ on Fundamental

La funci´on entrop´ıa S = S(U, V, N1 , N2 , . . . , Nr )

1.6 Relaci´on Fundamental

Rodrigo Ferrer

23

´ n fundamental de un sistema termodina ´ mico. se llama la ecuacio De esta relaci´on sale toda la termodin´amica del sistema en equilibrio termodin´amico.

Postulado III : La Entrop´ıa de un sistema compuesto es aditiva sobre los subsistemas constituyentes. Por otro lado, la Entrop´ıa es una funci´on conti´ tona creciente de nua y diferenciable, y es adem´as una funci´on mono la energ´ıa.

Tenemos que S=

X

S (∝)

(1.5)



´ CONSECUENCIAS MATEMATICAS

1.6.1.

Aditividad Tomemos un sistema de entrop´ıa S = S(U, V, N ), con ∝ subsistemas. Como la entrop´ıa es aditiva, ´esta ser´a la suma de las entrop´ıas de todos los subsistemas, como lo indica la ecuaci´on (1.5). Es claro que la energ´ıa interna total U ser´a la suma de las energ´ıas internas Ui de todos los subsistemas, al igual que el volumen V y el n´ umero de moles de part´ıculas N . Luego, si “amplificamos” por λ los par´ametros extensivos de cada subsistema, tendremos S(λU, λV, λN ) =

∝ X

 S (i) λU (i) , λV (i) , λN (i) ,

i=1

=

∝ X

 λS (i) U (i) , V (i) , N (i) ,

i=1

= λS(U, V, N ) .

S(λU, λV, λN1 , λN2 , . . . , λNr ) = λS(U, V, N1 , N2 , . . . , Nr ) ,   X (∝) (∝) =λ S (∝) U (∝) , V (∝) , N1 , N2 , . . . , Nr(∝) , ∝

=

X

  (∝) S (∝) λU (∝) , λV (∝) , λN1 , . . . , λNr(∝) ,



es decir, S

(∝)



λU

(∝)

, λV

(∝)

,

(∝) λN1 ,

...,

λNr(∝)





= λS U

(∝)

,V

(∝)

,

(∝) N1 ,

...,

Nr(∝)



La entrop´ıa de un sistema simple es una funci´on homog´enea de primer orden de los par´ametros extensivos.

24

CAP´ ITULO 1

POSTULADOS Y CONDICIONES DE EQUILIBRIO

Continuidad y diferenciabilidad

S = S(U, V, N1 , N2 , . . . , Nr ) ⇐⇒ U = U (S, V, N1 , N2 , . . . , Nr ) % Relaciones fundamentales

(1.6)

Nota: Sea N=

r X

Ni .

i=1

 S(U, V, N1 , N2 , . . . , Nr ) = N S

Nr U V N1 N2 , , , ,..., N N N N N

 .

Definici´on: u≡

U ; N

v≡

V . N

(1.7)

Si r = 1, entonces  S(U, V, N ) = N S

 U V , , 1 ≡ N s(u, v) N N

⇒ S(U, V, N ) = N s(u, v) Mon´ otona creciente respecto a U Esto significa que 

∂S ∂U

 >0

(1.8)

V,N1 ,N2 ,...,Nr

Postulado IV : La Entrop´ıa de cualquier sistema se anula en el estado para el cual   ∂U =0 (1.9) ∂S V,N1 ,N2 ,...,Nr Plank (Nerst), 3a ley.

1.6 Relaci´on Fundamental

Rodrigo Ferrer

25

Ejemplo La ecuaci´on (relaci´on) fundamental de un sistema A es  SA =

R2 vo θ

1/3

R2 vo θ

1/3

[NA VA UA ]1/3 ,

(1.10)

[NB VB UB ]1/3 .

(1.11)

y de un sistema B es  SB =

◦ ¿Cu´al es la ecuaci´on del sistema compuesto? ◦ ¿Cumplen las ecuaciones anteriores los requisitos para describir un sistema termodin´amico? Problema 1.1-3, H. Callen.

´ n: Solucio La entrop´ıa total ser´a la suma de las entrop´ıas parciales: S = SA + SB  2 1/3   R 1/3 1/3 = [NA VA UA ] + [NB VB UB ] . vo θ

(1.12)

Notemos que las relaciones (1.10) y (1.11) son termodin´amicamente posibles, ya que cumplen con todas las condiciones de los postulados. En efecto, definiendo i = {A, B}, se tiene que ◦ Son aditivas: 1/3 R2 Si (λNi , λVi , λUi ) = [(λNi )(λVi )(λUi )]1/3 , vo θ = λSi (Ni , Vi , Ui ) . 

◦ Son continuas y diferenciables respecto a sus par´ametros extensivos: Como  Si =

R2 vo θ

1/3

[Ni Vi Ui ]1/3 ,

(1.13)

Entonces, 







Vi ,Ni

1 = 3



Ui ,Ni

1 = 3 1 = 3



∂Si ∂Ui



∂Si ∂Vi



∂Si ∂Ni

 Vi ,Si

R2 vo θ

1/3

R2 vo θ

1/3

R2 vo θ

1/3

−2/3

,

−2/3

,

−2/3

.

[Ni Vi ]1/3 Ui [Ni Ui ]1/3 Vi [Si Vi ]1/3 Ni

26

CAP´ ITULO 1

POSTULADOS Y CONDICIONES DE EQUILIBRIO

◦ Son mon´otonas crecientes de UA y UB , respectivamente, ya que    1/3 ∂Si 1 R2 −2/3 = [Ni Vi ]1/3 Ui ∂Ui Vi ,Ni 3 vo θ

> 0.

La relaci´on anterior se cumple para todo Ni , Ui y Vi . ◦ Seg´ un el Postulado IV, Si se anula cuando   ∂U = 0, ∂S V,N1 ,N2 ,...,Nr

(1.14)

Despejando Ui en la ecuaci´on (1.13), 



vo θ R2



3vo θ R2

Ui =

Si3 , Ni Vi

entonces, 

∂Ui ∂Si

 = Vi ,Ni



Si2 . (Ni Vi )1/3

Luego, usando (1.14), 

3vo θ R2



Si2 = 0. (Ni Vi )1/3

Claramente, la u ´ltima relaci´on se cumple s´olo cuando Si = 0.

Ejemplo Muestre que si un sistema monocomponente es tal que P V k = B en un proceso adiab´atico, con B y k constantes, entonces la energ´ıa es U=

1 P V + N f (P V k /N k ) , k−1

donde f es una funci´on arbitraria. Problema 1.7-8, H. Callen.

´ n: Solucio Por conservaci´on de la energ´ıa, dU = −P dV . Luego Z U = − P dV , Z = −B V −k dV , =−

B V 1−k + A, 1−k

1.6 Relaci´on Fundamental

Rodrigo Ferrer

27

con A alguna constante. Luego, usando nuevamente que B = P V k , (P V k )V 1−k +A k−1 PV = + A. k−1

U=

Como P V k = cte, existe una funci´on f tal que A = f (P V k ). Luego U (Po , Vo , No ) = 0 ⇒ A =

Po Vo . 1−k

Entonces

PV + f (P V k ). (1.15) k−1 Ahora bien, U es homog´enea de primer orden en los par´ametros extensivos V y N , es decir, U (P, λV, λN ) = λU (P, V, N ). Entonces, de (1.15), obtenemos U (P, V, N ) =

P λV + f (P λk V k ) = λU (P, V, N ). k−1 Tomando λ =

1 , resulta N

V PN 1 + f (P V k /N k ) = U (P, V, N ). k−1 N Al multiplicar es te u ´ltima ecuaci´on por N , obtenemos

U=

PV + N f (P V k /N k ). k−1

(1.16)

Ejemplo Considere dos sistemas A y B , separados por una membrana r´ıgida, impermeable y diat´ermica, y que cumplen con las ecuaciones (1.10) y (1.11).

A

B

Figura 1.8: Dos sistemas separados por una membrana. Adem´as, el sistema A est´a compuesto por NA = 3 moles, con un volumen VA = 9 cm3 , mientras que el sistema B, por NB = 2 moles y VB = 4 cm3 . La energ´ıa total del sistema es U = 20 cal. Problema 1.9-4, H. Callen.

28

CAP´ ITULO 1

POSTULADOS Y CONDICIONES DE EQUILIBRIO

´ n: Solucio Sean UA y UB las energ´ıas internas de los sistemas A y B, respectivamente. Entonces, la energ´ıa total (U = 20 cal) es UA + UB = 20 cal . (1.17) Por (1.12), tenemos S=

R2 vo θ







1/3 h

i 1/3 3UA + 2(20 − UA )1/3 .

Extremamos: 

∂S ∂UA

=

R2 vo θ

 1/3  2 −2/3 −2/3 = 0, UA − (20 − UA ) 3

2 −2/3 UA = (20 − UA )−2/3 , 3  3/2 3 UA ⇒ , = 2 20 − UA



Despejando UA de la u ´ltima ecuaci´on, se encuentra que UA = 12.9505911 cal ,

UB = 7.0494089 cal .

Estos valores extreman S. En la Figura 1.9 se aprecia claramente el m´aximo S.  S 11 10 9 8 7

5

10

12.95 15

Figura 1.9: Gr´afico de S¯ = S

20



R2 vo θ

− 13

25

versus UA .

UA

1.7 Ecuaciones de Estado

Rodrigo Ferrer

1.7.

29

Ecuaciones de Estado

Considere S = S(U, V, N1 , N2 , . . . , Nr ) ⇐⇒ U = U (S, V, N1 , N2 , . . . , Nr ) .

(1.18)

er

Calculamos la diferencial de 1 orden:      r  X ∂U ∂U ∂U dU = dS + dV + dNj ∂S V,N1 ,... ∂V S,N1 ,... ∂Nj S,V,N1 ,...,Nj−1 ,Nj+1 ,...,Nr j=1

(1.19)

A partir de (1.19), se definen las ecuaciones de estado: Temperatura,  T ≡

∂U ∂S

 ,

(1.20)

V,N1 ,...

´ n, Presio  P ≡−

∂U ∂V

 ,

(1.21)

S,N1 ,...

Potencial electroqu´ımico del componente j,   ∂U µj ≡ . ∂Nj V,N1 ,...,Nj−1 ,Nj+1 ,...,Nr

(1.22)

Con estas definiciones, podemos escribir (1.19) como dU = T dS − P dV + µ1 dN1 + · · · + µr dNr .

(1.23)

Si N1 , N2 , . . . , Nr son constantes, tenemos que dN1 = dN2 = dN3 = · · · = dNr = 0. En este caso, dU = T dS − P dV Pero seg´ un (1.3), la primera ley de la termodin´amica, dU = ąQ + ąW Luego, como ąW = −P dV , tenemos ąQ = T dS

(1.24)

La ecuaci´on (1.24) representa el flujo cuasi-est´atico de calor de un sistema, que est´a relacionado con un aumento de la entrop´ıa. ´ tico: Trabajo qu´ımico quasi-esta ąWc =

r X

µj dNj

(1.25)

j=1

En estos sistemas, la energ´ıa interna cambia quasi-est´aticamente seg´ un:

dU = T dS − P dV +

X j

µj dNj = ąQ + ąW + ąWc

(1.26)

30

1.7.1.

CAP´ ITULO 1

POSTULADOS Y CONDICIONES DE EQUILIBRIO

De las Ecuaciones de Estado a la Ecuaci´ on Fundamental

Con las definiciones anteriores, obtenemos r + 2 ecuaciones de estado, dadas por   ∂U (S, V, N1 , . . . , Nr ) ⇒ T = T (S, V, N1 , . . . , Nr ) T ≡ ∂S  V,N1 ,... ∂U (S, V, N1 , . . . , Nr ) P ≡ − ⇒ P = P (S, V, N1 , . . . , Nr ) ∂V S,N1 ,...  ∂U (S, V, N1 , . . . , Nr ) ⇒ µj = µj (S, V, N1 , . . . , Nr ) µj ≡ ∂Nj S,V,N1 ,...,Nj−1 ,Nj+1 ,...,Nr

Supongamos que conocemos P = P (S, V, N1 , . . . , Nr ). Entonces,   Z ∂U − = P (S, V, N1 , . . . , Nr ) , ⇒ U = − P dV + F (S, N1 , . . . , Nr ) , ∂S S,N1 ,...,Nr con F una funci´on desconocida. Esto quiere decir que una funci´on de estado no permite obtener la ecuaci´on fundamental. Para ello se necesitan todas las ecuaciones de estado. Por otro lado, T , P y µj son variables intensivas. En efecto, para la temperatura   ∂U (λS, λV, λN1 , . . . , λNr ) T (λS, λV, λN1 , . . . , λNr ) = , ∂λS V,N1 ,...,Nr   λ ∂U (S, V, N1 , . . . , Nr ) = , λ ∂S V,N1 ,...,Nr = T (S, V, N1 , . . . , Nr ) . De manera an´aloga, se demuestra que P (λS, λV, λN1 , . . . , λNr ) = P (S, V, N1 , . . . , Nr ) , y, µj (λS, λV, λN1 , . . . , λNr ) = µj (S, V, N1 , . . . , Nr ) . ´neas de orden cero. Luego, T , P y µj son homoge

Ahora bien, denotemos los par´ametros extensivos V , N1 , . . . , Nr , por los s´ımbolos X1 , X2 , . . . , Xt , por lo que la ecuaci´on fundamental adquiere la forma U = U (S, V, N1 , . . . , Nr ) = U (S, X1 , . . . , Xr ) . Luego, definimos a las variables intensivas por   ∂U , Pj ≡ ∂Xj

con j = {1, . . . , t}.

1.7 Ecuaciones de Estado

Rodrigo Ferrer

Notemos que V puede ser cualquier Xi . Por ejemplo, si V = X47 , entonces P47 = −P . De esta forma, tenemos  dU =

∂U ∂S



 t  X ∂U dS + dXj , ∂Xj X1 ,X2 ,...,Xt j=1 t X

= T dS +

Pj dXj .

j=1

Consideremos u = u(s, v), de un sistema monocomponente, donde s=

S , N

v=

V . N

Luego, 1 U (S, V, N ) = U (s, v, 1) . N

u(s, v) = As´ı, 

∂u ∂s

 v

! U ∂N (S, V, N ) , = ∂ NS V,N   ∂U (S, V, N ) = , ∂S V,N =T,

y 

∂u ∂v

 s

! U ∂N (S, V, N ) = , V ∂N S,N   ∂U (S, V, N ) = , ∂V S,N = −P .

Luego, 

   ∂u ∂u du = ds + dv , ∂s v ∂v s du = T ds − P dv . As´ı, para resolver un problema podemos tomar dos tipos de representaciones: ◦ ◦

Representaci´on entr´opica : S = S(U, V, N1 , . . . , Nr ) Representaci´on energ´etica : U = U (S, V, N1 , . . . , Nr )

31

32

CAP´ ITULO 1

POSTULADOS Y CONDICIONES DE EQUILIBRIO

Ambas son ecuaciones fundamentales y se ocupa la que m´as convenga. Pasar de una a otra es, a menudo, simple. Si S = S(Xo , X1 , . . . , Xt ), con {Xj } sus variables extensivas entr´opicas, entonces sus variables intensivas entr´opicas ser´an   ∂S Fk = , k = {0, 1, . . . , t} . ∂Xk De la misma forma, si U = U (S, X1 , . . . , Xt ), con {S, Xj } sus variables extensivas energ´eticas, entonces sus variables intensivas energ´eticas son   ∂U Pk = , k = {1, . . . , t} , ∂Xk y T =

∂U . ∂S

Ejemplo Encuentre las tres ecuaciones de estado para un sistema con la ecuaci´on fundamental   vo θ S 3 U= R2 N V

(1.27)

Problema 2.2-1, H. Callen.

´ n: Solucio Las ecuaciones de estado son  T =  P =−  µ=

∂U ∂S



∂U ∂V



∂U ∂N



= V,N

vo θ S 3 . R2 N V 2   3 S vo θ =− . 2 R N 2V

= S,N

S,V

3vo θ S 2 . R2 N V

(1.28) (1.29) (1.30)

Ejemplo En el problema anterior, encontrar µ = µ(T, V, N ). Problema 2.2-2, H. Callen.

1.7 Ecuaciones de Estado

Rodrigo Ferrer

33

´ n: Solucio De (1.28), tenemos que 3



S =

N V T R2 3vo θ

3/2 (1.31)

Luego, en (1.30) S3 , N 2V   −3/2 vo θ vo θ 1 N 3/2 V 3/2 T 3/2 =− , R2 R2 33/2 N 2V  2 1/2  3/2 r R T V =− . vo θ 3 N 

µ=−



vo θ R2

Ejemplo En el problema anterior, encontrar P = P (T, V, N ). Problema 2.2-3, H. Callen.

´ n: Solucio Considere la ecuaci´on (1.31). Luego, por (1.29) vo θ S 3 , R2 N V 2  3 vo θ 1 N V T R2 2 = 2 , R NV 2 3vo θ   23 s T N R2 = . 3 vo θV

P =

A veces, como revisaremos en el cap´ıtulo 9.5 (p´ag. 142), puede ser u ´til expresar los diferenciales de las ecuaciones de estado en funci´on de otras variables, como las que acabamos de ver, por ejemplo. En este caso, los diferenciales ser´ıan, si µ = µ(T, V, N ):       ∂µ ∂µ ∂µ dT + dV + dN . dµ = ∂T V,N ∂V T,N ∂N T,V

Si P = P (T, V, N ),  dP =

∂P ∂T



 dT +

V,N

∂P ∂V



 dV +

T,N

∂P ∂N

 dN . T,V

34

1.8.

CAP´ ITULO 1

POSTULADOS Y CONDICIONES DE EQUILIBRIO

Concepto de Temperatura

1 y 2 aislados, separados por una pared diat´ Sean dos sistemas ermica, fija e impermeable; es decir, s´olo se intercamba calor en el camino al equilibrio (ver Figura 1.10).

1

2

Figura 1.10: Sistema aislado Como el sistema compuesto est´a aislado, entonces la energ´ıa total interna no var´ıa, por lo que U1 + U2 = cte . Esto implica que dU1 + dU2 = 0 ,

(1.32)

dU2 = −dU1 .

(1.33)

o bien,

El sistema alcanzar´a un estado de equilibrio donde S es m´axima, es decir, dS = 0. Pero, dS = d(S1 + S2 ) , = dS1 + dS2 ,     ∂S1 ∂S2 = dU1 + dU2 . ∂U1 V,N ∂U2 V,N Usando (1.33), 

 ∂S1 ∂S2 dS = − dU1 , ∂U1 ∂U2   1 1 dS = − dU1 . T1 T2 Como dS se anula para cualquier cambio en la energ´ıa, necesariamente 1 1 = , T1 T2 o bien, T1 = T2 .

(1.34)

1.9 Equilibrio Mec´anico

Rodrigo Ferrer

1.8.1.

35

¿De d´ onde a d´ onde fluye el calor?

Sea T2 > T1 , tal que T2 = T1 + δ, con δ tan peque˜ no como se quiera. Entonces, de acuerdo a (1.34), se tiene que   1 1 ∆S ' − ∆U1 . T1 T2 Por el tercer postulado de la termodin´amica, tenemos que ∆S > 0. Entonces,   1 1 − ∆U1 > 0 . T1 T2 Como T2 > T1 , se deduce que ∆U1 > 0 . Del segundo postulado de la termodin´amica, dU = ąQ + ąW . Suponiendo que la pared es r´ıgida, tenemos dU = ąQ. Entonces, ∆Q1 > 0 . 1 (que hemos supuesto el sistema m´ El calor del sistema as fr´ıo) tender´a a aumentar para equilibrar el sistema compuesto. Adem´as, como dU2 = ąQ2 y U1 + U2 = cte, entonces el cambio en el calor 2 ser´ del sistema a negativo. Finalmente, concluimos que la energ´ıa (calor) fluye del sistema m´as caliente al m´as fr´ıo.

1.9.

Equilibrio Mec´ anico

Consideremos un sistema cerrado compuesto de dos subsistemas, separados por una pared diat´ermica, m´ovil e impermeable.

2

1

Figura 1.11: Pared movi´endose Tanto la energ´ıa y el volumen total del sistema deben mantenerse constantes. Luego, U1 + U2 = cte V1 + V2 = cte

⇒ ⇒

dU2 = −dU1 , dV2 = −dV1 .

Buscamos las condiciones de equilibrio, para las que dS = 0. Ya hemos visto que dS = dS1 + dS2 ,

(1.35) (1.36)

36

CAP´ ITULO 1

POSTULADOS Y CONDICIONES DE EQUILIBRIO

donde, S1 = S1 (U1 , V1 , N1 ) ,

S2 = S2 (U2 , V2 , N2 ) .

Diferenciando,  dS1 =  dS2 =

∂S1 ∂U1



∂S2 ∂U2



 dU1 +

V1

 dU2 +

V2

∂S1 ∂V1



∂S2 ∂V2



dV1 , U1

dV2 . U2

Observemos que 

∂Sj ∂Uj

 = Vj

1 , Tj

y, 

∂Sj ∂Vj



 = Uj

∂Sj ∂Uj

  Vj

∂Uj ∂Vj

 =− Sj

Pj , Tj

con j = {1, 2}. Luego,  dS =

   P1 P2 1 1 dU1 − dV1 + dU2 − dV2 , T1 T1 T2 T2

usando las relaciones (1.35) y (1.36),     1 1 P2 P 1 − − dS = dU1 + dV1 . T1 T2 T2 T1 En esta expresi´on, dS se anula para valores arbitrarios e independientes de dU1 y dV1 , por lo que se debe tener T1 = T2 ,

P1 = P2 .

Ejemplo 1 y 2 est´ Dos sistemas an contenidos en un cilindro cerrado, separados entre s´ı por un pist´on diat´ermico, m´ovil e impermeable. 1 est´ El sistema a descrito por las ecuaciones de estado 1 T (1)

3 N (1) = R (1) , 2 U

P (1) N (1) = R , T (1) V (1)

(1.37)

5 N (2) = R (2) , 2 U

P (2) N (2) = R , T (2) V (2)

(1.38)

2 mientras que para el sistema ,

1 T (2)

1.9 Equilibrio Mec´anico

Rodrigo Ferrer

37

con R = 1.986 molcal◦ K la constante universal de los gases ideales. (1)

Se sabe que N (1) = 0.5 mol y N (2) = 0.75 mol. Las temperaturas iniciales son Ti (2) Ti = 300o K. El volumen total es 20 L.

= 200o K y

◦ ¿Cu´al ser´a la energ´ıa y el volumen de cada sistema en el equilibrio? ◦ ¿Cu´al es la presi´on y la temperatura?

2

1

Figura 1.12: Estado inicial del sistema.

´n : Solucio (1)

En el equilibrio se debe tener Tf

(2)

= Tf

(1)

y Pf

(2)

= Pf . Luego, por (1.37) y (1.38), se tiene

3 N (1) 5 N (2) = R (2) , R 2 Uf(1) 2 Uf ⇒

2 (2) (1) Uf = Uf , 5

y N (1) (1)

Vf ⇒

(1)

Vf

=

N (2)

, (2) Vf 2 (2) = Vf . 3

Por las ecuaciones de estado, la energ´ıa interna inicial de cada subsistema es   3 (1) Ui = 200R = 297.9 cal , 4   15 (2) Ui = 300R = 1117.125 cal . 8 O sea, (1)

(2)

= 1415.025 cal ,

(2)

= 20 L .

U = Uf + Uf V

(1)

= Vf

+ Vf

38

CAP´ ITULO 1

POSTULADOS Y CONDICIONES DE EQUILIBRIO

Usando los resultados anteriores, 2 (1) (1) Uf = (U − Uf ) , 5 2 (1) (1) Vf = (V − Vf ) , 3 De este sistema, se concluye que (1)

Vf

(2)

Vf

Uf = 404.29 cal , Uf = 1010.73 cal ,

(1)

= 12 L ,

(2)

= 8 L.

Reemplazando estos valores en las ecuaciones de estado:    0.5 3 , ' 1.986 2 404.29

1 (1)

Tf es decir,

(1)

Tf

(2)

= Tf

≈ 271.427o K ,

y (1) Pf

=

(2) Pf

 ≈ (271.427)(1.986)

0.5 8

 ≈ 33.69 atm .

Cap´ıtulo 2 Gas Ideal Monoat´ omico 2.1.

Propiedades del Gas Ideal Monoat´ omico

Por definici´on, el gas ideal monoat´omico es aqu´el cuya ecuaci´on fundamental est´a dada por "  3     5 # −2 N U 2 V N So + N R ln , (2.1) S(U, V, N ) = No Uo Vo No donde No , Uo , Vo y So son constantes. N´otese que S(Uo , Vo , No ) = So . Calculemos las ecuaciones de estado: ◦ Temperatura: 1 T ⇒ U

 =

∂S ∂U

 = V,N

3 NR 2 U

3 = N RT 2

(2.2)

◦ Presi´on:  Sabemos que P = −

∂U ∂V

 . Luego, de (2.1) tenemos S,N

N S− So − N R ln No

"

V Vo



N No

− 52 #

 = N R ln

U Uo

 32

de donde, derivando a ambos lados con respecto a V y por regla de la cadena, con S y N constantes, podemos obtener  −N R

V Vo

−1 

 − 32   12   1 U 3 U 1 ∂U = NR Vo Uo 2 Uo Uo ∂V S,N  −1 NR 3 U 1 ⇒ − = NR (−P ) V 2 Uo Uo 1 3P ⇒ = V 2U

N No

 25 

N No

− 52

CAP´ ITULO 2

40

´ GAS IDEAL MONOATOMICO

o bien 2 P V = U = N RT 3   donde R es la constante universal de los gases ideales R = 8.3143

(2.3) J mol o K



◦ Potencial Electroqu´ımico: Derivando (2.1) con respecto a N , obtenemos µ 5 = R − R ln T 2

"

U Uo

 32 

V Vo



N No

− 52 #

que, al reemplazar (2.2), resulta 5U 2U µ= R− ln 3N 3N

2.2.

"

U Uo

 32 

V Vo



N No

− 52 # (2.4)

Expansi´ on adiab´ atica contra el vac´ıo

Figura 2.1: Expansi´on adiab´atica. La Figura 2.1 muestra un sistema termodin´amico, por lo tanto, tiene una ecuaci´on fundamental S = S(U, V, N ). Si removemos la restricci´on en el volumen, el sistema evolucionar´a hacia un nuevo estado de equilibrio que maximice la entrop´ıa: tratar´a de igualar las presiones.

2.3 Relaci´on de Euler

Rodrigo Ferrer

41

Si volvemos a colocar la restricci´on en el volumen, el sistema “sentir´a” que las presiones est´an igualadas, y la entrop´ıa deber´a ser mayor que la correspondiente a todos los volumenes anteriores. No hay energ´ıa cal´orica involucrada. No hay trabajo involucrado. Esto quiere decir que ∆U = ∆Q + ∆W = 0 + 0 = 0, es decir, la energ´ıa interna no cambia. En el caso del gas ideal, el cambio en la entrop´ıa depende u ´nicamente de un cambio en el volumen (ver ecuaci´on fundamental). Adem´as, en el caso del gas ideal, ∆U = 0 implica que la temperatura no cambia. Luego, para el gas ideal,     Vi Vf Vf − ln = N R ln ∆S = N R ln Vo Vo Vo Como la entrop´ıa es una funci´on de estado (al igual que la energ´ıa interna), el cambio en la entrop´ıa es independiente del proceso. Eligiendo un proceso quasi-est´atico isot´ermico, que involucre trabajo y calor, tambi´en se llega al mismo resultado: = T dS − P dV = 0 P dS = dV T Z Vf Z S dV dS = N R ⇒ V Vo So   Vf ⇒ ∆S = N R ln Vo dU

2.3.

Relaci´ on de Euler

Consideremos U (λS, λX1 , . . . , λXz ) = λU (S, X1 , . . . , Xz ) Diferenciando respecto a lambda, obtenemos la funci´on t

∂U ∂λS X ∂U ∂λXj + · =U ∂λS ∂λ ∂λXj ∂λ j=1 t

X ∂U ∂U S+ · Xj = U ∂λS ∂λX j j=1 Si tomamos λ = 1, tenemos t

X ∂U ∂U S+ Xj ∂S ∂X j j=1 ⇒ U = TS +

t X j=1

Pj X j

=U

42

CAP´ ITULO 2

´ GAS IDEAL MONOATOMICO

Esta u ´ltima es llamada la ecuaci´on de Euler.

2.4.

Relaci´ on de Gibbs-Duhem

Como vimos en (1.23), dU = T dS − P dV + µ dN cuando el sistema es monocomponente. Luego,     µ P 1 dU + dV − dN dS = T T T

(2.5)

(2.6)

Para un sistema hidrost´atico tenemos la ecuaci´on de Euler U = TS − PV +

t X

µj Nj

j=1

Consideremos t = 1. De esta forma, U = T S − P V + µN P µ 1 ⇒ S = U+ V − N T T T     1 1 ⇒ dS = U d + dU T T     µ µ P P − dN +V d + dV − N d T T T T     µ 1 P ⇒ dS = dU + dV − dN T T T     µ 1 P +U d +V d −Nd T T T y reemplazando (2.6),     µ 1 P ⇒ dS = dS + U d +V d −Nd T T T       µ 1 P ⇒ 0 =Ud +V d −Nd T T T

(2.7)

Esta ecuaci´on es conocida como la relaci´on de Gibbs-Duhem.

2.5.

De las Ecuaciones de Estado a la Ecuaci´ on Fundamental

De dos ecuaciones de estado podemos obtener la tercera, y con las tres, constru´ır la ecuaci´on fundamental, usando las relaciones de Euler y Gibbs-Duhem.

Rodrigo Ferrer

2.5 De las Ecuaciones de Estado a la Ecuaci´on Fundamental

43

Hag´amoslo para el gas ideal: 3 U = N RT 2

(Primera Ecuaci´on de Estado)

(2.8)

P V = N RT

(Segunda Ecuaci´on de Estado)

(2.9)

De la relaci´on de Gibbs-Duhem tenemos:

    µ P 1 +V d −Nd 0 = Ud T T T     µ 1 P U V ⇒ d = d + d T N T N T     µ 3 R 1 P R = d d + ⇒ d T 2 (1/T ) T (P/T ) T Integrando,

     µ µ 3 1 1 P P + R ln − ln = + R ln − ln T T o 2 T To T T o        µ µ 3 3R N 3R No NR No R ⇒ = + R ln − ln + R ln − ln T T o 2 2 U 2 Uo V Vo   µ µ 3 U/N (V /N ) ⇒ = − R ln − R ln T T o 2 (U/N )o (V /N )o "  3   3    # µ µ U 2 No 2 V No ⇒ = − R ln T T o Uo N Vo N "  3     5 # −2 µ µ U 2 V N ⇒ = − R ln T T o Uo Vo No

(2.10)

Esta u ´ltima es la tercera ecuaci´on de estado. Obtengamos ahora la ecuaci´on fundamental, usando las tres ecuaciones de estado: (2.8), (2.9) y (2.10). Por Euler, tenemos

44

CAP´ ITULO 2

´ GAS IDEAL MONOATOMICO

1 P µ U+ V − N T T T "  3     5 #! −2 µ 3 N U 2 V N ⇒ S = NR + NR − − R ln 2 T o Uo Vo No "  3     5 # −2 µ 5 N U 2 V ⇒ S = NR − N + N R ln 2 T o Uo Vo No "  3     5 # −2 N U 2 V N ⇒ S = So + N R ln , No Uo Vo No S =

donde µ 5 So = No R − No . 2 T o

2.6.

Temperatura del Gas Ideal

Considere el siguiente dispositivo experimental:

Figura 2.2: Dispositivo experimental para medir temperatura.

2.6 Temperatura del Gas Ideal

Rodrigo Ferrer

45

1. Poner un cierto gas puro (O2 , aire, N2 , H2 , etc) en el bulbo de volumen constante A y meterlo en la celda de punto triple del agua hasta que se alcance el equilibrio t´ermico. Se coloca tal cantidad de gas de modo que la presi´on sea de P3 = 1000 mmHg. 2. Ponerlo luego en equilibrio t´ermico con vapor condensado (pico de una tetera), medir la nueva P1 . Graficar este valor. presi´on P1 . Calcular θ = 273o K 1000 3. Sacarle gas al bulbo, introduci´endolo en la celda del punto triple del agua. Se le saca gas hasta que P3 = 500 mmHg. 4. Ponerlo luego en equilibrio t´ermico con vapor condensado (pico de una tetera), medir la nueva P1 presi´on P1 . Calcular θ = 273 ◦ K 500 . Graficar este valor. 5. Repetir esta operaci´on en sucesivos enrarecimientos del gas en el bulbo. 6. Se llega as´ı al l´ımite P3 → 0. Estos son los gr´aficos de θ = 273◦ K PP13 en funci´on de P3 , para vapor y azufre condensados. Notemos la interpolaci´on, en cada caso, de las rectas que se obtienen para los distintos gases. Las rectas se obtienen variando el n´ umero de moles, que, como consecuencia, trae la variaci´on del punto triple del agua.

Figura 2.3: Temperatura del vapor (izquierda) y del azufre (derecha).

Definimos la Temperatura del gas ideal por: 

o

θ = 273.16 K l´ım

P3 →0

a volumen constante.

P P3

 ,

(2.11)

46

CAP´ ITULO 2

´ GAS IDEAL MONOATOMICO

Cap´ıtulo 3 Calores Espec´ıficos 3.1.

Dos tipos de calores espec´ıficos y capacidades cal´ oricas

En nuestro estudio consideraremos dos tipos de calores espec´ıficos: ´ n constante. ◦ Calor espec´ıfico a presio       ∂s T ∂S 1 ąQ cp ≡ T = = . ∂T P N ∂T P N dT P ◦ Calor espec´ıfico a volumen constante       ∂s T ∂S 1 ąQ = = . cv ≡ T ∂T V N ∂T V N dT V

3.2.

(3.2)

Capacidades Cal´ oricas Cp ≡ N c p , Cv ≡ N c v .

3.3.

(3.1)

(3.3) (3.4)

Relaciones Entre los Calores Espec´ıficos

´ n te ´rmica ◦ Coeficiente de expansio 1 α≡ V



∂V ∂T

 .

(3.5)

P

´rmica: ◦ Coeficiente de compresibilidad isote   1 ∂V κT ≡ − , V ∂P T

(3.6)

48

CAP´ ITULO 3

CALORES ESPEC´ IFICOS

Relaciones entre los coeficientes reci´ en definidos. Mediante el uso de las relaciones entre las derivadas parciales, es posible establecer relaciones entre los coeficientes termodin´amicos. Por ejemplo, demostremos que   ∂V   ∂T P ∂P α  = = − . (3.7) ∂V ∂T V κT ∂P T

Expresemos las diferenciales totales: 

   ∂V ∂V dV = dT + dP ∂T P ∂P T     ∂P ∂P dV + dT dP = ∂V T ∂T V     ∂T ∂T dT = dP + dV ∂P V ∂V P

(3.8) (3.9) (3.10)

Reemplacemos (3.9) en (3.8):        ∂V ∂V ∂P ∂P dT + dV + dT = ∂P T ∂V T ∂P T ∂T V P              ∂V ∂V ∂P ∂V ∂P = 1− + dV − dT ∂P T ∂V T ∂T P ∂P T ∂T V 

dV ⇒

0

∂V ∂T





(3.11)

En (3.11) podemos tomar un proceso a temperatura constante, y obtenemos 

1  ⇒

∂V ∂P

 T

   ∂V ∂P = ∂P T ∂V T   −1 ∂P = ∂V T

Y si en (3.11) tomamos un proceso con V constante, tendremos 

     ∂V ∂V ∂P =− ∂T P ∂P T ∂T V   −1     ∂T ∂V ∂P ⇒ =− ∂V P ∂P T ∂T V       ∂V ∂P ∂T ⇒ −1 = ∂P T ∂T V ∂V P

(3.12)

3.3 Relaciones Entre los Calores Espec´ıficos

Rodrigo Ferrer





 ∂V   ∂T P ∂P α  = = − . ∂V ∂T V κT ∂P T

49

(3.13)

con lo que hemos demostrado (3.7). ◦ Consideremos

S = S(U, V, N ) T = T (S, V, N ) P = P (S, V, N )

(3.14) (3.15) (3.16)

Es posible dejar U en funci´on de, por ejemplo, P, T y N : De (3.15), tenemos T = T (S(U, V, N ), V, N ) = T (U, V, N ) ⇒ U = U (T, V, N ) De aqui que U = U (T, V, N ) ⇒ V = V (U, T, N )

(3.17)

P = P (S(U, V, N ), V, N ) = P (U, V, N ) ⇒ U = U (P, V, N )

(3.18)

Y de (3.16), tenemos

Finalmente, usando (3.17) en (3.18), obtenemos U = U (P, V (U, T, N ), N ) ⇒ U = U (P, T, N )

◦ Consideremos ahora U = U (T, V, N1 , . . . , Nr ) con Ni constante. Esto nos dice que  dU =

∂U ∂T



 dT +

V

∂U ∂V

Por conservaci´on de la energ´ıa, ąQ = dU + P dV, Por lo tanto,

 dV T

(3.19)

CAP´ ITULO 3

50

CALORES ESPEC´ IFICOS

   ∂U ∂U dT + dV + P dV ąQ = ∂T V ∂V T       ąQ ∂U ∂U dV ⇒ = + P+ dT ∂T V ∂V T dT 

Ahora, si en (3.20) el volumen es constante,     ąQ ∂U = Cv = . dT V ∂T V Si en (3.20) es constante la presi´on,        ąQ ∂U ∂V = C p = Cv + P + dT P ∂V T ∂T P

(3.20)

(3.21)

(3.22)

Luego, de (3.5), 

∂U ∂V

 = T

Cp − Cv −P . Vα

(3.23)

◦ Usando (2.3) en (3.22), tenemos

   ∂V ∂U Cv + P + ∂V T ∂T P        ∂ 3 ∂ N RT Cv + P + N RT ∂V 2 ∂T P T P   NR Cv + [P ] P Cv + N R . 

Cp = = = Cp =



Luego, para el gas ideal Cp = Cv + N R .

◦ Consideremos U = U (T, P, N1 , . . . , Nr ) y V = V (T, P, N1 , . . . , Nr ).

(3.24)

3.3 Relaciones Entre los Calores Espec´ıficos

Rodrigo Ferrer

51

Para N1 , . . . , Nr constantes, = dU + P dV     ∂U ∂U = dT + dP + P dV ∂T P ∂P T          ∂U ∂V ∂V ∂U dT + dP + P dT + dP = ∂T P ∂P T ∂T P ∂P T           ∂U ∂V ∂U ∂V = +P dT + +P dP ∂T P ∂T P ∂P T ∂P T   ∂U = Cp = + PV α ∂T p   ∂U ∴ = Cp − P V α . ∂T P

ąQ ⇒

ąQ



ąQ



ąQ   ąQ ∴ dT P

(3.25)

(3.26)

Luego, reemplazando (3.6), (3.7) y (3.26) en (3.25), tenemos 

ąQ ⇒ ⇒

ąQ dT V



Cv

       ∂U ∂U ∂V = + P V α dT + +P dP ∂T P ∂P T ∂P T     ∂U ∂P = Cp + − P V κT ∂P T ∂T     V ∂U α = Cp + − P V κT ∂P T κT   ∂U Cv − Cp ∴ = κT + P V κT . ∂P T α

◦ Consideremos ahora U = U (P, V )

ąQ = dU + P dV     ∂U ∂U ⇒ ąQ = dP + dV + P dV ∂P V ∂V P      ąQ ∂U dP ∂U dV ⇒ = + +P dT ∂P V dT ∂V P dT Para volumen constante, 

⇒ Cv ⇒ Cv

   ∂U ∂P = ∂P V ∂T V   ∂U α = ∂P V κT

(3.27)

52

CAP´ ITULO 3

CALORES ESPEC´ IFICOS

 ∴

∂U ∂P

 = V

Cv κT . α

(3.28)

Para presi´on constante, 

   ∂U ∂V = +P ∂V P ∂T P    ∂U = +P Vα ∂V P

⇒ Cp ⇒ Cp  ∴

∂U ∂V

 = P

Cp −P . Vα

(3.29)

Hemos calculado as´ı, todas las derivadas parciales de la energ´ıa U, para n´ umero de moles constante:             ∂U ∂U ∂U ∂U ∂U ∂U , , , , , . ∂T V ∂V T ∂T P ∂P T ∂P V ∂V P Tambi´en podemos demostrar la relaci´on Cp = Cv +

T V α2 κT

para N constante: consideremos la diferencial  T dS = T

∂S ∂T





dT + T   ∂S = Cp dT + T . ∂P T P

∂S ∂P

 dP T

Consideremos la relaci´on de Gibbs-Duhem

N dµ = −S dT + V dP ⇒ µ = µ(T, P ), es decir, 



   ∂µ ∂µ dµ = dT + dP ∂T P ∂P T     ∂µ ∂µ S = −N ,V =N . ∂T P ∂P T

3.3 Relaciones Entre los Calores Espec´ıficos

Rodrigo Ferrer

53

Como las diferenciales son exactas, sus segundas derivadas son iguales: 

      ∂µ ∂ ∂µ = ∂T P T ∂T ∂P T P     ∂V ∂S = (Relaci´on de Maxwell) − ∂P T ∂T P   ∂V T dS = Cp dT − T dP. ∂T P

∂ ∂P

⇒ ∴



(3.30) (3.31)

Ahora, S = S(T, V, N ). Con N constante,  T dS = T

∂S ∂T





∂S ∂V



dT + T   ∂S = Cv dT + T dV. ∂V T V

dV T

Ahora, consideremos la relaci´on de Euler: U − TS ⇒ d(U − T S) ⇒ d(U − T S)   ∂S ⇒ ∂V T ⇒

T dS

= Nµ − P V = N dµ − P dV − V dP = −S dT − P dV + V dP − V dP   ∂P = (Relaci´on de Maxwell) ∂T V   ∂P = Cv dT + T dV ∂T V

Igualando (3.31) con (3.33), tenemos 

   ∂V ∂P Cp dT − T dP = Cv dT + T ∂T P ∂T V     ∂V dP ∂P dV ⇒ Cp − T = Cv + T , ∂T P dT ∂T V dT que para P constante,  ⇒ Cp = Cv + T

∂P ∂T

 

∂V ∂T



∂V ∂T

 

∂P ∂T 



V

P

y para V constante,  ⇒ Cp = Cv + T

P α = Cv + T (V α) κT

V

(3.32) (3.33)

54

CAP´ ITULO 3

CALORES ESPEC´ IFICOS

Cp = Cv +



T V α2 . κT

(3.34)

Al dividir por N , tenemos la relaci´on entre los calores espec´ıficos cp = cv +

3.4. 3.4.1.

T V α2 . N κT

(3.35)

Problemas Equilibrio frente al flujo de materia

2

1

Figura 3.1: Flujo de materia a trav´es de una membrana. Tenemos S = S(U, V, N ) = S (1) (U1 , V1 , N1 ) + S (2) (U2 , V2 , N2 ) . Este problema V1 y V2 se mantienen constantes. Luego,  (1)   (1)  ∂S ∂S (1) dS = dU + dN (1) (1) (1) ∂U ∂N (1) (1) N U  (2)   (2)  ∂S ∂S dU (2) + dN (2) , + (2) (2) ∂U ∂N (2) (2) N U     1 ∂U (1) ∂S (1) = (1) dU (1) − dN (1) , T ∂N (1) S (1) ∂U (1) N (1)    (2)  1 ∂U (2) ∂S (2) + (2) dU − dN (2) , (2) (2) T ∂N ∂U (2) (2) S N =

1 T (1)

dU (1) −

µ(1) 1 µ(2) (1) (2) dN + dU − dN (2) , T (1) T (2) T (2)

donde hemos usado 

∂S ∂N

  U

∂N ∂U

  S

∂U ∂S

 = −1 . N

3.4 Problemas

Rodrigo Ferrer

55

Como U (1) + U (2) = cte y N (1) + N (2) = cte, tenemos dU (1) = −dU (2) , y dN (1) = −dN (2) . En el equilibrio, se tiene que dS = 0. Luego,  dS = 0 =

1 T (1)



1 T (2)

 dU

(1)

 −

µ(1) µ(2) − T (1) T (2)



dN (1) ,

lo cual implica que 1 T (1)

=

1 T (2)

,

µ(1) µ(2) = , T (1) T (2)

o bien, T (1) = T (2) ,

µ(1) = µ(2) .

Para determinar la direcci´on del flujo de materia, supongamos que µ(1) > µ(2) , y definamos T (1) = T (2) = T . Con esto dS =

µ(2) − µ(1) dN (1) > 0, T

1 al 2 si µ(1) > µ(2) . lo que implica que dN (1) < 0. Luego, el componente N fluye del sistema

3.4.2.

Variaci´ on de la temperatura en la atm´ osfera

El siguiente problema es un modelo para encontrar la variaci´on de la temperatura de la atm´osfera con respecto a la altura. Supondremos que la atm´osfera es un gas ideal y tomaremos de referencia el nivel del mar.

Figura 3.2: Columna de aire en un tubo aislado adiab´aticamente.

56

CAP´ ITULO 3

CALORES ESPEC´ IFICOS

El volumen de la columna es A · dh. Por Arqu´ımedes tenemos que la presi´on a una altura h est´a dada por: P = P o − ρgh ⇒ dP = −ρgdh m = − g dh V mP = −g dh N RT P ∴ dP = −gM dh, RT

(3.36)

donde M es la masa molecular. Tomemos U = U (T, P, N ), V = V (T, P, N ). ąQ = dU + P dV     ∂U ∂U ⇒ ąQ = dT + dP ∂T P ∂P T     ∂V ∂V +P dT + P dP ∂T P ∂P T      ∂U ∂V ⇒ ąQ = +P dT ∂T P ∂T P      ∂V ∂U +P dP + ∂P T ∂P T

(3.37) (3.38) (3.39)

Recordando de (3.27) que 

∂U ∂P

 = T

Cv − Cp κT + P V κT α

(3.40)

y de (3.26) que 

∂U ∂T

 = Cp − P V α

(3.41)

P

podemos reemplazarlas en (3.38), obteniendo que ąQ = (Cp − P V α + P V α) dT   Cv − Cp + κT + P V κT − P V κT dP α Cv − Cp ⇒ ąQ = Cp dT + κT dP α

(3.42) (3.43)

3.4 Problemas

Rodrigo Ferrer

57

Para el gas ideal, seg´ un (3.7), tenemos α κT



 ∂P = ∂T V   ∂ N RT = ∂T V V NR = V

Al reemplazar esta u ´ltima ecuaci´on junto con (3.24) en (3.43), obtenemos V dP NR ⇒ ąQ = Cp dT − V dP ąQ = Cp dT − N R

(3.44)

Trat´andose de un problema adiab´atico, ąQ = 0. Luego

Cp dT

= V dP = N RT

dP P

dP dT = NR Cp T P Z T Z dT N R P dP ⇒ = Cp Po P To T     T P Cp − Cv ⇒ ln ln = To Cp Po  1− γ1 T P Cp = ⇒ , con γ = To Po Cv ⇒



T

= cte P

(3.45)

γ−1 γ

De (3.45) deducimos inmediatamente que, al reemplazar (3.36), dT T dT ⇒ dh

γ − 1 dP γ P γ − 1 gM =− γ R =

Experimentalmente, se tiene que, para el aire, γ = 57 . Luego, con g = 980   107 erg oC , o   o  dT C C −5 = −9.8x10 = −9.8 , dh cm Km

 cm  s2

y R = 8.214 ×

58

CAP´ ITULO 3

CALORES ESPEC´ IFICOS

resultado que viene a ser un poco mayor que el promedio medido experimentalmente. Conclusi´on: La temperatura en la atm´osfera disminuye en 9.8o C por cada kil´ometro que se asciende. Esto nos dice, al integrar la ecuaci´on e imponer T (h = 0) = 30o C, que a 3[Km] de altitud hay Oo C de temperatura.

3.4.3.

Experimento de R¨ uchardt (1929)

El objetivo de este experimento es medir γ.

Figura 3.3: Recipiente con gas, cerrado. El experimento consiste en poner una bolita de masa m en el tubo situado en la boquilla del recipiente, de modo que cubra el tubo de ´area A, pero que pueda deslizarse libremente. Una vez situada, la bolita deber´a quedar en un equilibrio mec´anico. Si la empujamos hacia adentro, la bolita comenzar´a a oscilar debido a las variaciones de presi´on que esta misma provocar´a. Supongamos que un desplazamiento dz provoca una variaci´on dP = Supongamos que el gas experimenta un proceso adiab´atico.

dF A

en la presi´on.

Tomemos Q = Q(T, P ) y Q = Q(T, V ). Luego, para Q = Q(T, P ), 

 ąQ ąQ = dT + dP dP T P   ∂S ⇒ ąQ = Cp dT + T dP ∂P T ąQ dT





(3.46)

3.4 Problemas

Rodrigo Ferrer

59

y para Q = Q(T, V ),  ąQ dT + dV ąQ = dV T V   ∂S ⇒ ąQ = Cv dT + T dV ∂V T 

ąQ dT





(3.47)

Usando las ecuaciones de Maxwell (3.30) y (3.32) para este gas ideal, 

∂S ∂P

 ⇒ T





 ∂V =− ∂T P   ∂ N RT =− ∂T P P NR =− P

T

∂S ∂P



∂S ∂V



 ⇒ T



= −V T

∂S ∂V



 ∂P = ∂T V   ∂ N RT = ∂T V V NR = V

T

 = P. T

Podemos escribir (3.46) y (3.47) como

ąQ = Cp dT − V dP

(3.48)

ąQ = Cv dT + P dV

(3.49)

y

Como el proceso es adiab´atico, ąQ = 0. Luego

Cv dT = −P dV Cp dT = V dP Dividiendo ambas ecuaciones, tenemos

60

CAP´ ITULO 3

CALORES ESPEC´ IFICOS

⇒ ⇒ ⇒ ⇒ ⇒

Cp Cv dV γ V Z V dV γ Vo V  γ V ln Vo  γ V Vo PV γ

= γ=− = −

V dP P dV

dP P Z P

= − Po



P = ln Po   Po = P = cte

dP P −1

(3.50)

Diferenciando (3.50), tenemos

= P γV γ−1 dV + V γ dP dF 0 = P γV γ−1 A dz + V γ A γP A2 dz dF = − V γP A2 F (z) = − ∆z V γP A2 F (z) = (zo − z) V 0

⇒ ⇒ ⇒ ⇒

Esta es la fuerza que ejerce el gas sobre la esfera, donde en zo (la altura desde la cual se suelta) es cero, ya que inmediatamente despu´es de soltarla, el gas no ejerce fuerza (F (0) = 0). Por la tercera ley de Newton, la fuerza neta debe ser igual a la masa del objeto por su aceleraci´on. Luego, la ecuaci´on de fuerzas es = F (z) − mg γP A2 ⇒ m¨ z = (zo − z) − mg V γP A2 γP A2 ⇒ z¨ + z = −g mV mV m¨ z

Esta es la ecuaci´on no homog´enea del oscilador arm´onico, donde reconocemos

3.4 Problemas

Rodrigo Ferrer

ω2 = ⇒

ω



ν



τ

61

γP A2 mV r

γP A2 = 2πν = mV r 2 1 γP A = 2πs mV = 2π

mV , γP A2

donde τ es el per´ıodo de la oscilaci´on. Despejado γ de esta ecuaci´on, obtenemos

γ=

4π 2 mV . A2 P τ

Midiendo τ podemos obtener γ.

3.4.4.

Cambios de temperatura

Queremos llevar la temperatura de un cuerpo de capacidad cal´orica Cv , desde Ti a Tf . Para ello, lo vamos poniendo en contacto t´ermico con una serie de fuentes inagotables de calor, que est´an sucesivamente a temperaturas Ti + ∆T, Ti + 2∆T, Ti + ∆T, . . . , Tf = Ti + N ∆T , donde ∆T =

Ti + ∆ T

Tf − Ti . N

Ti + 2 ∆ T

Ti + 3 ∆ T

Ti + N ∆ T

Figura 3.4: Fuentes inagotables de calor (reservoir). Por m´as calor que entreguen, no cambian su temperatura.

Analicemos los cambios de entrop´ıa involucrados en el proceso de llevar el cuerpo de Ti + ∆T : ◦ En el cuerpo: Cv =

T dS , dT

CAP´ ITULO 3

62

CALORES ESPEC´ IFICOS

o bien, Z

Ti +∆T

Cv

∆Scuerpo = Ti

= Cv ln

dT , T

Ti + ∆T . Ti

◦ En la fuente inagotable de calor ∆Sfuente = −

∆Q Cv ∆T =− Ti + ∆T Ti + ∆T

Esto significa que hay un cambio total en la entrop´ıa: ∆S = ∆Scuerpo + ∆Sfuente Ti + ∆T ∆T = Cv ln − Cv Ti Ti + ∆T En el n-´esimo paso,

∆Sn = ∆Scuerpo (n) + ∆Sfuente (n) Ti + n∆T ∆T = Cv ln − Cv Ti + (n − 1)∆T Ti + n∆T

Al cabo de N pasos se llega a la temperatura Tf , y el cambio en la entrop´ıa ser´a

∆S =

N X

N

X ∆T Ti + n∆T Cv ln Cv − , Ti + (n − 1)∆T Ti + n∆T n=1 n=1 "

= Cv ln(Ti + ∆T ) − ln(Ti ) + ln(Ti + 2∆T ) − ln(Ti + ∆T ) + · · · + ln(Ti + N ∆T ) − ln(Ti + (N − 1)∆T ) # N X ∆T . − T + n∆T n=1 i Luego, " ∆S = Cv

N

Tf X ∆T ln − Ti T + n∆T n=1 i

# .

(3.51)

3.4 Problemas

Rodrigo Ferrer

63

◦ Tomemos el caso N = 1:



Tf Tf − Ti ∆S = Cv ln − Ti Ti + Tf − Ti   Ti Tf −1+ = Cv ln Ti Tf   Ti Ti = Cv − 1 − ln Tf Tf Definiendo x =

Ti , Tf



tenemos que, para Ti < Tf , x < 1. Luego,

f (x) ≡ Cv (x − 1 − ln x)   1 ⇒ f 0 (x) = Cv 1 − ≤0 x

∀ x ≤ 1.

Entonces, f es decreciente en ]0, 1]. Luego x ≤ 1



f (x) ≥ f (1) = 0 ∀ x ≤ 1.

Por lo tanto  ∆S = Cv

Ti Ti − 1 − ln Tf Tf

 ≥0

para Ti ≤ Tf . An´alogamente, cuando Ti > Tf , x > 1. Luego f 0 (x) > 0 ∀ x > 1. Con ´esto, f (x) > 0 ∀ x > 1. Por lo tanto   Ti Ti ∆S = Cv − 1 − ln ≥0 siempre. Tf Tf ◦ Tomemos el caso N → ∞: En este caso, la ecuacion (4.2) se vuelve Z Tf Tf dT ∆S = Cv ln − Cv Ti T Ti Tf Tf = Cv ln − Cv ln = 0 Ti Ti

Luego, hay reversibilidad en el proceso, como veremos en el siguiente cap´ıtulo.

64

CAP´ ITULO 3

CALORES ESPEC´ IFICOS

Figura 3.5: Pist´on movi´endose debido a un calefactor.

3.4.5.

Pistones M´ oviles

Un recipiente cerrado contiene 108 L de un gas monoat´omico inerte. En su interior hay un pist´on adiab´atico que no ejerce roce sobre las paredes y separa el recipiente en dos partes iguales, como muestra la Figura 3.5. (1) (2) La presi´on inicial en cada lado (subsistema) del recipiente es de Pi = Pi = 1 atm, y la tempe(2) (1) 1 un calefactor que, lentamente, entrega ratura, Ti = Ti = 273 ◦ K. Se conecta al subsistema (2) energ´ıa cal´orica, hasta que el otro subsistema alcanza una presi´on de Pf = 7.59 atm. Calcule el 2 y las temperaturas finales que alcanza cada subsistema. trabajo que se realiza sobre el subsistema ´ n: Solucio Como vimos en el Cap´ıtulo 2, para los gases monoat´omicos, como el He, Ne, y vapores met´alicos como el Na, Cd y Hg, se cumple U = 23 N RT = 23 P V . Luego,

 Cv =

∂U ∂T

 V

3 = NR , 2

y, Cp = Cv + N R , 5 = NR . 2 Luego,

γ=

5 Cp = . Cv 3

3.4 Problemas

Rodrigo Ferrer

65

Como el proceso es adiab´atico, P V γ = cte 5 5 5 ⇒ P V 3 = Po Vo 3 = 54 3 ! 35 5 3 54 ⇒ V = P !− 25 5 3 3 54 ⇒ dV = 5 P ⇒

=−

dV

5

54 3 − 2 P

! dP

3 54 dP 5 P 58

Luego, el trabajo realizado sobre el subsistema 2 est´a dado por Z ∆W = −

Pf

P dV   3 54 P − dP 5 P 85

Pi Z 7.59

= − 1

Z 7.59 3 3 = 54 P − 5 dP 5 1   7.59 3 5 2 = 54 P5 5 2 1 = 101.21 L · atm . La cantidad de moles N que hay en el primer compartimento (que en este caso resulta ser la misma cantidad que en el segundo), est´a dada por las condiciones iniciales: (1)

(1)

Pi Vi

(1)

=

N RTi

=

Pi Vi

(1)



N

,

(1)

(1)

,

RTi ⇒

N

=

(1)(54) L · atm  , 8.3143 molJ ◦ K (273◦ K)



N

=

1 · 54 · 101.3 J  , 8.3143 molJ◦ K (273◦ K)



N

=

2.41 mol .

2 que el proceso fuera adiab´ Ahora, en el subsistema , atico nos dice que

66

CAP´ ITULO 3

CALORES ESPEC´ IFICOS

dU

=

ąQ + dW ,



∆U

=

∆W ,



3 N R∆T 2

=

∆W ,

=

Ti

(2)



Tf

(2)

+

2∆W . 3N R

Num´ericamente, (2)

Tf

2(101.21)(101.3) J , 3(2.41 mol) 8.3143 molJ◦ K

=

273◦ K +

=

614.11 ◦ K .

Adem´as, (2)

(2)

Pf Vf

=

(2)

N RTf , (2)



(2) Vf

=

N RTf (2)

Pf = =

2.41 · (8.3143 · 101.3−1 ) · 614.11 L, 7.59 16 L .

Luego, (1)

(1)

Tf

=

(1)

Pf Vf NR

,

(1)

(2)

Pf (Vtotal − Vf ) = , NR 7.59 (108 − 16) ◦ K, 2.41(8.3143)(101.3−1 ) = 3530.17 ◦ K .

=

1 fue de Finalmente, el calor entregado al sistema

3 ∆Q = ∆U − ∆W = N R∆T + ∆W , 2 es decir, 3 ∆Q = (2.41)(8.3)(3530.17 − 273) + (101.21)(101.3) J , 2 = 108.15 kJ .

Cap´ıtulo 4 Procesos Reversibles e Irreversibles 4.1.

Procesos Quasi-Est´ aticos

Sean A y B estados de equilibrio termodin´amico. Considere N = cte. Para estos estados existe una ecuaci´on fundamental S = S(U, V, N ).

Figura 4.1: Superficie definida por la ecuaci´on fundamental.

Notemos que 

∂S ∂U

 ≥ 0. V

El punto indicado en la figura es un estado de equilibrio termodin´amico.

68

CAP´ ITULO 4

PROCESOS REVERSIBLES E IRREVERSIBLES

Figura 4.2: Superficie definida por la ecuaci´on fundamental..

Todos los estados de la curva indicada en la figura, son estados de equilibrio termodin´amico. Son una sucesi´on de estados de equilibrio, tan cercanos uno del otro como sea posible, y representan un proceso quasi-est´atico. Cabe destacar que un sistema no puede disminuir su entrop´ıa, s´olo podemos disminuir la entrop´ıa de una parte del sistema si esto involucra aumentar la entrop´ıa del otro. En la sucesi´on de estados quasi-est´aticos existen diferenciales (dV , dS, dT , etc.). En particular, dS = ąQ/T . Veamos lo que sucede para el gas ideal: ◦ Seg´ un (3.48), T dS = Cp dT − V dP , pero en un proceso adiab´atico ąQ = 0. Luego, Cp dT = V dP , usando que P V = N RT , Cp

dP dT = NR . T P

Si Cp = cte,  Cp ln

Tf Ti



 = N R ln

Pf Pi

 ,

4.2 Definici´on de Reversibilidad

Rodrigo Ferrer

69

pero para un gas ideal, N R = Cp − Cv . Luego,  ln

Tf Ti



Cp − Cv = ln Cp



Pf Pi

 .

Definiendo γ = Cp /Cv , obtenemos 

     Tf 1 Pf ln = 1− ln , Ti γ Pi    1− γ1 P T = . Ti Pi Finalmente, TP

1−γ γ

= cte ,

P V γ = cte , T V γ−1 = cte . Las u ´ltimas tres ecuaciones representan los procesos adiab´aticos en sus 3 representaciones. ◦ En un proceso isot´ermico, P V = N RT = cte.

4.2.

(4.1)

Definici´ on de Reversibilidad

Consideremos un sistema cerrado, compuesto, y con restricciones internas liberadas. El sistema evolucionar´a del estado A al estado H (Figura 4.3). El postulado II no permite que, mediante manipulaciones de los par´ametros internos del sistema, podamos llevarlo de vuelta a su estado inicial A. El proceso es irreversible. Para hacerlo, debemos abrir el sistema, aumentando la entrop´ıa del sistema acoplado. En principio, mediante procesos cuasi-est´aticos, podemos cambiar un sistema de un estado a otro sin que cambie apreciablemente su entrop´ıa; en el l´ımite de un cambio nulo estar´ıamos frente a un proceso reversible. Podemos definir los procesos reversibles como aquellos que no involucran un aumento en la entrop´ıa (Figura 4.4). La identificaci´on de −P dV como el trabajo y T ds como el calor es v´alida s´olo para procesos cuasi-est´aticos.

4.2.1.

Cambios reversibles e irreversibles de temperatura.

Queremos llevar la temperatura de un cuerpo de capacidad cal´orica Cv , desde Ti a Tf . Para ello, lo vamos poniendo en contacto t´ermico con una serie de fuentes inagotables de calor, que est´an sucesivamente a temperaturas Ti + ∆T, Ti + 2∆T, Ti + ∆T, . . . , Tf = Ti + N ∆T , donde

70

CAP´ ITULO 4

PROCESOS REVERSIBLES E IRREVERSIBLES

Figura 4.3: Proceso aleatorio para llegar de A a H.

Tf − Ti . N Estas fuentes de calor, por m´as calor que entreguen, no cambian su temperatura. Analicemos los cambios de entrop´ıa involucrados en el proceso de llevar el cuerpo de Ti + ∆T : ∆T =

◦ En el cuerpo:

Cv

=

T dS dT Z Ti +∆T

⇒ ∆S =

Cv Ti

dT Ti + ∆T = Cv ln T Ti

◦ En la fuente inagotable de calor

∆S = −

∆Q Cv ∆T =− Ti + ∆T Ti + ∆T

Esto significa que hay un cambio total en la entrop´ıa: ∆S = ∆Scuerpo + ∆Sfuente Ti + ∆T ∆T = Cv ln − Cv Ti Ti + ∆T En el n-´esimo paso,

4.2 Definici´on de Reversibilidad

Rodrigo Ferrer

71

Figura 4.4: Proceso isentr´opico (dS = 0) para llegar de A a B.

∆Sn = ∆Scuerpo (n) + ∆Sfuente (n) Ti + n∆T ∆T = Cv ln − Cv Ti + (n − 1)∆T Ti + n∆T

Al cabo de N pasos se llega a la temperatura Tf , y el cambio en la entrop´ıa ser´a N

N X

X ∆T Ti + n∆T Cv ∆S = Cv ln − Ti + (n − 1)∆T Ti + n∆T n=1 n=1 " = Cv ln(Ti + ∆T ) − ln(Ti ) + ln(Ti + 2∆T ) − ln(Ti + ∆T ) + · · · + ln(Ti + N ∆T ) − ln(Ti + (N − 1)∆T ) # N X ∆T − T + n∆T n=1 i " ∴

∆S = Cv

N

Tf X ∆T ln − Ti T + n∆T n=1 i

# (4.2)

72

CAP´ ITULO 4

PROCESOS REVERSIBLES E IRREVERSIBLES

◦ Tomemos el caso N = 1:



Tf − Ti Tf − ∆S = Cv ln Ti Ti + Tf − Ti   Tf Ti = Cv ln −1+ Ti Tf   Ti Ti = Cv − 1 − ln Tf Tf Definiendo x =

Ti , Tf



tenemos que, para Ti < Tf , x < 1. Luego,

f (x) ≡ Cv (x − 1 − ln x)   1 0 ≤0 ∀ x ≤ 1. ⇒ f (x) = Cv 1 − x ⇒ f es decreciente en ]0, 1] Luego x ≤ 1



f (x) ≥ f (1) = 0 ∀ x ≤ 1.

Por lo tanto  ∆S = Cv

Ti Ti − 1 − ln Tf Tf

 ≥0

para Ti ≤ Tf . An´alogamente, cuando Ti > Tf , x > 1. Luego f 0 (x) > 0 ∀ x > 1. Con ´esto, f (x) > 0 ∀ x > 1. Por lo tanto

 ∆S = Cv

Ti Ti − 1 − ln Tf Tf

 ≥0

siempre.

◦ Tomemos el caso N → ∞: En este caso, la ecuacion (4.2) se vuelve

Z Tf Tf dT ∆S = Cv ln − Cv Ti T Ti Tf Tf = Cv ln − Cv ln = 0 Ti Ti Luego, hay reversibilidad en el proceso.

4.2 Definici´on de Reversibilidad

Rodrigo Ferrer

73

Ejemplo Consideremos un sistema que tiene volumen constante, n´ umero de moles constante y capacidad cal´orica, C, constante. La ecuaci´on fundamental del sistema est´a dada por   U . S = So + C ln Uo de modo que U = CT . 1 y , 2 est´ Dos de estos sistemas, an inicialmente a temperaturas T10 y T20 , respectivamente, con T10 < T20 . Con estos dos sistemas (fuentes de calor) queremos mover una m´aquina mediante extracci´on de energ´ıa, con lo cual cambiar´an sus temperaturas. Queremos saber el trabajo m´aximo que se puede extraer hasta que estos cuerpos igualen sus temperaturas, lo cual vamos a suponer a priori. El cambio en la energ´ıa interna ser´a: ∆U = 2CT − C(T10 + T20 ) , y el trabajo liberado a la m´aquina ser´a W = −2CT + C(T10 + T20 ) = C(T10 + T20 − 2Tf ) . El cambio en la entrop´ıa total ser´a ∆S = C ln

Tf Tf Tf . + C ln = 2C ln √ T10 T20 T10 T20

Para maximizar el trabajo tenemos que minimizar Tf , es decir √ minimizar ∆S. Este m´ınimo es cero, que corresponde a un proceso reversible, y entonces Tf = T10 T20 . Luego:   p W = C T10 + T20 − 2 T10 T20 .

74

CAP´ ITULO 4

PROCESOS REVERSIBLES E IRREVERSIBLES

Cap´ıtulo 5 Ciclos Termodin´ amicos 5.1.

Ciclo de Carnot

Consideremos un gas ideal sometido al siguiente proceso c´ıclico:

T

isoterma

b

c

Tf

adiabata

adiabata

d

a To

isoterma

V Vb

Va

Vc

Vd

Figura 5.1: Ciclo de Carnot.

◦ Proceso a → b : Adiabata reversible El gas se comprime d´andole energ´ıa en forma de trabajo mec´anico. La energ´ıa interna aumenta. Esto implica que la temperatura aumenta, en efecto:

76

CAP´ ITULO 5

´ CICLOS TERMODINAMICOS

T dS 0

⇒ ⇒

= =

Cv dT Z Tf

Cv dT N R To T 1 Tb ⇒ ln γ − 1 Ta Tb ⇒ Ta ⇒ Tb > Ta



Cv dT + P dV Cv dT + P dV N RT = − dV V Vb = − ln > 0 Va Va = ln Vb Va γ−1 =( ) > 1 Vb

◦ Proceso b → c : Isoterma reversible El gas se expande a temperatura constante. Muy lentamente se le da calor al sistema y se le quita energ´ıa en forma de trabajo:

ąQ

=

⇒ Qb→c = =

Cv dT + P dV Z Vc dV N RTf V Vb Vc N RTf ln > 0. Vb

(5.1)

Esto significa que el gas absorbe calor (Qf > Qi ). ◦ Proceso c → d : Adiabata reversible El gas se expande realizando trabajo solamente. Su energ´ıa interna disminuye, por lo que la temperatura disminuye, en efecto:

⇒ ⇒

T dS 0 Cv dT Z Td

Cv dT N R Tc T 1 Td ⇒ ln γ − 1 Tc Td ⇒ Tc ⇒ Td < Tc



= =

Cv dT + P dV Cv dT + P dV N RT = − dV V Vd = − ln < 0 Vc Vc = ln Vd Vc = ( )γ−1 < 1 Vd

5.1 Ciclo de Carnot

Rodrigo Ferrer

77

◦ Proceso d → a : Isoterma reversible El gas se comprime a temperatura constante To . Se le extrae calor al sistema y se le da energ´ıa en forma de trabajo:

=

ąQ

P dV Z

⇒ Qd→a =

Va

N RTo Vd

=

N RTo ln

dV V

Va < 0. Vd

(5.2)

Hemos completado un ciclo en el cual hubo un intercambio de energ´ıas en forma de calor y en forma de trabajo mec´anico. Tenemos: N RT ln VVcb Qb→a . = Qd→a N RT ln VVad Adem´as 

To Va γ−1 = To Vd γ−1 = 

γ−1

Tf Vb γ−1 Tf Vc γ−1 γ−1 Vb Vc Vd ln Vc





Va Vd



ln

Va Vb



Qb→c Tf =− . Qd→a To

= =

De esta u ´ltima ecuaci´on tenemos Tf = −To

Qb→c >0 Qd→a

y Qb→c Qd→a + =0 Tf To

Nota:



Tf · Sb→c To · Sd→a + = ∆S = 0. Tf To

78

CAP´ ITULO 5

´ CICLOS TERMODINAMICOS

|Qb→c | = |Qd→a |

Tf >1 To

⇒ |Qb→c | >

|Qd→a |.

La energ´ıa es una funci´on de estado, por lo tanto, vuelve a ser la misma que cuando se inici´o el ciclo en el estado a. Esto implica que el sistema debe botar energ´ıa; y lo hace realizando trabajo neto. Notamos adem´as que entra m´as calor del que sale. ◦ Wa→b :

ąQ

=

0

=

Cv dT + P dV

Z ⇒ Wa→b = =



P dV

Cv (Tf − To ) > 0.

El gas recibe energ´ıa en forma de trabajo. S = So ,

So constante.

◦ Wb→c :

ąQ = ⇒ Wb→c = =

P dV −Qb→c < 0 Z Sf T dS −

=

−Tf (Sf − So ),

So

Tf constante.

El gas realiza trabajo. ◦ Wc→d :

ąQ = ⇒ dWc→d =

0 −PZ dV





Wc→d

El gas realiza trabajo. S = Sf ,

=

P dV

Sf constante.

=

Cv (To − Tf ) < 0.

5.1 Ciclo de Carnot

Rodrigo Ferrer

79

◦ Wd→a : ąQ = ⇒ Wd→a =

P dV −Qd→a > 0 Z So T dS −

=

Sf

−To (So − Sf ),

=

To constante

El gas recibe energ´ıa en forma de trabajo. Trabajo neto:

W = = = =

Wa→b + Wb→c + Wc→d + Wd→a Cv (Tf − To ) − Qb→c + Cv (To − Tf ) − Qd→a −Qb→c − Qd→a −|Qb→c | + |Qd→a | < 0.

(5.3)

De la ecuaci´on (5.3) podemos concluir que el sistema realiza un trabajo. Q

= = = ⇒ Q+W =

Qb→c + Qd→a |Qb→c | − |Qd→a | −W 0.

Definimos el rendimiento como η≡

flujo neto de calor . calor recibido

(5.4)

En nuestro caso, Q Qb→c Qb→c + Qd→a = Qb→c Qd→a = 1+ Qb→c

η =



η

=

1−

To Tf

(5.5)

80

CAP´ ITULO 5

´ CICLOS TERMODINAMICOS

Si tomamos el ciclo de Carnot en el sentido contrario, estamos sacando calor de un cuerpo a temperatura To y d´andole trabajo al gas que, a su vez, est´a sacando calor a un cuerpo a temperatura Tf . Por lo tanto ¡TENEMOS UN REFRIGERADOR! En este sentido, el ciclo de Carnot es un “antirefrigerador”, una bomba de calor (estufa). En esta bomba de calor, definimos el rendimiento como

QNeto

η =

QAbsorbido −WNeto = QAbsorbido QAbsorbido + QBotado = QAbsorbido QBotado , = 1+ QAbsorbido

donde η ser´a menor o igual a 1, garantizado. Lo que importar´a en el refrigerador, es la raz´on entre el calor absorbido y el trabajo entregado al gas.

5.2.

Ciclo de Carnot con Entrop´ıa versus la Temperatura. S Sf

adiabata isentrópica

d Qd

a

c

Área Qb

So

a

adiabata isentrópica

c

b T

To

Tf

Figura 5.2: Ciclo de Carnot: Entrop´ıa vs. Temperatura.

En la figura 5.2 podemos ver que el flujo neto de calor tambi´en viene dado por

5.2 Ciclo de Carnot con Entrop´ıa versus la Temperatura.

Rodrigo Ferrer

81

Q = Qb→c + Qd→a = Tf (Sf − So ) + To (So − Sf ) = (Tf − To )(Sf − So ) ´ = Area Rendimiento: η=

´ Q Area (Tf − To )(Sf − So ) = = ´ Qb→c Tf (Sf − So ) Area 1

(5.6)

Podemos ver la situaci´on anterior como en el siguiente gr´afico: T Tf

Qb

b

c

c

Área

To

a

d Qd

a

S So

Sf

Figura 5.3: Ciclo de Carnot: Temperatura vs. Entrop´ıa.

En general, adem´as del gas ideal, un gas, un l´ıquido o un s´olido pueden ser sometidos a un ciclo de Carnot, cuyas etapas resumimos: 1. Compresi´on isentr´opica a S = So 2. Expansi´on isot´ermica a temperatura Tf 3. Expansi´on isentr´opica a S = Sf 4. Compresi´on isot´ermica a temperatura To

CAP´ ITULO 5

82

´ CICLOS TERMODINAMICOS P b Pb isoterma

( Th ) c

Pc

adiabata ( S2 )

adiabata ( S1 )

Pa a

Pd

isoterma ( Tc )

d V

Vb

Va

Vc

Vd

Figura 5.4: Ciclo de Carnot: Presi´on vs. volumen.

5.3.

Ciclo de Carnot con la Presi´ on versus el Volumen.

En la figura 5.2, el proceso b → c representa una fuente caliente o reservoir de energ´ıa; mientras que el proceso d → a representa un reservoir de fuente fr´ıa. El sistema “bota” trabajo: −W = Qh +Qc . Esto implica que no todo el calor entregado se puede transformar en trabajo. Este es el segundo ´ mica. principio de la termodina I

Z dS

=

2

Z dS +

1

4

dS = 0 3

Qh Qc + = 0 Th Tc Th Qh ⇒ = − Tc Qc Qh ⇒ Th = −Tc Qc ⇒

OJO: Qc < 0 OJO: Qh > 0 Tc (o C)

Th (o C)

 (Carnot)

erp

 (observado)

Planta de vapor de carb´on quemado de West Thurrock (U.K.) ∼ 25 Reactor Nuclear PHW, CANDU (Canad´a) ∼ 25 Planta de vapor geotermal Larderello (Italia) 80

565 300 250

0.64 0.48 0.32

0.40 0.28 0.175

0.36 0.30 0.16

Planta de Poder

Tabla 5.1: Rendimiento de plantas de poder comparadas con el rendimiento de Carnot y con el rendimiento de un motor endoreversible maximizado para la potencia de salida (erp ).

5.4 Ciclo de Joule

Rodrigo Ferrer

5.4.

83

Ciclo de Joule

Consideremos un gas ideal haciendo el siguiente ciclo: P c

b Pf

adi

aba adi

aba

ta

ta

d Po

a V V2

V3 V1

V4

Figura 5.5: Ciclo de Joule: Presi´on vs. volumen.

Hay un intercambio de calor en las etapas b → c y d → a. ◦ Proceso b → c :

ąQb→c = Cp dT − V dP ⇒ ąQb→c = Cp dT ya que P es constante Z P dV ⇒ Qb→c = Cp NR P2 ⇒ Qb→c = Cp (V3 − V2 ) > 0 NR ∴ Entra calor al gas. ◦ Proceso d → a :

Qd→a = Cp

P1 (V1 − V4 ) > 0 NR

∴ Sale calor del gas. Calculemos el rendimiento:

η=

Qb→c + Qd→a Qd→a P1 V1 − V4 =1+ =1+ Qb→c Qb→c P2 V3 − V2

(5.7)

84

CAP´ ITULO 5

´ CICLOS TERMODINAMICOS

Pero en las adiabatas se cumple 

P1 V1γ = P2 V2γ P1 V4γ = P2 V3γ

V2 V1 = , que al reemplazar en (5.7), resulta V4 V3   V1  1 1 − V4 P1 V4 P1 P 1 γ   =1− η =1− P2 V3 1 − V2 P2 P 2

Dividiendo estas ecuaciones, obtenemos

V3





5.5.

η

=

1−

P1 P2

 γ−1 γ

(5.8)

Ciclo de Otto

Consideremos un gas ideal haciendo el siguiente ciclo: P c

P3

Qb

explosión

c

P2 P1

d b compresión Qd

Po

a

a V1

V2

V

Figura 5.6: Ciclo de Otto: Presi´on vs. volumen.

Este ciclo simula motores bencineros de cuatro tiempos, es decir, la mayor´ıa de los motores de los veh´ıculos que circulan por la ciudad.

5.5 Ciclo de Otto

Rodrigo Ferrer

85

Figura 5.7: Motor de cuatro tiempos.

Las v´alvulas de admisi´on dejan entrar mezcla desde el carburador; las de escape, permiten botar los gases de la combusti´on. El pist´on se mueve hacia arriba y hacia abajo, haciendo rotar el cig¨ ue˜ nal, el que a su vez gira los pi˜ nones de la caja de cambio, que transmiten distintas revoluciones a las ruedas v´ıa diferenciales, etc. El pist´on toma una posici´on de m´axima compresi´on y, en ese momento, la buj´ıa da el “chispazo”. Debido a la enorme presi´on, el pist´on baja, moviendo el cig¨ ue˜ nal. Cuando esto ocurre, ambas v´alvulas est´an cerradas. Antes de la explosi´on, el pist´on “aspira” la mezcla al abrirse, la v´alvula de admisi´on; mientras, la v´alvula de escape se encuentra cerrada. Para expulsar el producto inservible de la combusti´on, el sistema debe abrir la v´alvula de escape. En ese momento, la v´alvula de admisi´on se encuentra cerrada. El ciclo se repite, involucrando nuevamente cambios de volumen, presi´on y temperatura. En la figura (5.6), podemos apreciar que en el proceso de (0, Po ) → a (donde entra la mezcla), est´a abierta la v´alvula de admisi´on, y cerrada la de escape. En los procesos a → b (donde se comprime la mezcla), b → c (donde se produce la chispa) y c → d (donde ocurre la explosi´on), ambas v´alvulas est´an cerradas. En el proceso d → a (donde la energ´ıa es entregada), est´a cerrada la v´alvula de admisi´on, mientras que la de escape se encuentra abierta. Finalmente, de d → (0, Po ) (donde salen los gases de la combusti´on), est´a abierta la v´alvula de escape; mientras que la de admisi´on se encuentra cerrada. Calculemos el rendimiento:

86

CAP´ ITULO 5

´ CICLOS TERMODINAMICOS

η=

Qb→c + Qd→a . Qb→c

Tc

Z Qb→c =

dT

Cv Tb

= Cv (Tc − Tb ) El gas es ideal, luego T =

PV . Con esto, NR  P3 V 2     Tc = N R

⇒ Tc > Tb

  PV   Tb = 2 2 NR

Esto nos dice que Qb→c > 0, es decir, entra calor al gas (debido a la chispa).

Z Qd→a =

Ta

Cv

dT Td

= Cv (Ta − Td )

 Po V 1   T =  a  NR

⇒ Td > Ta

  PV   Td = 4 1 NR

Esto nos dice que Qb→c < 0, es decir, sale calor del gas. Con esto, el rendimiento queda

η =1+

Qd→a Cv (Ta − Td ) =1+ . Qb→c Cv (Tc − Tb )

Pero Tb V2γ−1 = Ta V1γ−1 y Tc V2γ−1 = Td V1γ−1 . Por lo tanto

5.5 Ciclo de Otto

Rodrigo Ferrer

(Tb − Tc )V2γ−1 = Ta − Td = ⇒ Tc − Tb ⇒

η

=



η

=

87

(Ta − Td )V1γ−1 V γ−1 − 2γ−1 V1 V γ−1 1 − 2γ−1 V1 1 1 −  γ−1 V1 V2



η



1−

1 rγ−1

(5.9)

r es llamada la “raz´on de compresi´on”. Nota: En la pr´actica se necesita r < 10 para que no haya pre-encendido. 1 Si r = 9 y γ = 1.5, entonces η = 1 − √ = 0.67 = 67 %; aunque en la pr´actica es mucho menor. 9

5.5.1.

Motor de Dos Tiempos

Figura 5.8: Motor de Dos Tiempos. En una vuelta, el cig¨ ue˜ nal realiza la secuencia de eventos del motor de cuatro tiempos. En el fondo, se est´an eliminando las etapas de admisi´on y escape.

88

5.6.

CAP´ ITULO 5

´ CICLOS TERMODINAMICOS

Ciclo de Diesel

Consideremos un gas ideal haciendo el siguiente ciclo: P b

P2

c adiabata

P3

d adiabata

P1

a

Vb

V

Va

Vc

Figura 5.9: Ciclo de Diesel: Presi´on vs. volumen.

En el proceso b → c ocurre la inyecci´on de petr´oleo Diesel. Se dosifica para mantener la presi´on constante (alta presi´on). Aqui hay auto-encendido. En el proceso d → a se abre la v´alvula de escape y se le insufla aire para que bote el producto de la combusti´on. Para el gas ideal tenemos



Qb→c = Cp (Tc − Tb ) > 0 Qd→a = Cv (Ta − Td ) < 0

Luego T 1 Td (1 − Tad ) Cv (Ta − Td ) =1− . η =1+ Cp (Tc − Tb ) γ Tc (1 − TTb ) c

De la ecuaci´on del gas ideal tenemos 

P2 Vb = N RTb P2 Vc = N RTc



Vb Tb = . Vc Tc

5.7 Cooling Energy Ratio

Rodrigo Ferrer

  Td Vaγ−1 = Tc Vcγ−1 

Ta Vaγ−1

=



Tb Vbγ−1

89

 Ta Tb Vbγ−1    =   Td Tc Vcγ−1   T Vcγ−1    d = Tc Vaγ−1

Finalmente, el rendimiento resulta ser  Vbγ 1 − γ Vc 1   η = 1− γ 1 − VVcb   γ  γ 1 − Vb Vc 1 (Vc /Va )    = 1− γ (Vc /Va ) 1 − Vb 

Vcγ−1 Vaγ−1

Vc

1 1  γ −  γ Va Vc

1 = 1− γ

1 1  −  Va Vc

Definiendo la raz´on de expansi´on como r ≡

η

=

Va Vb

Va Vb

Va Va , y la raz´on de compresi´on como rc ≡ . Vc Vb

 γ  γ 1 1 − 1 r rc 1− ·     1 1 γ − r rc

(5.10)

Por ejemplo, sea rc = 15, r = 5, γ = 1.5. Entonces η = 64 %, que tambi´en es menor en la pr´actica.

5.7.

Cooling Energy Ratio

Definimos cooling energy ratio por

ω

=

Qabsorbido Wneto

Analicemos ω para el caso del ciclo de Carnot invertido (refrigerador):

90

CAP´ ITULO 5

´ CICLOS TERMODINAMICOS T

Qc

b

b

c

Tf

adiabata

adiabata

d

a To Qa

d

V Vb

Va

Vc

Vd

Figura 5.10: Ciclo de refrigeraci´on (ciclo de Carnot invertido)

Aqui, Qa→d es el calor absorbido, mientras que el trabajo neto est´a dado por (−Qa→d − Qc→d ). Luego

ω = =

Qa→d −Qa→d − Qc→b −Qc→b − W W

= −1 −

Qc→b . W

Por ejemplo, sea ω = 5 (Qabsorbido = 5 veces W ). En ese caso,

ω = ⇒ 5 = ⇒ 6 =

Qc→b W Qc→b −1 − W Qb→c . W

−1 −

Esto significa que el calor entregado es de 6W , seis veces el trabajo neto realizado. Seg´ un (5.1) y (5.2), tenemos, para este caso,

5.8 Trabajo M´aximo

Rodrigo Ferrer

Qa→d −Qa→d − Qc→b

ω =

N RTo ln VVad

=

−N RTo ln VVad + N RTf ln VVcb 1

=

V Tf ln Vcb −1 + To ln VVd a

1 , Tf ln rc −1 To ln re

=

donde rc ≡

Vc Vb

91

es la raz´on de compresi´on y re ≡

Vd Va

es la raz´on de expansi´on.

Si rc = re , ω resulta ser ω=

1 Tf To

−1

=

To . Tf − To

Con este resultado, podemos escribir W =

Qabsorbido ω



W =

Qabsorbido · (Tf − To ). To

Si Tf ≈ To , entonces W ≈ 0. Esto nos dice que no se requiere trabajo neto para sacarle calor a la fuente a temperatura To . Si Tf  To , se tiene W → ∞, es decir, cuesta mucho trabajo neto enfriar un poco el sistema.

5.8.

Trabajo M´ aximo

En todo proceso termodin´amico, se cumple que ∆S ≥ 0. ∆S > 0 si el proceso es irreversible y ∆S = 0 si es reversible. En este motor, tenemos:

∆Q + ∆W = 0 ⇒ ∆Qsale + ∆Qentra + ∆Wneto = 0 ∆Qsale = −T2 ∆S2 < 0 ∆Qentra = −T1 ∆S1 > 0

92

CAP´ ITULO 5

´ CICLOS TERMODINAMICOS

Luego ⇒ ⇒ ⇒ ⇒ ⇒ ⇒ ⇒

∆S ∆S1 + ∆S2 ∆Qsale ∆S1 − T2 (∆Qentra − ∆W ) ∆S1 − T2 T2 ∆S1 + ∆Qentra + ∆W T2 ∆S1 + ∆Qentra −∆W ≤ T2 ∆S1 + ∆Qentra −∆W ≤ (T2 − T1 )∆S1

≥ 0 ≥ 0 ≥ 0 ≥ 0 ≥ 0 ≥ −∆W

Este u ´ltimo, es es trabajo que sale del motor (el trabajo obtenible). Esto dice que obtener el m´aximo trabajo de un sistema, corresponde a un proceso reversible, y adem´as ∆Wmax = ∆Qentra + T2 ∆S1 En la situaci´on anterior, el cuerpo a temperatura T1 se va enfriando (T1 disminuye paulatinamente) y el cuerpo a temperatura T2 se va calentando (T2 va aumentando paulatinamente). Cuando las temperaturas se igualan, ya no es posible obtener m´as trabajo. Tenemos ∆Qentra + ∆Qsale + ∆W = 0 Z Tf Z ⇒ −∆W = −Cp dT − Cp T1

= =

Tf

dT

T2

−Cp (Tf − T1 ) − Cp (Tf − T2 ) Cp (T1 + T2 − 2Tf ).

La ecuaci´on (5.11) representa el m´aximo trabajo que podemos extraer. En un proceso reversible

⇒ ⇒ ⇒ ⇒ ⇒ ⇒

∆S1 + ∆S2 ∆Q1 ∆Q2 + T1 T2 Z Tf Z Tf dT dT + Cp Cp T T T1 T2 Tf Tf ln + ln T T2 1  Tf Tf ln · T1 T2 Tf Tf · T1 T2 Tf

=

0

=

0

=

0

=

0

=

0

=

1 p T1 · T2

=

(5.11)

5.8 Trabajo M´aximo

Rodrigo Ferrer

93

La temperatura final alcanzada por ambos cuerpos es el proceso reversible es el promedio geom´etrico de las temperaturas. De este modo, se obtiene el trabajo m´aximo. Sistema Estado A

Estado B ( ∆ U) = UA

QRHS

Fuente de calor reversible

UB

W RWS

Fuente de trabajo reversible

Figura 5.12: Proceso de trabajo m´aximo. El trabajo entregado WRW S es m´aximo y el calor entregado QRHS es m´ınimo si el proceso completo es rreversible (∆Stotal = 0).

Si U = N CT ,   p ∆Wmax = N C T1 + T2 − 2 T1 T2 .

(5.12)

94

CAP´ ITULO 5

´ CICLOS TERMODINAMICOS

Parte II Teor´ıa Cin´ etica de los Gases

Cap´ıtulo 6 Movimiento de las Part´ıculas 6.1.

Ecuaciones de estado de un gas ideal

Las hip´otesis fundamentales de la teor´ıa cin´etica de un gas ideal son: 1. Toda peque˜ na muestra de gas contiene muchas mol´eculas id´enticas (en el sentido cl´asico). Si hay N mol´eculas de masa m, la masa total ser´a de N m. Si la masa molar (la masa de un mol de mol´eculas) es δ, tendremos que n=

Nm ´ mero de moles ≡ nu δ

(6.1)

En un volumen de 22.4 L, a una temperatura de 300K y presi´on de 1 atm, hay un mol de a´tomos. Luego, en la densidad molecular ser´a   ´culas] ´ 6.06225 × 1023 [mole 7 moleculas ρ= = 3 × 10 . 22.4 × 103 [cm]3 [µm]3

(6.2)

2. Las mol´eculas pueden considerarse como esferas duras en perpetuo y azaroso movimiento, de un di´ametro promedio de 2.5 × 10−8 cm. 3. Las mol´eculas no interact´ uan entre ellas hasta que chocan. El choque es el´astico, asi como el choque contra las paredes del recipiente. El movimiento, antes y despu´es de colisionar, es rectil´ılneo. El camino libre medio entre las mol´eculas, dentro del rango de temperatura y presi´on de un gas ideal, es del orden de 50 veces sus di´ametros. 4. Las paredes del recipiente son perfectamente lisas y el choque de las mol´eculas contra ellas es perfectamente el´astico. La situaci´on se muestra en la Figura 6.1, donde el cambio total en la velocidad est´a dado por ∆wk =

0

wk − wk 0

=0

∆w⊥ = w⊥ − w⊥ = −w⊥ − w⊥ = −2w⊥

98

CAP´ ITULO 6

MOVIMIENTO DE LAS PART´ ICULAS

ω ω

ω’

ω’

11 00 00 11 00 11 00 11 00 11 00 11 00 11 00 11 00 11 00 11 00 11 00 11 00 11 00 11 00 11 00 11 00 11

Figura 6.1: Mol´ecula chocando contra una pared.

5. En ausencia de agentes externos, las mol´eculas se distribuyen uniformemente en el volumen que las contiene. Para la densidad molecular se tiene ∆N dN N = = V ∆V dV

Para que esta u ´ltima igualdad sea cierta, el diferencial dV debe contener muchas part´ıculas. 6. No existen direcciones privilegiadas para el movimiento de las mol´eculas. En todo momento hay tantas part´ıculas movi´endose en una direcci´on como en otra. (Ojo: no estamos hablando del m´odulo de la velocidad). 7. No todas las mol´eculas tienen la misma velocidad (en m´odulo). Como concluiremos al final del pr´oximo cap´ıtulo, hay pocas part´ıculas moviendose lentamente, y pocas movi´endose r´apidamente.

p p

p’ p’

11 00 00 11 00 11 00 11 00 11 00 11 00 11 00 11 00 11 00 11 00 11 00 11 00 11 00 11 00 11 00 11 00 11

Figura 6.2: Mol´ecula chocando contra una pared.

6.1 Ecuaciones de estado de un gas ideal

Rodrigo Ferrer

99

El cambio de momentum est´a dado por ∆~p = −2~p⊥ . La presi´on sobre la pared est´a dada, para un a´rea A, por P =

F 1 dptotal = A A dτ

(6.3)

Para conocer el cambio total del momentum lineal (∆ptotal ), es necesario conocer la velocidad de cada una de las mol´eculas que chocan contra la unidad de a´rea por unidad de tiempo. Consideremos un sistema unidimensional. Sea Nwx ≡ N (wx ), el n´ umero de part´ıculas con velocidad wx . Entonces dNwx es el n´ umero de part´ıculas o puntos con velocidades comprendidas entre wx y wx + dwx . Este diferencial ser´a funci´on de wx , por lo tanto

dNwx =

Nw

dNwx dwx dwx

(6.4)

x

1111111111111111111111 0000000000000000000000 0000000000000000000000 1111111111111111111111 0000000000000000000000 1111111111111111111111 0000000000000000000000 1111111111111111111111 0000000000000000000000 1111111111111111111111 0000000000000000000000 1111111111111111111111 0000000000000000000000 1111111111111111111111 0000000000000000000000 1111111111111111111111 0000000000000000000000 1111111111111111111111 0000000000000000000000 1111111111111111111111 0000000000000000000000 1111111111111111111111 0000000000000000000000 1111111111111111111111 0000000000000000000000 1111111111111111111111 0000000000000000000000 1111111111111111111111 0000000000000000000000 1111111111111111111111 0000000000000000000000 1111111111111111111111 0000000000000000000000 1111111111111111111111 0000000000000000000000 1111111111111111111111 0000000000000000000000 1111111111111111111111 0000000000000000000000 1111111111111111111111 0000000000000000000000 1111111111111111111111 dw x

wx

Figura 6.3: Cantidad de part´ıculas con velocidad wx . Integrando sobre todas las velocidades tenemos el n´ umero total de mol´eculas (part´ıculas): Z N=

Z dNwx = 0



dN dwx . dwx

An´alogamente, para el caso bidimensional Nwx ,wy ≡ N (wx , wy ) (Figura 6.4), denotamos por d2 Nwx wy al n´ umero de mol´eculas con velocidades entre w ~ = wx xˆ + wy yˆ , y w ~ + dw ~ = (wx + dwx )ˆ x + (wy + dwy )ˆ y.

100

CAP´ ITULO 6

MOVIMIENTO DE LAS PART´ ICULAS

Nw

xw y

wy +dw y

wy

wy

wx

wx +dw x

wx

Figura 6.4: Espacio de velocidades bidimensional.

Integrando sobre todas las velocidades, obtenemos el n´ umero total de part´ıculas (mol´eculas): Z

Z N=





Z

dNwx wy = 0

0

d2 Nwx wy dwx dwy . dwx dwy

Usando coordenadas polares planas (Figura 6.5), d2 Nwφ representa la cantidad de part´ıculas con velocidades en el a´rea w dφdw del espacio de velocidades. Como la distribuci´on es homog´enea,

d2 Nwφ = dNw

dφ , 2π

(6.5)

dφ donde dNw representa la cantidad de part´ıculas con velocidades entre w y w + dw, y el factor 2π representa la fracci´on de mol´eculas que se encuentran en el ´angulo dφ. Nw

xw y



w dφ

wy

dw

wx

Figura 6.5: Espacio de velocidades bidimensional en coordenadas polares.

6.2 Choques Contra una Pared

Rodrigo Ferrer

101

En el espacio tridimensional de velocidades (Figura 6.6), todos los puntos de un volumen diferencial tienen sus velocidades comprendidas entre w ~ yw ~ + dw. ~ A la cantidad de part´ıculas en este volumen 3 la llamaremos d Nwθφ .

Figura 6.6: Espacio de velocidades tridimensional

Definimos el a´ngulo s´olido como dΩ ≡

´ rea subtendida a w sin θ dφ · w dθ = = sin θ dφ dθ 2 w w2

Por homogeneidad, d3 Nwθφ = dNw

dΩ sin θ dφ dθ = dNw 4π 4π

(6.6)

Luego Z N=

6.2.

d3 Nwθφ .

Choques Contra una Pared

Consideremos el cilindro diferencial definido en la Figura 6.7. Aqui, dτ es un intervalo infinitesimal de tiempo, por lo que el largo del cilindro es w dτ , y su a´rea basal, dA. Es claro que todas las mol´eculas del tipo (w, θ,φ) dentro de este cilindro chocan con el a´rea dA, a menos que sean

102

CAP´ ITULO 6

MOVIMIENTO DE LAS PART´ ICULAS

z dA

ω dτ

θ y

φ ω

x Figura 6.7: Choques contra una Pared.

expulsadas por otras mol´eculas. Para que esto no suceda, tomamos dτ lo suficientemente peque˜ no. Estamos suponiendo que el espacio es homog´eneo, por lo que



N V

=

dN dV

dN N

=

dV dA · w dτ cos θ = V V

(6.7)

Luego el n´ umero de mol´eculas provenientes del cilindro, golpeando el ´area dA, ser´a   dV 3 d Nwθφ V

(6.8)

El cambio en el momentum en la colisi´on est´a dado por ∆p = −2mw cos θ. Reemplazando (6.6) y (6.7), el cambio total del momentum queda determinado por

" dptotal =

 # "Fracci´on de estas# Cambio de momentum  · mol´eculas golpean- · debido al

n´ umero de mol´eculas en el ´ angulo s´ olido dΩ con velocidad w

= d3 Nwθφ · = dNw ·

do la pared

choque

dV · (−2mw cos θ) V

sin θ dθ dφ dA · w dτ cos θ · · (−2mw cos θ) 4π V

6.2 Choques Contra una Pared

Rodrigo Ferrer

103

Luego, de (6.3), conclu´ımos que la contribuci´on a la presi´on de estas mol´eculas con velocidad w, estar´a dado por 1 1 (−dptotal ) dA dτ Z 2π Z π/2 1 1 sin θ dθ dφ dA · w dτ cos θ = dNw · · · (2mw cos θ) , dA dτ 4π V 0 0 Z 2π Z π/2 2 dNw = 2mw cos2 θ sin θ dθ dφ , 4πV 0 0

dPw =

eligiendo u = − cos θ, dPw = 4mπw

= mw2

=

2 dNw

4πV

dNw V

Z

0

u2 du

−1

! u , 3 −1 3 0

mw2 dNw . 3 V

Considerando todos los posibles w tal que 0 < w < ∞,   Z Z 1 m ∞ 2 dNw 2 PV = w mw dNw = dw , 3 3 0 dw donde

(6.9)

dNw representa la distribuci´on de los m´odulos de las velocidades. dw

Para calcular promedios con variables continuas, como, por ejemplo, el promedio de notas, usamos la expresi´on N X

hni =

n P (n)

n=1 N X

,

(6.10)

P (n)

n=1

donde P (n) es la cantidad de veces que aparece la variable (nota) n. Luego, el denominador es claramente N . En un continuo, tendremos Z ∞ x P (x) dx −∞ hxi = Z ∞ , (6.11) P (x) dx −∞

104

CAP´ ITULO 6

MOVIMIENTO DE LAS PART´ ICULAS

donde R ∞el denominador, al igual que en el caso discreto, es el total de “notas”, o cantidad de variables. Si −∞ P (x) dx = N ∈ R, definimos Po (x) = N1 P (x), con lo cual Z ∞ Po (x) dx = 1 (6.12) −∞

y, de esta forma, podemos asociar Po (x) dx a la probabilidad de encontrar la variable x en un intervalo dx, es decir, Po (x) es la densidad de probabilidad en un punto x. Haciendo estas consideraciones, podemos expresar el promedio como Z ∞ xPo (x) dx Z ∞ −∞ xPo (x) dx (6.13) = hxi = Z ∞ −∞ Po (x) dx −∞

Tomando (6.13) como la definici´on de promedio, podemos considerar, ahora, el promedio del cuadrado de las velocidades como Z 1 ∞ 2 dNw 2 hw i = w dw , (6.14) N 0 dw donde, como vimos en (6.12), Po (w) =

1 dNw . Al reemplazar este promedio en (6.9), obtenemos N dw

1 P V = N mhw2 i , 3   2 1 2 = N mhw i , 3 2 2 ´tica por mole ´culai . = N henerg´ıa cine 3 Pero, como vimos en (2.3), P V = nRT , con n el n´ umero de moles y R = 8.3143

J . mol K

Si consideramos la constante de Boltzmann kB =

J n R = 1.3807 × 10−23 , N K

entonces, 1 N kB T = N mhw2 i , 3 3 1 kB T = mhw2 i . 2 2 Nota: En este gas s´olo se considera la energ´ıa cin´etica de traslaci´on:

(6.15)

6.2 Choques Contra una Pared

Rodrigo Ferrer

1 1 1 1 mhw2 i = mhwx2 i + mhwy2 i + mhwz2 i , 2 2 2 2 1 1 1 = kB T + kB T + kB T , 2 2 2 3 = kB T 2 Esto nos dice que hay una equipartici´on de la energ´ıa.

105

106

CAP´ ITULO 6

MOVIMIENTO DE LAS PART´ ICULAS

Cap´ıtulo 7 F´ ormula de Maxwell Para la Distribuci´ on de Velocidades 7.1.

Deducci´ on de la Ecuaci´ on

umero Consideremos el espacio de velocidades. Del total de N mol´eculas, denotemos por dNwx el n´ de part´ıculas o puntos con componente x de la velocidad comprendida entre wx y wx + dwx . wz

wy dw x

wx

Figura 7.1: N´ umero de particulas con velocidades entre (los planos) wx y wx + dwx . Geom´etricamente, dNwx representa la cantidad de puntos que hay en la capa de espesor dwx de la Figura 7.1. La fracci´on del n´ umero total N que hay en la capa es 1 dNwx dNwx = dwx . N N dwx Este diferencial depender´a de la capa considerada, es decir, depender´a de wx . Si el espesor es peque˜ no la fracci´on ser´a proporcional a este espesor. Es decir, tendremos dNwx = f (wx )dwx N

(7.1)

108

CAP´ ITULO 7

´ FORMULA DE MAXWELL

Por lo tanto, el n´ umero de mol´eculas con velocidades en x entre wx y wx + dwx es dNwx = N f (wx ) dwx .

(7.2)

Como todas las direcciones son igualmente posibles, dNwy = f (wy )dwy , N dNwz = f (wz )dwz . N

(7.3) (7.4)

Ahora, nos interesa saber la fracci´on del n´ umero total de mol´eculas con velocidades en x entre wx y wx + dwx que, al mismo tiempo, tienen velocidades en y entre wy y wy + dwy . La suposici´on de Maxwell fu´e que la fracci´on del n´ umero total de mol´eculas que posee velocidades en cierto rango, es la misma fracci´on que la de un subgrupo, siempre que el subgrupo sea lo suficientemente grande. Es decir, del grupo de mol´eculas con velocidades en x entre wx y wx + dwx , el cual contiene dNwx mol´eculas (ver Figura 7.1), tomamos el subgrupo con velocidades en y entre wy y wy + dwy (el cual contiene d2 Nwx wy mol´eculas); y afirmamos que la fracci´on de mol´eculas con velocidades en y entre wy y wy + dwy de este grupo, es la misma que la fracci´on de mol´eculas con velocidades en y entre wy y wy + dwy del n´ umero total de mol´eculas. Para establecer una analog´ıa, digamos que la fracci´on de mujeres mayores de cuarenta a˜ nos, del 2 total de habitantes en Chile, es 5 , es decir, cada 5 habitantes, hay 2 mujeres mayores de cuarenta. Lo que estamos afirmando es que si tomamos el subgrupo “habitantes de Temuco”, por ejemplo, encontraremos que la cantidad de mujeres mayores de cuarenta dividida por la cantidad de habitantes de Temuco, ser´a tambi´en 52 , ya que la poblaci´on est´a distribu´ıda homog´eneamente y Temuco contiene una cantidad lo suficientemente grande de habitantes como para hacer esta estad´ıstica. La fracci´on del n´ umero de mol´eculas con velocidades en x entre wx y wx + dwx que, al mismo tiempo, tienen velocidades en y entre wy y wy + dwy , es d2 Nwx wy dNwy = , dNwx N ya que el total de mol´eculas en este grupo es dNwx . Por la condici´on de Maxwell, esta fracci´on ser´a igual a la fracci´on del n´ umero total de mol´eculas con velocidades en y entre wy y wy + dwy , dada por (7.3), dNwy = f (wy )dwy . N Por lo tanto d2 Nwx wy = dNwx f (wy )dwy . Pero, por la ecuacion (7.2), dNwx = N f (wx ) dwx . Luego

(7.5)

7.1 Deducci´on de la Ecuaci´on

Rodrigo Ferrer

d2 Nwx wy = N f (wx )f (wy ) dwx dwy

109

(7.6)

Geom´etricamente, d2 Nwx wy representa el n´ umero de puntos en aquella parte del espacio de velocidades que es com´ un a las dos franjas perpendiculares a los ejes wx y wy respectivamente. Podemos verlo como el a´rea de la intersecci´on en la Figura 6.4, o como la cara paralela al plano wx − wy del cubo de la Figura 7.2. Tiene la forma de un prisma de secci´on recta rectangular (ya que no necesariamente dwx = dwy ) y se extiende desde −∞ a ∞ en el eje wz . wz

wy

dw x

wx

dw y

Figura 7.2: N´ umero de particulas con velocidades entre (los planos) wx y wx + dwx y entre wy y wy + dwy . Por el mismo razonamiento anterior tenemos d3 Nwx wy wz dNwz = d2 Nwx wy N

(7.7)

o sea: d3 Nwx wy wz = d2 Nwx wy

dNwz 1 dNwz = dNwx dNwy , N N N

y, finalmente, el n´ umero de mol´eculas que tienen simult´aneamente velocidades en x entre wx y wx + dwx , velocidades en y entre wy y wy + dwy , y velocidades en z entre wz y wz + dwz , ser´a d3 Nwx wy wz = N f (wx )f (wy )f (wz ) dwx dwy dwz .

(7.8)

110

CAP´ ITULO 7

´ FORMULA DE MAXWELL

Este es el n´ umero de mol´eculas en el elemento de volumen dwx dwy dwz de la Figura 7.3, es decir, el elemento de volumen com´ un al prisma vertical y la franja de ancho dwz . wz dw z

wy dw x dw y

wx

Figura 7.3: N´ umero de particulas con velocidades entre wx y wx + dwx , wy y wy + dwy y entre wz y wz + dwz . Denotando por ρ la densidad de mol´eculas en el espacio de velocidades, tenemos ρ≡

d3 Nwx wy wz = N f (wx )f (wy )f (wz ) dwx dwy dwz

(7.9)

Puesto que no hay direcciones privilegiadas, esta densidad es la misma en todos los puntos con wx2 + wy2 + wz2 = constante ≡ ω . Tenemos entonces las ecuaciones acopladas: N f (wx )f (wy )f (wz ) = cte , wx2 + wy2 + wz2 = cte . Diferenci´andolas tenemos: ∂f (wx ) ∂f (wy ) ∂f (wz ) f (wy )f (wz ) dwx + f (wx )f (wz ) dwy + f (wx )f (wy ) dwz = 0 , ∂wx ∂wy ∂wz wx dwx + wy dwy + wz dwz = 0 . Dividiendo la primera por f (wx )f (wy )f (wz ) resulta: 1 ∂f (wx ) 1 ∂f (wy ) 1 ∂f (wz ) dwx + dwy + dwz = 0 , f (wx ) ∂wx f (wy ) ∂wy f (wz ) ∂wz wx dwx + wy dwy + wz dwz = 0 .

(7.10)

7.1 Deducci´on de la Ecuaci´on

Rodrigo Ferrer

111

Multiplicando la segunda ecuaci´on por mβ y sumando tenemos 

     1 ∂f (wx ) 1 ∂f (wy ) 1 ∂f (wz ) + mβwx dwx + + mβwy dwy + + mβwz dwz = 0 . f (wx ) ∂wx f (wy ) ∂wy f (wz ) ∂wz

Ahora bien la restricci´on (7.10) nos dice, por ejemplo, que

wx =

q cte − wy2 − wz2 .

es decir s´olo dos componentes, en este caso wy y wz son independientes. Elijamos mβ tal que 1 ∂f (wx ) + mβwx = 0 . f (wx ) ∂wx entonces, la independencia lineal de wy y wz nos dice que 1 ∂f (wy ) + mβwy = 0 , f (wy ) ∂wy 1 ∂f (wz ) + mβwz = 0 . f (wz ) ∂wz Como la condici´on es an´aloga para las tres componentes, denotemos por i = {x, y, z}. Luego: 1 ∂f (wi ) + mβwi = 0 , f (wi ) ∂wi ⇒

d ln f (wi ) + mβwi = 0 . dwi

Integrando: 1 ln f (wi ) = − mβwi2 + ln αi . 2 Y entonces:

  1 2 f (wi ) = αi exp −β mwi , 2

(7.11)

con i = {x, y, z}. Reemplazando esta expresi´on en (7.8), obtenemos 1 1 1 2 2 2 d3 Nwx wy wz = N α1 α2 α3 e(−β 2 mwx ) e(−β 2 mwy ) e(−β 2 mwz ) dwx dwy dwz

(7.12)

112

CAP´ ITULO 7

´ FORMULA DE MAXWELL

Con esto, y definiendo α1 α2 α3 ≡ A3 , la densidad ρ de (7.9) queda determinada por   d3 Nwx wy wz 1 2 2 3 2 ρ= = N A exp − βm(wx + wy + wz ) . dwx dwy dwz 2

y reemplazando (7.10), 

1 ρ = N α exp − βmw2 2 3

 (7.13)

Pero como vimos en (6.6),

d3 N = dNw

dΩ sin θ dφ dθ = dNw , 4π 4π

y, adem´as dwx dwy dwz = dV = w2 sin θ dw dθ dφ . Luego,   d3 Nwx wy wz 1 3 2 ρ= = N A exp − βmw , dwx dwy dwz 2   dNw sin θ4πdφ dθ 1 3 2 = N A exp − βmw , ⇒ 2 w sin θ dw dθ dφ 2 ⇒

1 dNw 2 = 4πw2 N A3 e− 2 βmw . dw

w Como el lector recordar´a, en (6.12) vimos que N1 dN tiene un sentido probabil´ıstico, ya que su dw integral sobre todo el espacio es uno, y representa la probabilidad de que una mol´ecula tenga velocidad w. Luego

1 N 3

Z

⇒ 4πA

0

Definiendo α = 21 βm,



Z



0 1

dNw dw = 1 , dw 2

w2 e− 2 βmw dw = 1 .

7.1 Deducci´on de la Ecuaci´on

Rodrigo Ferrer

1 = 4πA3

Z



113

2

w2 e−αw dw ,

0

1 ∂ = − 3 4πA ∂α



Z

2

e−αw dw .

0

Definamos Z



e

I= 0

−αw2

r Z ∞ 1 2 dw = e−x dx . α 0

Sabemos que esta integral es invariante ante un cambio de ´ındice (x → y). Por lo tanto, podemos decir que "r Z # "r Z # ∞ ∞ 1 1 2 2 I2 = e−x dx e−y dy , α 0 α 0 Z Z 1 ∞ ∞ −(x2 +y2 ) e dx dy . = α 0 0 Pasando a coordenadas polares, y como estamos barriendo todo el espacio tal que x > 0 e y > 0 (un cuarto de espacio), se tiene Z Z 1 π/2 ∞ −r2 2 e r dr dθ , I = α 0 0 Z ∞ π 2 e−r r dr , = 2α 0 Z ∞ π 2 = e−r d(r2 ) , 4α 0 π −r2 ∞ e , =− 4α 0 π = . 4α Por lo tanto, r Z ∞ 1 π −αw2 I= e dw = . 2 α 0 Luego, 1 ∂ =− 3 4πA ∂α

Z



2

e−αw dw ,

0

∂ 1 √ −1/2 =− πα ∂α 2 √ π 1 = . 4 α3/2 

 ,

114

CAP´ ITULO 7

´ FORMULA DE MAXWELL

despejando A, 3

A =

 α 3/2

, π 3/2  βm . = 2π

Con esto, la probabilidad de que una mol´ecula tenga velocidad w queda expresada como dNw = 4πw2 N dw



βm 2π

3/2

1

e− 2 βmw

2

(7.14)

Para determinar β, tomemos la ecuaci´on (6.14) y reemplacemos en ella la u ´ltima ecuaci´on: 1 hw i = N 2

w2

0

 = 4π =



Z

βm 2π

dNw dw , dw

 32 Z



1

2

w4 e− 2 βmw dw ,

0

3 mβ

Pero por (6.15) 3 1 kB T = mhw2 i . 2 2 Entonces, 1 3 3 m = kB T , 2 mβ 2 β=

1 . kB T

Finalmente, la probablidad de que una mol´ecula tenga velocidad w est´a completamente determinada y su expresi´on es

dNw dw

=

4N √ π



m 2kB T

 32

2

− 12 m kw T

w2 e

B

´ ´ n de Maxwell-Boltzmann Esta es la llamada Distribucio Comprobaci´on experimental: Experimento de Estermann, Simpson y Stern.

(7.15)

7.2 Problemas

Rodrigo Ferrer

7.2.

115

Problemas

1. Muestre que el n´ umero de mol´eculas NT que golpean una unidad de ´area de una pared de un recipiente que contiene gas ideal a temperatura T en una unidad de tiempo, es r

N hwi NT = 4V

con hwi =

8kB T πm

(7.16)

Soluci´ on El n´ umero de mol´eculas golpeando el ´area dA en un intervalo de tiempo dτ es, como vimos en (6.8), d3 Nwθφ

dV , V

con dV = dA w dτ cos θ. Integrando por sobre todo el espacio, obtenemos que la cantidad de mol´eculas que golpean un a´rea dA de la pared en un tiempo dτ es ZZZ NT =

3



d Nwθφ

dV V

 ,

  ZZZ  w cos θ sin θ dθ dφ = dNw , 4π V    ZZZ  sin θ dθ dφ w cos θ dNw dw = dw 4π V Z = 0



dNw w dw dw

Z 0

π 2

sin θ cos θ dθ 4π V

Z



dφ , 0

Z 1 1 2π sin θ d(sin θ) , = N hwi 4πV 0 Z 1 N hwi = u du , 2V 0 =

N hwi . 4V

Calculemos hwi usando la Distribuci´on de Maxwell-Boltzmann: Z 1 ∞ dNw hwi = w dw , N 0 dw   32 Z ∞ 2 1 4N m − 1 mw = √ w3 e 2 kB T dw . N π 2kB T 0

(7.17)

116

CAP´ ITULO 7

´ FORMULA DE MAXWELL 1 m 2 kB T

El problema se reduce a calcular la integral. Definiendo α ≡

Z



1 3 −2

w e

mw2 kB T



Z

2

w3 e−αw dw ,

dw =

0

se tiene

0

∂ =− ∂α



Z

2

w e−αw dw ,

0

1 ∂ =− 2 ∂α

Z



2

e−αw d(w2 ) ,

0

 ∞ 1 ∂ 1 −αw2 =− − e , 2 ∂α α 0 =−

1 ∂ 1 , 2 ∂α α

1 = α−2 , 2  2 kB T =2 . m Reemplazando esta integral en (7.17), obtenemos  hwi =

8kB T πm

1/2 ,

Finalmente, la cantidad de mol´eculas que chocan por unidad de a´rea y por unidad de tiempo es r N 8kB T NT = (7.18) 4V πm

Para tener una idea, digamos que el gas est´a a una temperatura de T = 300 K, a una presi´on P = 1 atm, y que su masa sea m = 10−26 Kg. Por la ecuaci´on fundamental de los gases,

P V = N kB T ⇒ N =

PV . kB T

Entonces la cantidad de mol´eculas que golpean el recipiente el a´rea dA en un tiempo dτ es

7.2 Problemas

Rodrigo Ferrer

NT = = = = =

117

N hwi 4V r 1 PV 8kB T 4V kB T πm r P 8 4 kB T πm   ´ 27 moleculas 6.28 × 10 m2 · s   ´culas moles de mole 10428.18 m2 · s

Esto significa que a una plancha de un metro cuadrado la golpean 10428.18 moles de mol´eculas en cada segundo. Si se considera un c´ırculo de 1 mm de di´ametro en la pared del recipiente, a esta a´rea la estar´an golpeando 0.033 moles de mol´eculas en cada segundo, o, en otras palabras, 2 × 1022 mol´eculas en cada segundo.

2. Un recipiente de volumen V contiene un gas que se mantiene a temperatura constante. El gas se escapa por un agujero de a´rea A. La presi´on externa es tan peque˜ na que ninguna mol´ecula vuelve a entrar al recipiente. Muestre que la presi´on en cualquier instante t est´a dada por 0 P = Po e−k t , donde Po es la presi´on en t = 0. Calcule adem´as, k 0 en t´erminos de V , A y hwi. Soluci´ on Sabemos que el n´ umero de mol´eculas que golpean la unidad de a´rea en una unidad de tiempo es N hwi/4V . Ahora bien, N va cambiando con el tiempo, es decir, N es funci´on del tiempo. Escribimos N = N (t). El cambio en el n´ umero de mol´eculas que golpean la pared por unidad dN (t) de tiempo est´a dado por dt , por lo tanto dN (t) N Ahwi =− . dt 4V Luego,

dN Ahwi =− dt , N 4V ln y eligiendo to = 0,

N Ahwi =− (t − to ) , No 4V

(7.19)

118

CAP´ ITULO 7

´ FORMULA DE MAXWELL

ln

Ahwi N =− t, No 4V   Ahwi N = No exp − t . 4V

y como P V = N kB T , tenemos   Ahwi P = Po exp − t , 4V por lo cual k0 =

Ahwi . 4V

3. Un bulbo esf´erico de vidrio de 10 cm de radio, se mantiene a 300 K, excepto por un ap´endice de a´rea transversal 1 cm2 inmersa en l´ıquido N2 , como se muestra en la Figura 7.4. En su interior hay vapor de agua a 0.1 mmHg de presi´on. Suponiendo que cada mol´ecula de H2 O que entra en el ap´endice se queda ah´ı, encuentre el tiempo que se requiere para que la presi´on del bulbo baje a 10−6 mmHg.

Figura 7.4: Bulbo esf´erico.

Soluci´ on Sabemos que P = Po e −

A<w> t 4V

.

Por lo tanto, para t = tf , P = Pf = 10−6 mmHg. Despejando el tiempo, tenemos

7.2 Problemas

Rodrigo Ferrer

tf = −

119

Pf 4V ln , A < w > Po

=− A

4V q

8kB T πm

 · ln

10−6 , 10−1

4 · 5 · 43 π(10 × 10−2 )3 = q , 8·1.38×10−23·300 (10−2 )2 −27 3.14·14×10 ≈ 2.2 s .

4. Un cilindro cerrado de volumen 2V se divide en dos compartimentos iguales (V1 = V2 = V ) mediante un delgado tabique. El lado izquierdo contiene, inicialmente, gas ideal a presi´on Po ; mientras que el lado derecho est´a evacuado. En el instante inicial se practica un peque˜ no orificio de a´rea A en el tabique. Suponiendo que la temperatura permanece constante y es la misma en ambos lados, encuentre las presiones en cada lado para todo instante de tiempo.

Figura 7.5: Flujo de materia

Soluci´ on Sean N1 (t) y N2 (t) las funciones que representan el n´ umero de mol´eculas en los lados izquierdo y derecho, respectivamente, en funci´on del tiempo. En el instante inicial N1 (0) = N y N2 (0) = 0. En el orificio inciden mol´eculas de ambos lados, siendo N1 (t)hwi 4V

y

N2 (t)hwi 4V

(7.20)

el n´ umero de mol´eculas que impactan en el a´rea A por unidad de tiempo. En el lado izquierdo el cambio en la cantidad de mol´eculas ser´a dN1 (t) N1 (t)Ahwi N2 (t)Ahwi =− + . dt 4V 4V

(7.21)

donde el primer t´ermino representa a las mol´eculas que salen de este compartimento, y el segundo, a las que entran.

120

CAP´ ITULO 7

´ FORMULA DE MAXWELL

Para el segundo compartimento, el de la derecha, el cambio en la cantidad de mol´eculas estar´a dado por dN2 (t) N1 (t)Ahwi N2 (t)Ahwi = − . dt 4V 4V

(7.22)

donde el primer t´ermino representa a las mol´eculas que entran a este compartimento, y el segundo, a las que salen. Sumando (7.22) a (7.21) se tiene dN1 (t) dN2 (t) + = 0, dt dt d(N1 (t) + N2 (t)) = 0, dt lo cual es consistente con que N1 (t) + N2 (t) = N ≡ cte ∀ t , pero no nos da informaci´on para poder resolver el problema. Si restamos las ecuaciones, obtenemos dN1 (t) dN2 (t) Ahwi − = − [N1 (t) − N2 (t)] , dt dt 2V Ahwi d[N1 (t) − N2 (t)] =− dt , N1 (t) − N2 (t) 2V   N1 (t) − N2 (t) Ahwi t, ln =− N1 (0) − N2 (0) 2V 

 Ahwi N1 (t) − N2 (t) = [N1 (0) − N2 (0)] exp − t . 2V Pero N1 (0) = N y N2 (0) = 0. Adem´as, para todo t, N2 (t) = N − N1 (t) . Luego,  Ahwi N 1 + e− 2V t , 2  Ahwi N N2 (t) = 1 − e− 2V t . 2 N1 (t) =

7.2 Problemas

Rodrigo Ferrer

121

De la ecuaci´on del gas ideal tenemos P1 (t) =

N1 (t)kB T , V

P2 (t) =

N2 (t)kB T . V

Entonces,

 Ahwi Po  1 + e− 2V t , 2  Ahwi Po  P2 (t) = 1 − e− 2V t . 2 P1 (t) =

N kB T donde Po = V

r y

hwi =

8kB T . πm

5. Una ampolleta de 10 cm de radio se evac´ ua continuamente a alto vac´ıo. En ella hay un peque˜ no recipiente cerrado en el cual se ha practicado un agujero circular de radio 0.2 mm de di´ametro, situado en el centro de la ampolleta. El recipiente contiene mercurio a 100◦ C. Su presi´on de vapor es de 0.28 mmHg. a) Calcule hwi en el recipiente de Hg. b) Calcule la velocidad de efusi´on del mercurio a trav´es del agujero en mil´ıgramos/hora. c) Calcule el tiempo que se requiere para que se deposite un microgramo de mercurio en un cent´ımetro cuadrado de la superficie interior de la ampolleta en una direcci´on que forma 45◦ con la normal del agujero.

Soluci´ on a) Evaluando, r

8kB T , πm

s

  8 · 1.38 × 10−23 KJ · 373 K , 3.14 · 3.32 × 10−25 Kg

hwi =

=

= 198.7 m/s .

122

CAP´ ITULO 7

´ FORMULA DE MAXWELL

b) ve =

AN hwi , 4V

=

AP hwi , 4kB T

m π · 10−8 [m]2 · 0.28 [atm] · 198.7 760  s  = , 4 · 1.38 × 10−23 oJK · 373 o K h i 3 µg . = 13.28 × 10 h

c) Tenemos que N´umero de mol´eculas golpeando el ´area dA en un tiempo dτ en la sin θ dθ dφ dA w dτ cos θ  direcci´on del ´angulo s´olido dΩ = dNw · = . 4π V sin θ dθ dφ con velocidad cercana aw ~ = (w, θ, φ)

Por unidad de a´ngulo s´olido de ´area y de tiempo,

dNw

w cos θ 4πV

Considerando todas las velocidades, Z dNw ·

w cos θ N hwi cos θ = , 4πV 4πV

considerando por unidad de tiempo, de a´rea y de ´angulo s´olido. El ´angulo s´olido que subtiende 1 cm2 a una distancia de 1 cm es 1 dΩ = 2 = 10−2 10

  dA dΩ = 2 . r

Luego, las que salen del agujero en la direcci´on θ y pegan en el a´rea de 1 cm2 colocada en θ = 45◦ y a una distancia de 10 cm son

7.2 Problemas

Rodrigo Ferrer

123

AN hwi cos θ AN hwi cos θ · dΩ = dΩ , 4πV 4V π h µg i cos θ = 13.28 × 103 · · 10−2 , h π h i 30 µg = . 3600 h

Luego, el tiempo total es t=

3600 = 120 s . 30

(7.23)

124

CAP´ ITULO 7

´ FORMULA DE MAXWELL

Parte III Potenciales Termodin´ amicos

Cap´ıtulo 8 Transformaciones de Legendre y Formulaciones Alternativas 8.1.

El Principio de M´ınima Energ´ıa

Hemos deducido directamente muchas consecuencias a partir de los postulados de la Termodin´amica. En este punto, es necesario usar formulaciones alternativas para simplificar los problemas. Se hace necesaria una teor´ıa general de transformaciones entre las representaciones equivalentes de un problema. Debemos saber elegir una formulaci´on te´orica conveniente para determinados tipos de problemas; esto es un arte. Dos representaciones equivalentes son, por ejemplo, la representaci´on entr´opica y la energ´etica. Ya hemos visto el principio de m´axima entrop´ıa para una energ´ıa total dada; pero existe un principio de m´ınima energ´ıa para una entrop´ıa dada: Principio de M´ axima Entrop´ıa :

En el equilibrio, el valor de cualquier par´ametro interno sin restricci´on es tal que maximiza la entrop´ıa total para el valor dado de la energ´ıa interna total.

Principio de M´ınima Energ´ıa

En el equilibrio, el valor de cualquier par´ametro interno sin restricci´on es tal que minimiza la energ´ıa para el valor dado de la entrop´ıa total.

:

Esta formulaci´on es equivalente al problema de maximizar el ´area de una figura, dado su per´ımetro; y cambiarlo por minimizar su per´ımetro, dada su a´rea. En ambos casos la soluci´on es un c´ırculo (circunferencia).

128

CAP´ ITULO 8

TRANSFORMACIONES DE LEGENDRE

Se hace necesaria un teor´ıa general de transformaciones entre las representaciones equivalentes.

Figura 8.1: Proceso termodin´amico a U = cte.

En la Figura 8.1, tenemos ◦ A y B tienen la misma entrop´ıa. B tiene menor energ´ıa que A. ◦ A y C tienen la misma energ´ıa. C tiene mayor entrop´ıa que B.

Seg´ un el principio de m´axima entrop´ıa, ten´ıamos 

∂S ∂X



 =0

;

U

∂ 2S ∂X 2

 <0 U

Definamos,  P ≡

∂U ∂X

 . S

Esto nos dice que P = P (U, X, S). Sabemos que 

Luego,

∂U ∂X

  S

∂X ∂S

  U

∂S ∂U

 = −1 X

8.1 El Principio de M´ınima Energ´ıa

Rodrigo Ferrer

 P =

∂U ∂X

= −

 , S

∂S ∂U

1   X



∂X ∂S

 , U



∂S ∂X U  , = − ∂S ∂U X   ∂S , = −T ∂X U = 0. Es decir, U tiene un extremo. Para clasificarlo, notemos que 

∂ 2U ∂X 2



 =

S

∂P ∂X

 . S

Entonces, como P = P (U, S, X),

 dP =

∂P ∂U





∂P ∂U

 





dU + S,X

∂P ∂X



 dX +

S,U

es decir, ∂P ∂X



∂ 2U ∂X 2





 =

S

X

∂U ∂X



 +

S

∂P ∂X

luego, 

En P = 0 tenemos

=P S

∂P ∂U

 + X

∂P ∂X

 . U

 , U

∂P ∂S

 dS , U,X

129

130

CAP´ ITULO 8



∂ 2U ∂X 2

TRANSFORMACIONES DE LEGENDRE



 =

S

∂P ∂X

 U

 

  ∂S  ∂  −  ∂X  U  =  ∂S ∂X  ∂U X U      ∂ ∂S ∂ 2S   ∂X ∂U X U ∂X 2 U ∂S  + = −  2 ∂S ∂X U ∂S ∂U X ∂U X 

 Luego, en

∂S ∂X

 = 0 tenemos U



∂ 2U ∂X 2



 = −T S

∂ 2S ∂X 2

 >0 U

y, entonces, U es un m´ınimo. Ejercicio: Demostrar el converso. Ilustraci´on:

V 2

V 1 S

j

V 2

V 1 S

k

V 1

V 2 S max

Figura 8.2: Sistema equilibr´andose. Independiente de cu´al sea el proceso, el estado de equilibrio final satisface ambas condiciones: M´axima entrop´ıa total y M´ınima energ´ıa total. Considere los sistemas cerrados 1 y 2 de la Figura 8.3. (1) (2) U = U (1) (S (1) , V (1) , N1 , . . .) + U (2) (S (2) , V (2) , N1 , . . .). Para un volumen constante, tenemos dU = T (1) dS (1) + T (2) dS (2)

8.2 Transformaciones de Legendre

Rodrigo Ferrer

131

2

1

Figura 8.3: Pared diat´ermica y fija.

Pero, en el equilibrio, dS = dS (1) + dS (2) = 0, luego

dU = (T (1) − T (2) ) dS (1) = 0 ⇒ T (1) = T (2)

8.2.

Transformaciones de Legendre

Los Par´ametros Extensivos son las variables independientes. Los Par´ametros Intensivos son los conceptos derivados. Es m´as f´acil medir un par´ametro intensivo (T, P, µ, . . .) que uno extensivo (U, V, N, . . .), por ejemplo, es posible medir la temperatura y no la entrop´ıa. Para hacerlo, notamos que es posible invertir la situaci´on y tomar los par´ametros intensivos como las variables independientes. Consideremos la ecuaci´on fundamental U = U (S, V, N ), donde S, V y N son los par´ametros extensivos.

Conceptos derivados

   ∂U   T =      ∂S V,N   ∂U P = −    ∂V S,N   ∂U     µ = ∂N S,V

Queremos a T, P y µ como nuestras variables independientes. La t´ecnica matem´atica de las Transformaciones de Legendre permite hacerlo: Para ejemplificar, tomemos el caso de una dimensi´on, en que y = y(x), con x la variable independiente.

132

CAP´ ITULO 8

TRANSFORMACIONES DE LEGENDRE y

x

Figura 8.4: Curva de una funci´on y : R → R.

Sea y(x) = x2 . Luego P = 2x. Por lo tanto P2 . y(P ) = 4 Ya tenemos a y en funci´on de los par´ametros intensivos. Pero hay muchas curvas y(x) que tienen P2 pendiente , por lo que hemos perdido informaci´on, ya que 4 dy = 2x dx



y(x) = x2 + cte .

y

x

Figura 8.5: Curvas y(x) = x2 + cte. Ahora bien, una curva puede ser graficada de dos maneras: a) Asociando una imagen y(x) a cada x, como es usual. b) Como la envolvente de una familia de tangentes (Pluecker, Figura 8.6) Definimos ψ(P ) el punto de intersecci´on de la pendiente con la ordenada (el eje y). Si y(x) es una ecuaci´on fundamental, tambi´en lo ser´a ψ(P ), ya que estas relaciones son matem´aticamente

8.2 Transformaciones de Legendre

Rodrigo Ferrer

133

y

x

Figura 8.6: Envolventes de una curva.

equivalentes. Ejercicio urgente: Graficar las pendientes P en los puntos ψ = −P 2 y luego graficar y(x) = 14 x2 . ¿C´omo obtener ψ(P ) a partir de y(x)? Debemos considerar el punto de intersecci´on con el eje y: P =

y−ψ x−0

Luego, ψ = y − Px. Dada y(x) tenemos:

P =

∂Y ≡ P (X) , ∂X

entonces x = x(P ). As´ı, Y (X(P )) = Y (P ) . Luego, ψ = Y (P ) − P X(P ) = ψ(P ) .

Retomemos el ejemplo y(x) = x2 .

(8.1)

134

CAP´ ITULO 8

TRANSFORMACIONES DE LEGENDRE

∂y = 2x , ∂x P x(P ) = , 2 P (x) =

⇒ ⇒

y(P ) =

P2 , 4



ψ(P ) =

P2 P P4 −P =− . 4 2 4 2

Luego, la Transformada de Legendre de y(x) = x2 es ψ(P ) = − P4 . Ejercicios 1. y(x) = sin(ωx) 2. y(x) = ex 3. y(x) = ax

8.2.1.

(ojo con la estabilidad)

Transformaci´ on de Legendre Inversa

Por definici´on, ψ = y − Px. Tomando su diferencial, dψ = dy − P dx − x dP , = P dx − P dx − x dP , = −x dP .

Por lo tanto, ⇒x=−

dψ . dP

Con esto recuperamos P en funci´on de x, ya que de la u ´ltima ecuaci´on, x = x(P )



P = P (x) .

Luego, y = ψ(P (x)) + xP (x) = y(x) .

(8.2)

8.2 Transformaciones de Legendre

Rodrigo Ferrer

Ejemplifiquemos usando el mismo problema anterior. Ten´ıamos ψ = −

x=− ⇒

P dψ = , dP 2

P = 2x ,  1 4x2 = −x2 , 4



ψ(x) = −



y(x) = −x2 + x(2x) = x2 .

es decir, la transformaci´on inversa de ψ = −

P2 es y(x) = x2 . 4

P2 : 4

135

136

CAP´ ITULO 8

TRANSFORMACIONES DE LEGENDRE

Cap´ıtulo 9 Potenciales Termodin´ amicos 9.1.

Potencial de Helmholtz

Figura 9.1: Hermann von Helmholtz. 1821-1894

Tomemos la ecuaci´on fundamental U (S, V, N1 , . . . , Nr ). Se define el Potencial de Helmholtz como F ≡ U [T ] = U − T S

(9.1)

Hacemos de T nuestra variable independiente, en vez de S, mediante la transformaci´on de Legendre. Como transformaci´on de Legendre, adoptamos la notaci´on F = U [T ].

Transformaci´ on de Legendre: U (S, V, N1 , . . . , Nr ) 

∂U T = ∂S

⇒ ∴

Luego F es funci´on de T .

T = T (S) ⇒ U = U (T )



S = S(T )

F = U (S(T )) − T S(T ) = F [T ]

138

CAP´ ITULO 9

´ POTENCIALES TERMODINAMICOS

Transformaci´ on de Legendre Inversa: F (T, V, N1 , . . . , Nr ). ∂F ⇒ S = S(T ) ⇒ T = T (S) ∂T U = F (T (S)) + T (S)S = U (S, V, N1 , . . . , Nr )

S=− ∴

9.2.

Potencial Entalp´ıa

Tomemos nuevamente la ecuaci´on fundamental U (S, V, N1 , . . . , Nr ). Se define la Entalp´ıa como H ≡ U + PV

(9.2)

La presi´on P reemplaza al volumen V , convirti´endose en la variable independiente mediante la transformaci´on de Legendre. Como transformaci´on de Legendre, H = U [P ].

Transformaci´ on de Legendre: U (S, V, N1 , . . . , Nr ) ∂U P =− ∂V ∴

 ⇒

P = P (V ) U = U (P )



V = V (P )

H = U (P ) + P V (P ) = H[P ]

Luego H es funci´on de P . Transformaci´ on de Legendre Inversa: H(S, P, N1 , . . . , Nr ) ∂H ⇒ V = V (P ) ⇒ P = P (V ) ∂P U = H(V ) − P (V )V = U (S, V, N1 , . . . , Nr )

V = ∴

9.3.

Potencial de Gibbs

Tomemos nuevamente la ecuaci´on fundamental U (S, V, N1 , . . . , Nr ). Se define el Potencial de Gibbs como G ≡ U − TS + PV

(9.3)

Reemplazamos S y V por T y V simult´aneamente, convirti´endose estas u ´ltimas en las variable independientes mediante la transformaci´on de Legendre. Como transformaci´on de Legendre, G = U [T, P ].

9.3 Potencial de Gibbs

Rodrigo Ferrer

139

Figura 9.2: Josiah Willard Gibbs. 1839-1903

Transformaci´ on de Legendre: U (S, V, N1 , . . . , Nr )  ∂U   P =−    ∂V ∂U T = ∂S

    





P = P (V ) U = U (P )



T = T (S) U = U (T )









V = V (P )

S = S(T )

G = U (T, P ) − T S(T ) + P V (P ) = G[T, P ]

Luego G es funci´on de P y T . Transformaci´ on de Legendre Inversa: G(T, P, N1 , . . . , Nr )  ∂G   V =   ∂P   ∂G   S=− ∂T ∴

⇒ ⇒

V = V (P ) S = S(T )

⇒ ⇒

P = P (V ) T = T (S)

U = G + T (S)S − P (V )V = U (S, V, N1 , . . . , Nr )

Ejercicio: Hacer este mismo procedimiento para el Potencial Gran Can´onico: U [T, µ] = U − T S − µN

(9.4)

140

CAP´ ITULO 9

´ POTENCIALES TERMODINAMICOS

9.4.

Funciones Generalizadas de Massieu

Fueron inventadas por Massieu en el a˜ no 1869. Actualmente son predadoras de las introducidas por Gibbs en 1875. Seg´ un Gibbs, tenemos   U [T ] = U − T S ≡ Energ´ıa libre de Helmholtz U [P ] = U + P V ≡ Entalp´ıa U (S, V, N1 , . . . , Nr )  U [T, P ] = H − T S ≡ Energ´ıa libre de Gibbs La propuesta de Massieu es

S(U, V, N1 , . . . , Nr )

                

  1 1 F S = S− U = − T T T   P P S = S− V T T   1 P P G 1 S , = − = S− U− T T T T T

Estas transformaciones son de Legendre. Comprobemos para el caso S

P  T

= S − PT V :

Tenemos la ecuaci´on fundamental S(U, V, N1 , . . . , Nr ). Transformaci´ on de Legendre: ∂S P = T ∂V ∴

 ⇒

P T

= PT (V ) S = S PT



V =V

P T



      P P P P S − ·V =S T T T T

P . T

Luego S es funci´on de

Transformaci´ on de Legendre Inversa: M (S, PT , N1 , . . . , Nr )   ∂M P P P  ⇒ V =V ⇒ = (V ) V = P T T T ∂ T   P P ∴ S=S (V ) + (V ) · V = S(U, V, N1 , . . . , Nr ) T T



9.5 Relaciones de Maxwell

Rodrigo Ferrer

9.5.

Relaciones de Maxwell U H ≡ U + PV F ≡ U − TS G ≡ U + PV − TS = H − TS

Energ´ıa Interna Entalp´ıa Energ´ıa Libre de Helmholtz Energ´ıa Libre de Gibbs

Para un sistema monocomponente, tenemos

U

= U (S, V, N )       ∂U ∂U ∂U ⇒ dU = dS + dV + dN ∂S V,N ∂V S,N ∂N S,V ⇒ dU = T dS − P dV + µ dN H ⇒ dH ⇒ dH ⇒ dH

= = = =

U + PV dU + P dV + V dP T dS − P dV + µ dN + P dV + V dP T dS + V dP + µ dN

F ⇒ dF ⇒ dF ⇒ dF

= = = =

U − TS dU − T dS − S dT T dS − P dV + µ dN − T dS + S dT S dT − P dV + µ dN

G ⇒ dG ⇒ dG ⇒ dG

= = = =

H − TS dH − T dS − S dT T dS + V dP + µ dN − T dS − S dT V dP − S dT + µ dN

Luego U = U (S, V, N ), H = H(S, P, N ), F = F (V, T, N ), G = G(P, T, N ). Puesto que dU, dH, dF, y dG son diferenciales exactos, se tiene

141

CAP´ ITULO 9

142

dU : 





dH : 





´ POTENCIALES TERMODINAMICOS

∂T ∂V





∂T ∂N



∂P ∂N



∂T ∂P



∂T ∂N



∂V ∂N



= − S,N

 =

S,V

 = −

S,V

 =

S,N

 = S,P

 = S,P

dF : 

∂P ∂S



∂µ ∂S



∂µ ∂V



V,N



V,N



S,N

dG : 

∂V ∂S



∂µ ∂S



∂µ ∂P



P,N



P,N



S,N

∂S ∂V





∂S ∂N



∂P ∂N



∂S ∂P



∂S ∂N



∂V ∂N



= T,N

 = −

T,V

 = −

T,V

 = −

T,N

 = −

T,P

 =

T,P

Tabla 9.1: Relaciones entre las razones de cambio.

9.6.

Problemas

1. Calcular Cv (T, P ) sabiendo que 2

H = AS N

−1

 ln

P Po



Soluci´ on Sabemos que T Cv = N



∂S ∂T



Luego, como V =

∂H , ∂P

. V

.

∂P ∂T



∂µ ∂T



∂µ ∂V



∂V ∂T



∂µ ∂T



∂µ ∂P



V,N

V,N

T,N

P,N

P,N

T,N

9.6 Problemas

Rodrigo Ferrer entonces, V = ⇒

PV =

AS 2 1 , N P AS 2 . N

Luego, U = H − PV   AS 2 P = ln − PV , N Po   AS 2 P AS 2 = ln − , N Po N     AS 2 P = ln −1 , N Po     AS 2 AS 2 = ln −1 . N N V Po Por lo tanto,  T =

∂U ∂S

 , N

       2AS AS 2 AS 2 N V Po 2AS = ln −1 + , N N V Po N AS 2 N V Po   2AS AS 2 = ln . N N V Po Finalmente, T Cv = N =

=

=



∂S ∂T

 , V

T 1   , N ∂T ∂S V 1 T  , P 2AS 2 N 2A ln + · N Po N S T  . P 2A ln +2 Po

143

144

CAP´ ITULO 9

´ POTENCIALES TERMODINAMICOS

Cap´ıtulo 10 El Principio Extremal en las Representaciones Transformadas de Legendre 10.1.

Principio de M´ınima Energ´ıa

Consideremos un sistema compuesto de dos componentes, sin p´erdida de generalidad, en contacto t´ermico con una fuente de calor a temperatura T (r) . En el equilibrio, la energ´ıa interna total es m´ınima, es decir: d(U + U (r) ) = 0

(10.1)

Por otro lado, d2 (U + U (r) ) = d2 U + d2 U (r) X ∂  ∂U  2 dXi dXj = dU+ ∂X ∂X j j i,j ⇒

d2 U

> 0

(10.2)

d(S + S (r) ) = 0

(10.3)

Adem´as la entrop´ıa es constante. Luego

Otras condiciones de clausura dependen del proceso en el sistema compuesto: i) Paredes m´oviles e impermeables: (1)

dNj

(2)

= dNj

= d(V (1) + V (2) ) = 0

∀ j

(10.4)

146

CAP´ ITULO 10

EL PRINCIPIO EXTREMAL

ii) Paredes r´ıgidas y permeables al componente k: (1)

(2)

(1)

d(Nk + Nk ) = dNj

(2)

= dNj

= dV (1) = dV (2) = 0

∀ j 6= k

(10.5)

Estas condiciones son suficientes para determinar el estado de equilibrio. Ahora bien, en contacto t´ermico hay intercambio de calor ąQ = T dS entre los subsistemas, y entre ellos y la fuente. Pueden tambi´en haber t´erminos que involucren intercambio de trabajo. Tenemos, pues, por conservaci´on de la energ´ıa: d(U + U (r) ) = − + =

T (1) dS (1) + T (2) dS (2) + T (r) dS (r) P dV (1) − P dV (2) (1) (1) (2) (2) µk dNk + µk dNk 0

Por independencia de la entrop´ıa, el volumen y el n´ umero de mol´eculas, tenemos: T (1) dS (1) + T (2) dS (2) + T (r) dS (r) = 0 y por (10.3), T (1) dS (1) + T (2) dS (2) − T (r) d(S (1) + S (2) ) = 0 ⇒ (T (1) − T (r) ) dS (1) + (T (2) − T (r) ) dS (2) =0 ⇒ T (1) = T (r) = T (2) Las otras condiciones de clausura dan condiciones dependiendo de su naturaleza. Tenemos d(U + U (r) ) = dU + T (r) dS (r) = 0 ⇐⇒ dU − T (r) dS = 0 Si T (r) es constante, tenemos   

d(U + T (r) S) = 0 y d2 U = d2 (U − T (r) S) = 0

es decir, la cantidad U − T (r) S es m´ınima en el estado de equilibrio. Si aceptamos que en el equilibrio los sistemas tienen T = T (r) , debemos buscar entre ellos los estados que tengan esta temperatura, es decir, dF = d(U − T S) = 0 con la condici´on T = T (r) .

Rodrigo Ferrer

10.2.

10.2 Principio Minimal Para la Energ´ıa Libre de Helmholtz

147

Principio Minimal Para la Energ´ıa Libre de Helmholtz

El valor en el equilibrio de cualquier par´ ametro interno no restringido en un sistema en contacto diat´ ermico con una fuente de calor, minimiza el potencial de Helmholtz entre los estados para los cuales T = T (r) . Nota: N´otese que se minimiza de todas maneras la energ´ıa total U + U (r) . En efecto, d(U − T S) = = = = =

dU − d(T S) dU − T (r) dS dU + dU (r) d(U + U (r) ) 0

Ilustraci´on: T = T (r) constante. U = U (S, V, N ).   ∂U = T (r) . T = ∂S V,N Si tomamos F = U − T S = F (T, V, N ) = F (T (r) , V, N ) y minimizamos, las cosas se simplifican. Ejemplo: Analicemos el caso del t´ıpico pist´on m´ovil y diat´ermico movi´endose en un cilindro con gas. Luego dF = dU − T dS − S dT = T dS − P dV − T dS − S dT + µ dN = −P dV − S dT + µ dN Pero para T y N constantes, tenemos dF = −P dV Luego dF

= −P (1) dV (1) − P (2) dV (2) = −P (1) (T (r) , V (1) , N (1) ) dV (1) − P (2) (T (r) , V (2) , N (2) ) dV (2) = 0

⇒ P (1) (T (r) , V (1) , N (1) ) = P (2) (T (r) , V (2) , N (2) ) con V = V (1) + V (2) constante.

(10.6)

CAP´ ITULO 10

148

EL PRINCIPIO EXTREMAL

Figura 10.1: Sistema termodin´amico unido a una fuente reversible de trabajo (RWS).

Considere el siguiente sistema: Tenemos

ąWrws = = = = =

−dU − dU (r) −dU − T (r) dS (r) −dU + T (r) dS −d(U − T (r) S) −dF

El trabajo entregado por un sistema en contacto t´ermico con una fuente a temperatura T (r) en un proceso reversible, es igual al decrecimiento de F , la energ´ıa potencial de Helmholtz. En el caso del gas ideal, tendremos, reemplazando de (2.2) en (2.1),

N S(U, V, N ) = So + N R ln No

"

U Uo

 32 

V Vo



N No

− 52 #

N So + N R ln ⇒ S(T, V, N ) = No

"

T To

 32 

V Vo



N No

− 52 #



F

= U − TS "  3     5 # −2 3 N T 2 V N = N RT − T So + N RT ln 2 No To Vo No " "  3     5 ## −2 Fo T 2 V N = N RT − ln No RTo To Vo No

El decrecimiento de esta energ´ıa ser´a el trabajo entregado por el sistema.

10.3.

Principio de Entalp´ıa M´ınima

Tomemos una fuente de presi´on (reservoir ) conectado a un sistema compuesto de energ´ıa interna U , cuyas paredes son adiab´aticas y m´oviles. En este sistema, tendremos

10.3 Principio de Entalp´ıa M´ınima

Rodrigo Ferrer

d(U + U (r) ) = = = = =

149

dU + dU (r) dU − P (r) dV (r) dU + P (r) dV d(U + P (r) V ) 0

Aceptando que P = P (r) , se tiene d(U + P V ) = 0 Adem´as d2 (U + P (r) V ) = d2 U > 0 es decir tenemos el principio minimal para la Entalp´ıa En el equilibrio, el valor de cualquier par´ ametro no restringido de un sistema en contacto con una fuente de presi´ on, minimiza la entalp´ıa sobre todos los estados, a presi´ on constante P (r) . Para la entalp´ıa tenemos

H = U + PV dH = dU + P dV + V dP = T dS − P dV + µ1 dN1 + µ2 dN2 + · · · + µr dNr + P dV + V dP = T dS + V dP + µ1 dN1 + µ2 dN2 + · · · + µr dNr Para un sistema en contacto con una fuente de presi´on y encerrado en paredes impermeables se tiene dH = T dS = ąQ (proceso isob´arico) es decir, el calor agregado a este sistema es el aumento en su entalp´ıa, siempre que se mantengan constantes todos los par´ametros, a excepci´on de S y V . Recordemos que para N y V constantes (proceso isoc´orico):

dU = T dS − P dV +

X

µi dNi

i

= ąQ es decir, Z Qi→f =

f

Z

dH = Hf − Hi

ąQ = i

f

i

150

CAP´ ITULO 10

EL PRINCIPIO EXTREMAL

P

GAS

r

Pistón sin roce Resistencia

Figura 10.2: Cilindro con pist´on m´ovil y resistencia el´ectrica en contacto con una fuente de presi´on.

Consideremos el sistema de un cilindro adiab´atico provisto de un pist´on, que se desplaza contra una fuente de presi´on P r , y de una resistencia que le otorga calor. Al encender la resistencia el gas se calienta y se expande contra la presi´on P r , realizando un trabajo. Entalp´ıa inicial Hi = Ui + P r Vi Entalp´ıa final Hf = Uf + P r Vf Luego, ∆H = Hf − Hi = Uf − Ui + P r · (Vf − Vi ) = ∆U − ∆W = ∆Q, Es decir, en este proceso, el cambio de Entalp´ıa es igual al calor otorgado por la resistencia.

10.4.

El Proceso de Joule Thomson: “Throttling Process” o “Proceso de Estrangulamiento”

Tomemos un sistema monocomponente: ∂H ⇒ H = H(S, P, N ) ⇒ V = ∂P



V = V (S, P, N ) ⇒ S = S(V, P, N ) H = H(V, P, N )

Qi→f = H(Vf , P, N ) − H(Vi , P, N )

Figura 10.3: Proceso de “estrangulamiento” de un gas.

10.4 El Proceso de Joule Thomson

Rodrigo Ferrer

151

En el proceso de estrangulamiento, la entalp´ıa inicial Hi es igual a la entalp´ıa final Hf . Consideremos un gas sometido a estrangulamiento. ◦ Energ´ıa interna molar inicial: ui . ◦ Energ´ıa interna final: uf . ◦ Trabajo hecho por el pist´on izquierdo: Pi vi > 0. ◦ Trabajo hecho por el pist´on derecho: −P f ivf < 0. Por conservaci´on de la energ´ıa, uf = ui + Pi vi − Pf vf . Luego | {z } Trabajo Neto

uf + Pf vf = ui + Pi vi ⇒ hf = hi

Durante el proceso no se sabe nada de la entalp´ıa: los estados no son de equilibrio termodin´amico.

Figura 10.4: Isentalp´ıas (s´olido), temperaturas de inversi´on (oscuro), y curva de coexistencia para el nitr´ogeno; semi-cuantitativa.

152

CAP´ ITULO 10

EL PRINCIPIO EXTREMAL

Considere, en un proceso de estrangulamiento, que las diferencias de presi´on son muy peque˜ nas:      ∂S ∂H   = T +V   ∂P ∂P T T dH = T dS + V dP ⇒      ∂H ∂S   = T  ∂T P ∂T P  dT =

∂T ∂P

= − 

 dP H,N

∂P ∂H

1   T

∂H ∂T

 P



∂H ∂P T  = − ∂H ∂T P   ∂S T +V ∂P T   = − ∂S T ∂T P   ∂V −T +V ∂T P = − N cp v (T α − 1) dP cp

=

donde dP < 0. Luego sgn dT = sgn (T α − 1). Pero para el gas ideal, α = concu´ımos que no hay cambio en T .

1 , T

por lo que

Notemos que existe una temperatura despu´es de la cual el proceso es de enfriamiento, es la llamada temperatura de inversi´on, definida por αTinversi´on = 1 Para usar este efecto para enfriar un gas debemos realizar un proceso de pre-enfriamiento. Ejemplo: Gas de Van Der Waals Este gas est´a caracterizado por la ecuaci´on P = Luego

RT a − 2 v−b v

(10.7)

10.4 El Proceso de Joule Thomson

Rodrigo Ferrer

153

  RT R 2a dT + − dv dP = + v−b (v − b)2 v 3 A presi´on constante, tenemos 



∂v ∂T



= P

R · v−b

1 = v 1 = v

α



∂v ∂T

1 RT 2a − 3 2 (v − b) v  P

R RT 2a(v − b) − (v − b) v3 1 = 2a(v − b) vT − v−b Rv 2 −1  vT 2a(v − b) = − v−b Rv 2 Queremos T tal que T α = 1. Definimos 1 ≡

b v

2 ≡

a RT v

(10.8)

Luego −1 T 2T (v − b)2 α = − 1 − 1 v  −1 1 1 = − 2(1 − 1 )2 T 1 − 1  −1 1 Tinv α = − 2(1 − 1 )2 = 1 1 − 1 





1

=

⇒ ⇒

1 1 b v

= =



=

⇒ Tinv =

1 − 2(1 − 1 )2 1 − 1 1 − 1 + 22 − 21 2 22 2a RTinv v 2a bR

(1 2 → 0)

154

CAP´ ITULO 10

EL PRINCIPIO EXTREMAL

Tinv (H2 ) = 224 K Tinv (N2 ) = 850 K Tinv (CO2 ) = 2260 K

Tinv (Ne ) = 302 K Tinv (O2 ) = 1020 K

Cap´ıtulo 11 El Cuadrado Termodin´ amico Max Born 1929 Valid Facts and Theoretical Understanding Generates Solutions to Hard Problems V

F

U

T

G

S

H

P

Figura 11.1: El cuadrado termodin´amico.

Note que cada potencial termodin´amico tiene a sus flancos sus variables naturales. La calcular sus diferenciales debemos ver la direcci´on de las flechas; si salen de la variable el signo es positivo, de lo contrario es negativo. As´ı tenemos:

dU dF dG dH

= = = =

T dS − P dV −S dT − P dV −S dT + V dP V dP + T dS

Empiece tomando una esquina. La derivada parcial de esta variable respecto a la siguiente, situada en el mismo lado del cuadrado, ser´a igual a la derivada parcial obtenida haciendo el mismo proceso en el lado paralelo del cuadrado, en la misma direcci´on, escribiendo un signo menos si el cuadrado es anti-sim´etrico respecto a las flechas. Dentro de lo que vimos en la Tabla 9.1, ten´ıamos las relaciones de Maxwell:

156

CAP´ ITULO 11

´ EL CUADRADO TERMODINAMICO

     ∂V ∂T   = Relaciones  ∂S  P  ∂P S de Max-  ∂P ∂S  =  well ∂T V ∂V T



   ∂S ∂V = −  ∂P T  ∂T P ∂T ∂P = − ∂V S ∂S V

; ;

Considere ąQ = T dS . Por lo tanto dS ąQ =T si T es constante dV dV     ∂S ∂P ąQ = T dV = dV ∂V T ∂T V

Luego

Considere α = T1 . Luego 1 V

11.1.



∂V ∂T

 = P

1 T

Procedimiento Para la Reducci´ on de las Derivadas en Sistemas Monocomponentes

Considere z = z(x, y). Luego x = x(y, z) e y = y(x, z). Por lo tanto     ∂x ∂x dx = dy + dz  ∂z y  ∂y z ∂y ∂y dx + dz dy =  ∂x z  ∂z x ∂z ∂z dz = dx + dy ∂x y ∂y x Se cumple que 

∂x ∂y

 z

1  ∂y ∂x  z ∂x ∂w =  z ∂y ∂w z   ∂z ∂y = −  x ∂z ∂x y = 

(11.1)

(11.2)

(11.3)

11.1 Reducci´on de Derivadas

Rodrigo Ferrer

157

Hay s´olo tres independientes. Por convenci´on se eligen, para N constante: α=

1 ∂V ( )P V ∂T

, 1 ∂V κT = − ( )T V ∂P CP = (

∂U )P + P V α ∂T

CV = (

∂U )V ∂T

. Recuerde que cv = cp −

T vα2 κT

La elecci´on de α, κT y Cp corresponde a una transformaci´on impl´ıcita en la representaci´on de Gibbs. En efecto:

g = u − Ts + Pv ⇒ dg = −s dT + v dP   ∂g , ⇒ s = − ∂T P 1 cp = N



1 α= v



ąQ ∂T



∂v ∂T



P

 v=

∂g ∂P

 T

   T ąQ ∂s = dT ∂T  2  P ∂ g ⇒ T cp = − ∂T 2 P 1 ⇒ T cp = N



Adem´as,  ⇒

vα =

P

∂v ∂T



 =−

P

Luego  vα =

∂ ∂P



∂g ∂T

 

 =

P

T

∂ 2g ∂P ∂T



∂s ∂P

 T

CAP´ ITULO 11

158

´ EL CUADRADO TERMODINAMICO

y, finalmente,

κT ⇒ vκT

      1 ∂v 1 ∂ ∂g = − =− v ∂P T v ∂P ∂P T T  2  ∂ g = − ∂P 2 T

Conclusi´on: Si N es constante, todas las primeras derivadas, involucrando tanto par´ametros extensivos como intensivos, pueden ser escritas en t´erminos de las segundas derivadas del Potencial de Gibbs, de las cuales hemos demostrado que cp , α, y κT constituyen un conjunto completo independiente. Receta Para Reducir Derivadas 1. Si la derivada contiene cualquier potencial, ll´evelos uno a uno al numerador y elim´ınelo usando el cuadrado termodin´amico. 2. Si la derivada contiene el potencial qu´ımico, ll´evelo al numerador y eliminelo por medio de la relaci´on de Gibbs-Duhem, dµ = −s dT + v dP . 3. Si la derivada contiene a la entrop´ıa, ll´evela al numerador y trate de eliminarla con una de las cuatro relaciones de Maxwell. Si las relaciones de Maxwell no la eliminan, ponga un ∂T debajo del ∂S y use       ∂X ∂Y ∂X = ∂Y Z ∂W Z ∂W Z En este caso, el numerador podr´a ser expresado como uno de los calores espec´ıficos. 4. Lleve el volumen al numerador. La derivada restante ser´a expresable en t´erminos de α y κT . 5. La derivada dada originalmente ha sido expresada ahora en t´erminos de las cantidades α, κT , cp y cv . El calor espec´ıfico a volumen constante puede ser eliminado por la ecuaci´on T vα2 cv = cp − κT

11.2.

Aplicaciones Simples

11.2.1.

Compresi´ on Adiab´ atica

Considerar

11.2 Aplicaciones Simples

Rodrigo Ferrer

159

Figura 11.2: Compresi´on Adiab´atica de un Gas. Queremos conocer los cambios en los par´ametros termodin´amicos del sistema. Si conocemos la ecuaci´on fundamental, S(U, V, N ) ⇒ U = (S, V, N )   ∂U T = ⇒ T = T (S, V, N ) ∂S V,N   ∂U ⇒ P = P (S, V, N ) P = − ∂V S,N S =

Inicialmente T = Ti y P = Pi . Luego Ti = T (Si , Vi , N ) Pi = P (Si , Vi , N ) por lo que podemos conocer Si y Vi de estas dos ecuaciones. Ahora bien, en el proceso adiab´atico y quasi-est´atico, dS = 0, es decir, la entrop´ıa se mantiene constante. Entonces, considerando T = T (S, V, N ) P = P (S, V, N ) tenemos que V = V (S, P, N ) ⇒ T = T (S, V (S, P, N ), N ) ⇒ T = T (S, P, N ) Por lo tanto, si se conoce la ecuaci´on fundamental, ∆T = T (S, Pf , N ) − T (S, Pi , N ) Si no se conoce la ecuaci´on fundamental, pero se conocen cp , α y κT , podemos escribir

160

CAP´ ITULO 11

´ EL CUADRADO TERMODINAMICO

 dT =

∂T ∂P

 dP

(11.4)

S,N

Llevemos S al numerador: 

∂T ∂P

 







∂T ∂P

S

S

∂P ∂S

  T

∂S ∂T

= −

=

=

= =

 = −1 P

1  

∂S ∂P ∂S T ∂T   ∂S ∂P −  T ∂S ∂T P   ∂V − ∂T −  P T ∂S T ∂T P αV 1 C T p T αV Cp

 P

Esta relaci´on se puede tambi´en obtener a partir del cuadrado termodin´amico, donde:     ∂T ∂V = ∂P S ∂S P ∴

dT =

T vα dP cp

Consideremos ahora, para un peque˜ no cambio de la presi´on,   ∂µ dµ = dP ∂P S,N Como aparece el potencial qu´ımico, usamos la relaci´on de Gibbs-Duhem:

 ⇒

dµ = −s dT + v dP    ∂µ ∂T = −s +v ∂P S,N ∂P S,N

(11.5)

11.2 Aplicaciones Simples

Rodrigo Ferrer

161

Ahora, llevamos la entrop´ıa al numerador:   ∂S  ∂P T,N    dµ = v + s    dP ∂S   ∂T P,N     ∂V  ∂T P,N      = v − s  dP ∂S   ∂T P,N   sαV T = v− dP Cp   sT vα dP = v− cp   sT vα dµ = v − dP cp 

11.2.2.



(11.6)

Compresi´ on Isot´ ermica

Nos podemos preguntar varias cosas: i) Cantidad de calor que hay que extraer a temperatura constante si conocemos la ecuaci´on fundamental: ∆Q = T ∆S = T S(T, Pf , N ) − T S(T, Pi , N ) Como  S = S(U, V, N )          ∂U P = −    ∂V  S,N   ∂U    T = ∂S V,N

  U = U (S, V, N ) P = P (S, V, N ) ⇒  T = T (S, V, N )  ⇒

S = S(P, V, N ) S = S(T, V, N )

CAP´ ITULO 11

162

´ EL CUADRADO TERMODINAMICO

⇒ S = S(T, P, N ) ii) Si no conocemos la ecuaci´on fundamental,

ąQ = T dS   ∂S dP = T ∂P T,N   ∂V dP = −T ∂T P,N = −T V α dP



ąQ = −T V α dP

(11.7)

En cuanto al cambio en la energ´ıa interna, tenemos  dU =

∂U ∂P

 dP T,N

= TdS −P dV   dU  ∂U ∂S ∂V ⇒ =T −P ∂P T,N ∂P T,N ∂P T,N   ∂V = −T + P V κT ∂T P,N = −T V α + P V κT



11.2.3.

dU = (−T V α + P V κT ) dP

Expansi´ on Libre

Figura 11.3: Gas expandi´endose libremente

(11.8)

11.2 Aplicaciones Simples

Rodrigo Ferrer

163

Soltar el sistema implica un cambio en la entrop´ıa. No hay trabajo. No hay intercambio de energ´ıa interna: ∆U = ∆Q + ∆W = 0 Si conocemos la ecuaci´on fundamental,

S ⇒ U ⇒ T ⇒ T

= = = =

S(U, V, N ) U (S, V, N ) T (S, V, N ) T (U, V, N )

∆T = Tf − Ti = T (U, Vf , N ) − T (U, Vi , N ) Para un proceso infinitesimal,  dT =

∂T ∂V

 dV U,N

Debemos poner U en el numerador:

dU = T dS − P dV De esta ecuaci´on obtenemos que 





∂U ∂T

=T V,N

∂S ∂T

 = Cv V,N

y, adem´as, 

∂U ∂V



 = T

T,N



=

=

= =

∂S ∂V

 −P T,N

 ∂P T −P ∂T V,N   ∂V ∂T P,N  −T  −P ∂V ∂P T,N TV α −P V κT Tα −P κT

164

CAP´ ITULO 11

´ EL CUADRADO TERMODINAMICO

 ∂U Tα P− ∂V T,N κT  ∴ dT = −  dV = ∂U Cv ∂T V,N 

11.2.4.

7.4-8, H. CALLEN

Demuestre que 

∂cp ∂P

 T

    ∂α 2 = −T v α + ∂T P

De (3.1), tenemos T cp = N



∂S ∂T



 =T

P

∂s ∂T

 P

Luego 

∂cp ∂P





= T

=

=

=

=

=

=

   T ∂S N ∂T P T    T ∂ ∂S N ∂P ∂T P T     T ∂ ∂S N ∂T ∂P T P     ∂ ∂V T − N ∂T ∂T P P   T ∂ 2V − N ∂T 2 P   2   T 1 ∂ V − V N V ∂T 2 P   2   T 1 ∂ V 2 2 − V −α +α N V ∂T 2 P ∂ ∂P 



(11.9)

11.2 Aplicaciones Simples

Rodrigo Ferrer

T = − V N

"

T = − V N

"

T = − V N



=

=

=



1 V



1 V



∂ 2V ∂T 2



∂ 2V ∂T 2



∂ 2V ∂T 2



P

1 − 2 V



− V −2

∂V ∂T



P

−V P

−2



2

# + α2

P

#

∂V ∂T

2

∂V ∂T

 

+ α2

P

P

∂V ∂T

 +α

2



P

    ∂(V −1 ) ∂V 2 + +α ∂T ∂T P P P         2  T ∂ 1 ∂V 1 ∂ V 2 − V + +α N V ∂T 2 P ∂T V ∂T P P           T 1 ∂ 1 ∂V ∂ ∂V 2 − V + +α N ∂T V ∂T P V ∂T ∂T P P P       T ∂ 1 ∂V 2 − V +α N ∂T V ∂T P P    ∂α 2 +α −T v . ∂T P

T = − V N =

1 V



1 V



∂ 2V ∂T 2





165

166

CAP´ ITULO 11

´ EL CUADRADO TERMODINAMICO

Cap´ıtulo 12 Estabilidad en los Sistemas Termodin´ amicos El principio b´asico de la termodin´amica implica que dS = 0 (la entrop´ıa es un extremo) y que d2 S < 0 (este extremo es un m´aximo). Esta u ´ltima condici´on determina la estabilidad de los estados de equilibrio predichos. En la mec´anica cl´asica el estado de equilibrio de un p´endulo r´ıgido est´a en la posici´on en la cual la energ´ıa potencial es m´ınima; pero hay adem´as un est´ado de “equilibrio inestable” en el punto opuesto, donde la energ´ıa potencial es m´axima. Luego, es l´ogico preguntarnos en este punto ¿bajo qu´e condiciones un sistema termodin´amico es estable?. Para contestar esta pregunta, considere dos subsistemas id´enticos, cada uno con ecuaci´on fundamental S = S(U, V, N ), separados por una pared totalmente restrictiva (cerrados). Suponga que la dependencia de S con respecto a U est´a dada por la curva de la Figura 12.1.

Figura 12.1: Para una relaci´on fundamental de curva convexa, la entrop´ıa se aumenta a trav´es de una transferencia de energ´ıa entre los dos subsistemas. Tal sistema es inestable.

Si la pared que separa a los subsistemas se hace diat´ermica, habr´a un flujo de energ´ıa interna, el cual ser´a espont´aneo, en busca de un estado con mayor entrop´ıa. M´as a´ un, entre las porciones de cada subsistema habr´a una transferencia de energ´ıa, desarroll´andose inhomogeneidades internas. Esto se traducir´a en una transici´on de fase.

168

CAP´ ITULO 12

´ ESTABILIDAD EN LOS SISTEMAS TERMODINAMICOS

La entrop´ıa total cambiar´a de su valor inicial 2S(U, V, N ) a S(U + ∆U, V, N ) + S(U − ∆U, V, N ). S(U + ∆U, V, N ) + S(U − ∆U, V, N ) > 2S(U, V, N ) Considere ahora que la dependecia de S con respecto a U est´a dada por la curva de la Figura 12.2.

Figura 12.2: Relaci´on fundamental de curva c´oncava. Este sistema es estable.

Aqu´ı la entrop´ıa total cambiar´a de su valor inicial 2S(U, V, N ) a S(U +∆U, V, N )+S(U −∆U, V, N ), el cual, en este caso, es menor que el inicial. La condici´on de estabilidad est´a dada entonces por la concavidad de la curva de entrop´ıa: S(U + ∆U, V, N ) + S(U − ∆U, V, N ) ≤ 2S(U, V, N )

Sea ∆U → 0 :    1 ∂ 2 S ∂S (∆U )2 + · · · S(U + ∆U, V, N ) = S(U, V, N ) + ∆U + ∂U V,N 2! ∂U 2 V,N     ∂S 1 ∂ 2 S S(U − ∆U, V, N ) = S(U, V, N ) − ∆U + (∆U )2 + · · · ∂U V,N 2! ∂U 2 V,N 

Luego  S(U + ∆U, V, N ) + S(U − ∆U, V, N )

= 2S(U, V, N ) + 

⇒ 0 ≥ S(U + ∆U, V, N ) + S(U − ∆U, V, N ) − 2S(U, V, N ) ≈ Por lo tanto

∂ 2S ∂U 2



(∆U )2

 ∂ 2 S (∆U )2 + · · · ∂U 2 V,N

169

Rodrigo Ferrer



∂ 2S ∂U 2

 ≤0

Al hacer c´alculos con mec´anica estad´ıstica (o a partir de datos experimentales), se podr´ıa obtener una ecuaci´on fundamental que no satisface la condici´on de concavidad. Podemos constru´ır una ecuaci´on fundamental estable.

Figura 12.3: Envolvente de las tangentes sobre la curva de la ecuaci´on fundamental, termodin´amicamente estable.

[ y EF [ \ es un En la Figura 12.3, las curvas ABC G procesos de estabilidad local, la curva CDE \ proceso de inestabilidad local, y el punto H es el punto de separaci´on de fase. En CDE,  2  ∂ S ≥0 ∂U En el espacio tridimensional S(U + ∆U, V + ∆V, N ) + S(U − ∆U, V − ∆V, N )

≤ 2S(U, V, N )

⇒ S + SU ∆U + SV ∆V + 2!1 SU U (∆U )2 + 2!1 SV V (∆V )2 + SU V ∆U ∆V + S − SU ∆U − SV ∆V + 2!1 SU U (∆U )2 + 2!1 SV V (∆V )2 + SU U ∆U ∆V ⇒ SU U (∆U )2 + SV V (∆V )2 + 2SU V ∆U ∆V ≤ 0

para ∆U y ∆V arbitrarios. Sea ∆U = 0, entonces SU U ≤ 0 Sea ∆V = 0, entonces SV V ≤ 0. Con esto podemos escribir la u ´ltima implicancia en la forma:

≤ 2S

170

CAP´ ITULO 12

´ ESTABILIDAD EN LOS SISTEMAS TERMODINAMICOS

(SU U ∆U + SU V ∆V )2 + (SU U SV V − SU2 U )(∆V )2 ≥ 0

(12.1)

SU U SV V − SU2 V ≥ 0

(12.2)

y tenemos

Consideremos la segunda derivada de la entrop´ıa: 

∂ 2S ∂U 2

 = V,N

= = =

  ! ∂ ∂S ∂U 2 ∂U V,N V,N    ∂S 1 ∂U T V,N   1 ∂T − 2 T ∂U V,N 1 − 2 T Cv

Por lo tanto Cv > 0

Esta u ´ltima es una restricci´on para la estabilidad.

12.1 Condiciones de Estabilidad para los Potenciales Termodin´amicos 171

Rodrigo Ferrer

12.1.

Condiciones de Estabilidad para los Potenciales Termodin´ amicos

El m´ınimo en la energ´ıa exige una convexidad en la curva: U (S + ∆S, V + ∆V, N ) + U (S − ∆S, V − ∆V, N ) ≥ 2U (S, V, N ) 

∂ 2U ∂S

 ⇒

∂T ∂S



 ≥0 ; V,N



∂ 2U ∂V

 ≥0

;

V,N

∂P ∂V

 ≥0 S,N

 ≤0 S,N

Adem´as, de (12.1), USS (∆S)2 + UV V (∆V )2 + 2USV ∆S∆V ≥ 0

Pasemos a caracterizar los Potenciales Termodin´amicos. ◦ Considere F = U − T S = U [T ], donde  T =

∂U ∂S





∂S ∂T

 −S =

; V,N

∂F ∂T

 V,N

Luego, 

 =− V,N

∂ 2F ∂T 2

 V,N

Pero T  ⇒  ⇒



∂ 2U ∂S 2



∂U ∂S



2

=

∂T ∂S



∂S ∂T



 V,N



∂ U ∂S 2 V,N  2  ∂ F 1 = − =  2  2 ∂ U ∂T V,N ∂S 2 V,N =

V,N

V,N



 ≥ 0 (convexo)



V,N

∂ 2F ∂T 2

 ≤0

(c´oncava).

(12.3)

V,N

Adem´as 

∂ 2F ∂V 2

 ≥0 T,N

(convexa)

(12.4)

172

CAP´ ITULO 12

´ ESTABILIDAD EN LOS SISTEMAS TERMODINAMICOS

◦ Considere H=U+PV=U[P], donde  P =−

∂U ∂V



 ;

V =

S,N

∂H ∂P

 S,N

Luego 

∂V ∂P



 =

S,N

∂ 2H ∂P 2

 S,N

Pero

⇒  ⇒



∂ 2U ∂V 2

∂U ∂V



2

= −

P 

 ∂P ∂V



∂V ∂P



= − S,N

S,N

 S,N

∂ U ∂V 2

 S,N

1 = − 2  = ∂ U ∂V 2 S,N 

 ≥0

(convexo)



S,N

∂ 2H ∂P 2



∂ 2H ∂P 2

 V,N

 ≤0

(c´oncava)

(12.5)

S,N

Adem´as 

∂ 2H ∂S 2

 ≥0

(convexa)

(12.6)

P,N

◦ Considere G = U + P V − T S = U [T, P ], donde  −S =

∂G ∂T



 ;

P,N

V =

∂G ∂P

 T,N

Luego     2  ∂ G 1 ∂V   = = − 2    2  ∂ U ∂P T,N ∂P T,N    ∂V 2 T,N    2  ∂S ∂ G 1     = − =  2   ∂ U ∂T P,N ∂T 2 P,N    ∂S 2 P,N

12.1 Condiciones de Estabilidad para los Potenciales Termodin´amicos 173

Rodrigo Ferrer

Luego   2  ∂ 2U ∂ G ≥ 0 (convexo) ⇒ ≤ 0 2 2 ∂V ∂P T,N T,N  2   2  ∂ U ∂ G ≥ 0 (convexo) ⇒ ≤ 0 2 ∂S P,N ∂T 2 P,N



En resumen, los potenciales termodin´amicos U, F, H, G son funciones convexas de los par´ametros extensivos, y c´oncavas de los par´ametros intensivos.

Figura 12.4: U vs. S, U vs. V, U vs. T y U vs. P. La energ´ıa es convexa de los par´ametros extensivos, y c´oncava de los par´ametros intensivos.

174

CAP´ ITULO 12

´ ESTABILIDAD EN LOS SISTEMAS TERMODINAMICOS

Parte IV Cambios de Fase

Cap´ıtulo 13 Transiciones de Fase de Primer Orden 13.1.

Ecuaci´ on de la Presi´ on de Vapor

Las mol´eculas interact´ uan y ocupan volumen. Considere el proceso isot´ermico siguiente:

Figura 13.1: Cambio de fase: l´ıquido → gas.

En este proceso, hubo un intervalo en el cual el compuesto estuvo en fase l´ıquida, digamos hasta un volumen V1 para despu´es pasar a ser gaseoso, desde V2 en adelante. La situaci´on se ejemplifica en el siguiente gr´afico Usemos una de las siguientes notaciones: ◦ volumen molar de la fase l´ıquida: vl =

Vl n

◦ volumen molar de la fase gaseosa: vg =

Vg n

o bien • volumen por a´tomo de la fase l´ıquida: vl =

Vl N

• volumen por a´tomo de de la fase gaseosa: vg =

Vg N

178

CAP´ ITULO 13

TRANSICIONES DE FASE DE PRIMER ORDEN

Figura 13.2: Interfase en un proceso isot´ermico. La presi´on y temperatura se mantienen constantes en ese proceso.

´ Atomo He Ne Ar Kr Xe

Tc /o K 5.2 44.4 151.1 210 289.7

Mol´ecula H2 N2 O2 H2 O CO2

Tc /o K 33.2 126 154.3 647.1 304.2

Tabla 13.1: en donde n es el n´ umero de moles de ´atomos, y N es el n´ umero de a´tomos. Equilibrio de Fases ≡ Dos fases est´an en equilibrio cuando se encuentran en equilibrio t´ermico, difusivo y mec´anico; es decir, cuando sus temperaturas son iguales, sus potenciales qu´ımicos son iguales y cuando sus presiones son iguales. Caso l´ıquido-vapor: Tl = Tg ; µl = µg ; Pl = Pg Nota:Recordemos que este estado global de equilibrio corresponde a un estado de m´axima entrop´ıa S(U, V, N ).

13.2.

Curva de Coexistencia de Fases

Si agregamos o quitamos calor a una de las fases del sistema, cambiar´an la temperatura, el potencial qu´ımico y la presi´on. En un cambio de fase isot´ermico, la presi´on permanece constante (ver Figura 13.3), de modo que agregar calor al sistema significa cambiar s´olo el potencial qu´ımico. Hemos demostrado que las mol´eculas difunden de potenciales qu´ımicos mayores a potenciales menores, hasta que se alcanza el equilibrio difusivo. Para la interfase l´ıquido-vapor, aumentar µl se traduce en hacer que mol´eculas en fase l´ıquida pasen a la fase vapor:

13.2 Curva de Coexistencia de Fases

Rodrigo Ferrer

Figura 13.3: Punto cr´ıtico en un cambio de fase.

 ∆µl = −T

∂Sl ∂Nl

 Ul ,Vl

Considerar el estado con p = po y T = To . En el equilibrio se tiene µg (po , To ) = µl (po , To )

;

µg (po + dp, To + dT ) = µl (po + dp, To + dT )

Expandiendo al primer orden,      ∂µg ∂µg   dp + dT  µg (po + dp, To + dT ) = µg (po , To ) + ∂p  T  ∂T p ∂µl ∂µl   dp + dT  µl (po + dp, To + dT ) = µl (po , To ) + ∂p T ∂T p  ⇒

∂µg ∂p



 dp +

T

 ⇒

dp dT

∂µg ∂T



 p



   ∂µl ∂µl dT = dp + dT ∂p T ∂T p     ∂µl ∂µg − ∂T p ∂T p    =  ∂µg ∂µl − ∂T p ∂p T

179

CAP´ ITULO 13

180

TRANSICIONES DE FASE DE PRIMER ORDEN

Ahora bien. U + P V = µN + T S y G = H − T S. Luego G = µN y dH − T dS − S dT dU + P dV + V dP − T dS − S dT T dS − P dV + V dP − T dS − S dT V dP − S dT     ∂G ∂G V = ; S=− ∂P T,N ∂T P,N

dG = = = = ⇒

Definiendo v ≡

S V y s ≡ , tenemos N N

1 N



∂G ∂p



 =v= T,N

∂µ ∂p

 ; T

 ⇒

1 N

dp dT

 =



∂G ∂T



 = −s =

p,N

sg − sl vg − vl

∂µ ∂T

 p

(13.1)

sg − sl es la diferencia de entrop´ıa de las fases por part´ıcula. Cada vez que una part´ıcula pasa del l´ıquido al gas, hay un aumento de entrop´ıa del sistema, igual a sg − sl . Lo mismo sucede para la diferencia de volumen molar: cada vez que una mol´ecula pasa del l´ıquido al gas, el volumen espec´ıfico del sistema aumenta en vg − vl Observaci´ones: ◦

dP dT

no se deduce de la ecuaci´on de estado.

◦ Ng y Nl cambian a medida que las part´ıculas cambian a otra fase. Siempre se cumple Ng +Nl = N. Para sacar una mol´ecula del l´ıquido y ponerla en estado gaseoso, hay que darle energ´ıa. Esto baja la temperatura del l´ıquido, haciendo que el proceso no sea isot´ermico. Para solucionar esto, debemos darle calor al sistema. En un proceso quasi-est´atico, este calor es T (sg − sl ) ≡ `

(13.2)

llamado calor latente de vaporizaci´on. Definiendo ∆v = vg − vl , obtenemos dp ` = dT T ∆v La ecuaci´on (13.3) es conocida como la Ecuaci´on de Clausius-Clapeyron. Aproximaciones:

(13.3)

13.3 Ecuaci´on de Van der Waals

Rodrigo Ferrer a) vg >> vl



∆v ≈ vg =

181

vg Vg . Si p = 1[atm], = 103 . Ng vl

b) En la fase gaseosa tenemos un gas ideal: pVg = Ng kB Tg . Luego kB T p `p dp = ⇒ dT kB T 2 d ` ⇒ ln p = dT kB TZ2 l 1 `o ⇒ ln p = + cte dT = − 2 kB T kB T ∆v



p

=

− k`oT

= po e

B

Figura 13.4: Gr´afico de la Presi´on en funci´on de la Temperatura.

Transici´ on de Primer Orden

Figura 13.5: De izquierda a derecha: Potencial de Gibbs Molar vs. Temperatura, volumen Molar vs. Temperatura, Entrop´ıa Molar vs. Temperatura.

13.3.

Ecuaci´ on de Van der Waals

Ecuaci´on emp´ırica que funciona bien en las fases l´ıquida, vapor y cerca del punto cr´ıtico.

182

CAP´ ITULO 13

TRANSICIONES DE FASE DE PRIMER ORDEN



donde v =

V n

p+

a (v − b) = RT v2

(13.4)

y n es el n´ umero de moles de mol´eculas.

Figura 13.6: Isotermas de Van der Waals. T1 < T2 < T3 . . .

En funci´on del volumen total V , la ecuaci´on de Van der Waals toma la forma  p+a

 n 2  V

(V − nb) = nRT

(13.5)

Definamos

pb ≡

p pc

;

V Vb ≡ Vc

;

T Tb ≡ Tc

Vc ≡ 3nb ;

RTc ≡

(13.6)

donde

pc ≡

a 27b2

;

Con estas definiciones, escribimos (13.5) como

8a 27b

(13.7)

13.3 Ecuaci´on de Van der Waals

Rodrigo Ferrer

 pc ⇒



183

   p a  n 2 V nb + Vc − = nRT pc pc V Vc Vc    p 27b2 2 V 1 nRT + 2 n − = p V V 3 pc Vc  c  c   3  V 1 nR p − = 27b2 T  +  2  pc V 3 3anb V c Vc







  3  V 1 p −  +  2  pc Vc 3 V



   1 3 Vb − pb + 3 Vb 2

=

8 T · 3 Tc

=

8b T 3

Vc

pb =



8Tb 3 − b b 3V − 1 V 2

(13.8)

De esta u ´ltima ecuaci´on tenemos   ⇒

∂ pb ∂ Vb



∂ 2 pb ∂ Vb 2

24Tb 6 + 2 Vb 3 (3Vb − 1) 48 · 3Tb 18 = − (3Vb − 1)3 Vb 4 = −

Tb

 Tb

Extremos: 

∂ pb ∂ Vb

 =0



Tb

24Tb 6 = (3Vb − 1)2 Vb 3

(13.9)

Punto de inflexi´on (primera y segunda derivadas iguales a cero): 

∂ 2 pb ∂ Vb 2

 =0 Tb



8Tb (3Vb 2 − 1)3

=

Dividiendo (13.9) con (13.10), tenemos 3(3Vb − 1) = 6Vb ⇒ Vb = 1 24Tb = 6 ⇒ Tb = 1 4

1 b V4

(13.10)

184

CAP´ ITULO 13

TRANSICIONES DE FASE DE PRIMER ORDEN

Reemplazando en la ecuaci´on de Van der Waals (13.8), obtenemos pb =

8 −3=1 3

(13.11)

Q1 , Q2 , M1 , M2

V

Q1

1 2

Q2

M1 V1 ,

1 2

M 2V 2 , M 1gh 1 , M 2 gh 2

M1 M2 Q Q , k 1 2 −G r r 12 12 Figura 13.7: Gr´afico pb vs Vb

13.4.

Transiciones de Fase en Sistemas Monocomponentes

Los criterios de estabilidad deben ser satisfechos por la ecuaci´on fundamental de todo sistema que permanezca estable y homog´eneo. Si estos criterios no se satisfacen, el sistema se separa en dos o m´as porciones. Esta separaci´on se llama Transici´on de Fase. Estas distinguibles fases s´olidas, designadas por “hielo I”, “hielo II”, “hielo III”,..., difieren en la estructura de cristal y en escencialmente todas las propiedades termodin´amicas (como compresibilidad, capacidad cal´orica molar, y varios potenciales molares). En la Figura 13.8 se muestra el “diagrama de fase” del agua. - Cubos de hielo en la hielera (1[atm]). - Helio l´ıquido a 2.18 K. He I, He II (1[atm]). - Transiciones polim´orficas en s´olidos. - Transiciones magn´eticas. Ecuaci´ on de Van der Walls: Representa bien a l´ıquidos que exhiben transiciones l´ıquido-gas, pero en regiones sobre y bajo el punto de ebullici´on. Sobre la temperatura cr´ıtica, es una correcci´on al gas ideal. Para temperaturas menores, es razonable para el estado l´ıquido y para el estado gaseoso a bajas presiones; pero falla al mantener constante la presi´on del vapor en la transici´on l´ıquido gas. En la materia existen fuerzas moleculares y dipolos. Las mol´eculas ocupan un volumen y se atraen, F ∝ ρ. por lo que hay una disminuci´on de N La estabilidad intr´ınseca requiere

Rodrigo Ferrer

13.4 Transiciones de Fase en Sistemas Monocomponentes

185

Figura 13.8: Diagrama de fase del agua. La regi´on de estabilidad de la fase gaseosa est´a representada por una angosta e indiscernible l´ınea horizontal.



∂P ∂V

 <0 V

Esta condici´on se pierde en la regi´on F → K → M de la Figura 13.9. Por lo tanto, hay una transici´on de fase.

186

CAP´ ITULO 13

TRANSICIONES DE FASE DE PRIMER ORDEN

Figura 13.9: Isoterma particular de la ecuaci´on de Van der Waals.

Perspectiva interesante: dµ = −s dT + v dP

Gibbs-Duhem

Para T constante, tenemos Z µ=

v(P ) dP + φ(T )

Fijando µA arbitrario, Z

B

µB = µA +

v(P ) dP A

De este modo, obtenemos la Figura 13.10. Esta figura, que representa µ versus P , puede ser considerada como una secci´on plana de la representaci´on tridimensional de µ versus P y T , como se muestra en la Figura 13.11. Aqu´ı se muestran cuatro secciones diferentes de temperatura constate de la superficie µ, que corresponden a cuatro isotermas. Tambi´en se aprecia que el “loop” cerrado de las curvas µ versus P , las cuales resultan del hecho de que v(P ) es triple-evaluado en P (ver Figura 13.9), desaparecen para altas temperaturas, lo cual concuerda con la Figura 13.6. Finalmente, notamos que la relaci´on µ = µ(T, P ) constituye una relaci´on fundamental para un mol del material, cuando el potencial qu´ımico µ es la funci´on de Gibbs por mol. Parecer´ıa entonces, de la Figura 13.11, que hemos logrado la construcci´on de la ecuaci´on fundamental a partir de una simple ecuaci´on de estado dada, pero cabe destacar que aunque cada una de las curvas de la superficie µ (en las distintos planos de temperaturas constantes de la Figura 13.11) tienen una constante aditiva φ(T ). Por lo tanto, no sabemos la forma completa de la superficie µ(T, P ), aunque podemos hacernos una imagen de sus escenciales propiedades topol´ogicas.

Rodrigo Ferrer

13.4 Transiciones de Fase en Sistemas Monocomponentes

187

Figura 13.10: Dependencia isot´ermica del potencial molar de Gibbs de la presi´on.

Recorrer esta curva con una fuente de temperatura y presi´on, cambiando quasi-est´aticamente la presi´on.

Figura 13.11: Dependencia isot´ermica del potencial molar de Gibbs de la presi´on y la temperatura.

Retomemos la situaci´on de la Figura 13.4. Concentremonos en una isoterma: Entre los puntos S y O de la Figura 13.12 podemos notar una alta pendiente. Este intervalo corresponde a la fase l´ıquida, donde 

∂P ∂V

 =− T

1 KT V

En la secci´on comprendida entre D y A, se aprecia una baja pendiente. Esta fase Z Ocorresponde al estado gaseoso. Los puntos O y D se obtienen de la condici´on µD = µO , es decir v(P ) dP = 0. D

Considere

188

CAP´ ITULO 13

TRANSICIONES DE FASE DE PRIMER ORDEN

Figura 13.12: La isoterma f´ısica de Van der Waals. La isoterma “fundamental” es SOMKFDA, pero la construcci´on de igual a´rea la convierte en la isoterma f´ısica SOKDA

Z

O

Z

F

Z

K

Z

O

v dP = 0

v dP + M

K

F

D

D

M

v dP +

v dP +

v dP =

Z

Luego Z

F

Z

F

v dP − D

Z

K

O

v dP −

v dP = K

Z

M

v dP M

´ ´ Por lo tanto, Area I = Area II. Hay un cambio discontinuo en el volumen molar en la vecindad de la presi´on PD = PK = PO . Esto puede modificar las propiedades del sistema, dependiendo del volumen molar. El sistema puede encontrarse en una u otra fase.

13.5.

La Discontinuidad en el volumen

Sea xD la fracci´on de un sistema en su fase menos condensada. Sea xO la fracci´on de un sistema en su fase m´as condensada. Luego xO + xD = 1. Llamemos vT al volumen molar promedio.

vT = xD vD + xO vO = xD vD + (1 − xD )vO = vO + xD (vD − vO ) Regla de la Palanca:

13.6 Discontinuidad en la Entrop´ıa

Rodrigo Ferrer

Figura 13.13: Gr´afico P vs. v para una transici´on de fase.

1 · vT

= (xD + xO )vT = xD vD + xO vO ⇒ xO (vT − vO ) = xD (vD − vT ) Por lo tanto, no hay una discontinuidad f´ısica en la transici´on.

13.6.

Discontinuidad en la Entrop´ıa

Sea S = S(U, V, N ). Tenemos

 T =

∂U ∂S

 V,N

⇒ T = T (U, V, N ) ⇒ U = U (T, V, N ) ⇒ S = S(T, V, N ) Luego  dS =

∂S ∂T

Para T y N constantes, tenemos



 dT +

V,N

∂S ∂V



 dV +

T,N

∂S ∂N

 dN T,V

189

190

CAP´ ITULO 13

TRANSICIONES DE FASE DE PRIMER ORDEN

 ∂S dV ∂V T,N   ∂s dv ∂v T Z D  ∂s s − so = dv ∂v T O  Z D ∂P dv ∂T v O

 dS = ⇒ ds = ⇒ ∆s = =

Figura 13.14: Discontinuidad en la entrop´ıa molar. El a´rea entre las isotermas adyacentes est´a relacionada con la discontinuidad en la entrop´ıa y, por lo tanto, con el calor latente.

´ Area =

Z

vD

Z

vD

P (T + ∆T, v) dv −

P (T, v) dv vO

ZvOvD

[P (T + ∆T, v) − P (T, v)] dv     ∂P = P (T, v) + ∆T − P (T, v) dv ∂T v vO  Z vD  ∂P dv = ∆T ∂T v vO = ∆T ∆s =

v Z OvD

Luego, el cambio en la entrop´ıa est´a dado por ∆s =

1 ´ Area ∆T

(13.12)

Como vimos en (13.2), este ∆s est´a asociado al flujo de calor entre el sistema y la fuente (reservoir):

13.6 Discontinuidad en la Entrop´ıa

Rodrigo Ferrer

191

∆q = T ∆s = ` Notaci´on: `DO = T (sD − sO ) `DO es el calor emitido por un mol de material, efectuando la transici´on desde D hasta O. L´ıquido ←→ S´olido `: calor latente de fusi´on L´ıquido ←→ Gas `: calor latente de vaporizaci´on S´olido ←→ Gas `: calor latente de sublimaci´on Tabla 13.2: Calor latente en los distintos cambios de fase. Si la fase O es m´as condensada, tendremos `DO = T (sD − sO ) > 0. El cambio, o discontinuidad, en la energ´ıa estar´a dado por

ąq = T ∆s ąw = −P ∆v ∆u = T ∆s − P ∆v

192

Temp oC T 0.01 5 10 15 20 25 30 35 40 45 50 55 60 65 70 75 80 85 90 95 100 105 110 115 120 125 130 135 140 145 150 155 160 165 170 175 180 185 190 195 200 205 210 215 220 225 230 235 240 245 250 255 260 265 270 275

CAP´ ITULO 13

Press kP a P 0.6113 0.8721 1.2276 1.7051 2.339 3.169 4.246 5.628 7.384 9.593 12.349 15.758 19.940 25.03 31.19 38.58 47.39 57.83 70.14 84.55

TRANSICIONES DE FASE DE PRIMER ORDEN

volumen Espec´ıfico Sat. Sat. L´ıquido Vapor vf vg 0.001 000 206.14

Tabla de Vapores1 Energ´ıa Interna Entalp´ıa Sat. Sat. Sat. Evap. Evap. L´ıquido Vapor L´ıquido uf uf g ug hf hf g 0.00 2375.3 2375.3 0.01 2501.3

Entrop´ıa Sat. Vapor hg 2501.4

Sat. L´ıquido sf 0.0000

Evap. sf g 9.1562

Sat. Vapor sg 9.1562

tabla de vapores

1 De R. E. Sonntag y G. J. Van Wylen, “Introduction to Thermodynamics, Classical and Statistical”, Jhon Wiley & Sons, New York, 1982 (adaptado por J. H. Keenan, F. G. Keyes, P. G. Hill, y J.G. Moore, Tablas de Vapor, John Wiley & Sons, New York, 1978).

Rodrigo Ferrer

13.6 Discontinuidad en la Entrop´ıa Tit hojas

280 285 290 295 300 305 310 315 320 330 340 350 360 370 374.14 Tabla de Vapores Tabla 13.3: Propiedades de las Fases L´ıquida y Gaseosa en la Curva de Coexistencia del Agua.

193

194

13.7.

CAP´ ITULO 13

TRANSICIONES DE FASE DE PRIMER ORDEN

La Ecuaci´ on de Clausis Clapeyron

Como vimos en (13.3), la ecuaci´on de Clausius Clapeyron est´a dada por ` dp = dT T ∆v

(13.13)

donde ` es el calor latente. Podemos apreciar esta variaci´on de la presi´on con respecto a la temperatura en el siguiente gr´afico de isotermas:

Figura 13.15: Clasificaci´on de fases del plano P − v.

Analicemos lo que sucede con la presi´on de un gas cuando se aumenta su temperatura. La situaci´on se representa en el gr´afico siguiente, y m´as detallado en la Figura 13.8:

Figura 13.16: Punto cr´ıtico de un gas.

La pendiente de esta curva puede ser relacionada con el calor latente y la discontinuidad en el volumen:

13.7 La Ecuaci´on de Clausis Clapeyron

Rodrigo Ferrer

PB − P A = PB 0 − P A 0

infinitesimal dP

TB − TA = TB 0 − TA0

infinitesimal dT

195

El equilibrio exige µA = µA0 , µB = µB 0 . Luego dµ = µB − µA = µB 0 − µA0 = dµ0 Por Gibbs Duhem, tenemos dµ = −s dT + v dP . Luego ⇒ −s dT + v dP = −s0 dT + v 0 dP ⇒ (v − v 0 ) dP = (s − s0 ) dT dP s0 − s ∆s ⇒ = 0 = dT v −v ∆v y como ` = T ∆s, ` dP = dT T ∆v

(13.14)

196

CAP´ ITULO 13

TRANSICIONES DE FASE DE PRIMER ORDEN

Parte V Ap´ endice

Ap´ endice A Teoremas Matem´ aticos A.1.

Continuidad en Rn

En termodin´amica son de particular inter´es las funciones del tipo f (x, y, z) cuya imagen es un escalar. Estas funciones son llamadas funciones escalares y tienen la siguiente propiedad: Teorema 1 Sea f : R3 → R continua, con derivadas continuas. Entonces las derivadas cruzadas son iguales. Las posibles segundas derivadas son ∂ 2f ∂x2

∂ 2f ∂x∂y

∂ 2f ∂x∂z

∂ 2f ∂y 2

∂ 2f ∂y∂x

∂ 2f ∂y∂z

∂ 2f ∂z 2

∂ 2f ∂z∂x

∂ 2f ∂z∂y

(A.1)

Pero, al usar el teorema, ∂ 2f ∂ 2f = , ∂x∂y ∂y∂x

∂ 2f ∂ 2f = , ∂x∂z ∂z∂x

por lo que las derivadas se reducen solamente a seis:

∂ 2f ∂ 2f = ∂y∂z ∂z∂y

(A.2)

200

´ APENDICE A

´ TEOREMAS MATEMATICOS

∂ 2f ∂x2

∂ 2f ∂x∂y

∂ 2f ∂y 2

∂ 2f ∂x∂z

∂ 2f ∂z 2

∂ 2f ∂y∂z

(A.3)

Teorema 2 Sea y : R → R tal que y 0 (x) 6= 0 ∀ x. Entonces dy 1 = dx dx dy

(A.4)

Si y 0 (x) 6= 0 ∀ x, entonces existe la funci´on inversa de y, llamada y ← . Luego (y ◦ y ← )(x) = (y(y ← (x))) = x ⇒ (y ◦ y ← )0 (x) = 1 ⇒ y 0 (y ← (x)) · y ←0 = 1 1 ⇒ y ←0 = 0 ← y (y (x)) 1 ⇒ y ←0 = 0 (y ◦ y ← )(x) Introduciendo la notaci´on y ← (x) = x(y) como la funci´on inversa y y 0 (x) = tenemos que la ultima ecuaci´on la escribimos como dx 1 = dy dy dx El resultado se extiende a Rn , obteniendo 

∂f ∂xi

 =

A.2.

Expansi´ on de Taylor

A.3.

Funciones Compuestas

A.4.

Funciones Impl´ıcitas

1  ∂xi ∂f

dy dx

como la derivada,

(A.5)

(A.6)

Ap´ endice B Constantes y Conversi´ on de Unidades Descripci´on Cte. de Boltzmann Cte. de Plank Cte. Univ. de los Gases Ideales N´ umero de Avogadro

S´ımbolo kB h R NA = R/kB

Valor   1.3807 × 10−23 oJK −34 6.626 × 10  J [J · s] 8.3143 mol o K 6.022 × 1023

Tabla B.1: Constantes termodin´amicas.

1 1 1 1 1 1 1

[erg] [Joule] [cal] [Btu] [L · atm] [Watt · h] [Hp · h]

1 [erg] 1 107 4.19 · 107 1.06 · 1010 1.01 · 109 3.6 · 1010 2.68 · 1013

1 [Joule] 10−7 1 4.19 1054.35 101.3 3600 2.68 · 106

1[cal] 2.39 · 10−8 0.2389 1 252 24.21 860.42 6.42 · 105

1 [Btu] 9.48 · 10−11 9.48 · 10−4 0.003968 1 0.09607 3.4144 2546.14

1 [L · atm] 9.87 · 10−10 0.009869 0.04131 10.41 1 35.524 2.5 · 104

1 [Watt · h] 2.78 · 10−11 2.78 · 10−4 0.001163 0.2930 0.02815 1 745.7

Tabla B.2: Tabla de conversi´on de unidades energ´eticas.

1 [Hp · h] 3.72 · 10−14 3.724 · 10−7 1.559 · 10−6 3.929 · 10−4 3.775 · 10−5 1.34 · 10−3 1

202

´ APENDICE B

´ DE UNIDADES CONSTANTES Y CONVERSION

Bibliograf´ıa [1] M.W. Zemansky, Heat and Thermodynamics. [2] H. Callen, Thermodynamics. [3] Enrico Fermi, Termodin´amica. [4] A.B. Pippard, Classical Thermodynamics. [5] Kittel, Thermal Phisics.

Cosas importantes ◦ IDEA: COLOCAR UNA SECT ION DE PROBLEMAS AL FINAL DE CADA CAPITULO. ◦ Comentarios: • Hay problemas con el experimento de Rush. No se si la ecuaci´on diferencial no homogenea conserva la frecuencia. • No se pq en teoria cin´etica el cambio en la presion es - el cambio en el momentum. • La parte de transiciones de fase est´a desordenada (se define dos veces calor latente).

Related Documents


More Documents from "irving66"